Clinical Consequences of Iron Deficiency (Part Three)

So, you can live in a room filled with oxygen (it is said that without oxygen there is no life due to its significant structural and functional role in the organism), but without iron, oxygen will not reach the cells of the organism to perform its functions (where the most important is exactly oxidative phosphorylation that ensures the production of large amounts of ATP in mitochondria), so without iron there will be no life (as an oxygen transporter and accelerator of vital reactions of the organism).

These functions of iron should be emphasized, because iron deficiency and its consequences, one of which is Iron-Deficiency Anemia, are the most common in the World (both deficiency and anemia), are very common in medical practice, attempted to be treated by everyone, but in most cases not properly.

Treat Iron Deficiency and Iron-Deficiency Anemia properly, or refer immediately to a specialist. Keep in mind, that small, repeated blood losses, resulting in Iron Deficiency and subsequently Iron-Deficiency Anemia, can be caused by a cancer of the mucosa of all body systems.

In our medical practice and popular treatments, anemia is treated (iron deficiency in most cases is not evaluated at all, despite and as we will see below it has a clinic, so it is not so hidden), while the cause (in this case cancer which is malignant by definition) develops normally. And then to think of another fact, that anemia (regardless of what type and origin it is) should be corrected with foods containing iron (liver, red wine, etc.), mistakes follow one another.

Iron-Deficiency Anemia is the most common anemia in the World and very common among females in the reproductive period due to the continuous hemorrhage of the menstrual cycles. So, in general, it is based on the loss of iron for various reasons (mostly through blood), resulting in the reduction of its reserves in the organism (accurately assessed through ferritinemia) and then clinically and laboratory manifestations appear.

This means that if the cause is removed and the stores are replenished (through treatment that continues for at least 3 months) the anemia is cured and does not recur. If the cause persists, despite the replenishment of the stores, the anemia will recur.

Clinical consequences of iron deficiency in the organism

Stages of iron deficiency. Laboratory tests help in the staging of IDA.

Stage 1. When the organism is in a status of negative iron balance, there will initially be a decrease in iron stores in the BM (ferritin and hemosiderin) that will be mobilized for the synthesis of Hb (and other enzymes containing iron). The compensatory increase in iron absorption will cause an increase in the binding capacity of iron in transferrin. When deposits decrease, iron absorption from the GI mucosa increases. On the other hand, clinic and laboratory changes have not yet appeared (and serum iron continues to stay in almost normal values). Hb and serum iron (iron stores) remain normal, but ferritinemia drops < 20 ng/mL (< 15 μg/l), in BM with Perls staining no iron is detected.

Stage 2. Normal erythropoiesis is inhibited. Although the level of transferrin (iron-binding capacity) is increased, serum iron decreases, transferrin saturation decreases. Erythropoiesis is inhibited when serum iron drops < 50 μg/dL (< 9 μmol/L) and transferrin saturation falls less than 16% (15%). The level of serum ferritin receptors increases (> 8.5 mg/L).

Stage 3. Anemia develops with normal appearance of RBC and normal indicators. There will be development of specific ferrodeficiency erythropoiesis, but Hb, MCV, MCH continue to remain still in normal values (it takes 3 months for normochromic, normocytic RBC to be replaced with hypochromic, microcytic RBC). In this phase (thus low ferritinemia and lacking clinic) if iron is added to therapy, laboratory changes are quickly corrected. Thus when we observe hypochromia, microcytosis iron deficiency has been installed for at least 3 months.

If the cause persists, thus the iron balance is negative and no iron medication is taken, there will be development of the clinic and laboratory of iron-deficiency anemia. Thus, for iron-deficiency anemia to appear, the Fe stores must be completely empty (ferriprive, deprived, in absence of iron, from the deficiency of iron).

If the stores are empty, but anemia has not set in then take medication until the stores are replenished, thus at least 6 weeks.

Stage 4. Microcytosis and hypochromia develop.

Stage 5. Iron deficiency affects tissues with specific symptoms and signs. In this phase, all signs of iron-deficiency anemia appear, which are general signs and symptoms of the anemic syndrome, but there are also some specific signs and symptoms of the lack of iron in the organism such as:

The clinical consequences of iron deficiency are; non-hematological because as we said above, initially Iron Deficiency appears (evaluated with hypoferritinemia) and if this condition persists (thus the cause/causes are not eliminated and the Deficiency corrected) hematological ones will also appear (Anemic Syndrome - Iron-Deficiency Anemia).

1. Clinic of non-hematological changes from iron deficiency (Clinical manifestations that are not related to anemia, but to iron deficiency).

A. From Iron Deficiency due to the compromise of enzyme function and myoglobin there will be fatigue and breakdown. Iron deficiency and superimposed anemia limit physical performance, coping with work and spontaneous physical activity, resulting in decreased productivity and economic loss. It has been observed that in athletes with iron deficiency, but with normal Hemoglobin (nonanemic iron deficiency) the use of iron preparations for correcting its stores, will be accompanied by improvement in work performance and of VO2max.

Iron-Deficiency Anemia decreases work performance where muscle strength is required, compared to healthy people in anaerobic metabolism. Fatigue and reduced ability to cope with heavy work are related to the decrease in Hb, but they are not proportional to the degree of anemia (it depends on the speed of anemia installation and the age of the patient). This means that they are also related to the function of proteins (enzymes) that require iron to perform their function, thus they are more related to the deficiency of iron in iron-enzymes of cellular respiration than from anemia.

B. Epithelial changes (the cells that suffer the most from the lack of iron are those with rapid proliferation such as BM cells, skin epithelium, and mucous membranes. Nonetheless, they are not specific only to the lack of iron, but also in other deficiencies such as Vitamin B12, Folic Acid, Vitamin B6, and in diseases that are not related to micronutrient and vitamin deficiencies).

Hair loss. Hair becomes thin, fragile, and falls out. Thus, the hair does not fall out from anemia, but from iron deficiency. If a patient comes complaining about hair loss, ferritinemia should also be searched.

Cracking of the corners of the mouth. Angular stomatitis (cheilosis) is less specific for iron-deficiency anemia compared to changes in the nails, because it also appears in cases of riboflavin (Vitamin B2) and pyridoxine (Vitamin B6) deficiencies.

Nails initially are thin, fragile, with longitudinal striations and later koilonychia appears (thus taking the shape of a spoon).

Atrophic Glossitis. Shiny, pale, wet, shiny, flat (smoothed) tongue due to atrophy of the papillae, with various degrees of redness. They have burning, pain in different degrees, which can be spontaneous and after eating food and liquids. The changes in the epithelium of the tongue return after 1-2 weeks from the start of treatment.

Dry mouth. Iron Deficiency hampers the protection provided by the Peroxidase System (H2O2) in saliva and decreases the quantity of secreted saliva (dry tongue).

Sideropenic dysphagia (difficulty in swallowing food). Dysphagia is initially for solid foods and later also for liquid ones. The most pronounced changes appear in the place of passage from the oropharynx to the esophagus and can be localized in a certain area, but can also surround the entire diameter and causing stricture. The cause of these specific signs is related to the deficiency of Fe in the epithelial enzymes. Iron-deficiency anemia due to the consumption of Fe in epithelial enzymes becomes also a cause for the development of atrophic gastritis.

C. Neuromuscular System. Even a mild degree of iron-deficiency anemia is accompanied by involvement of muscle function (evaluated with exercise tests). Muscles suffer due to the reduction of their oxygenation not only from the deficiency of hemoglobin but also of myoglobin. The patient complains of quick muscle fatigue. Work performance, prolonged activity are also affected and improve when adequate treatment of anemia is done.

This article has been sent by:
This article has been read 2378 times.
Komente nga lexuesit

if the iron will be high what will happen???

Sent by merikler mucorovi, më 07 February 2014 në 10:01

Thank you for the accurate information

Sent by duro dalipi, më 07 June 2014 në 10:29

Thank you, I am pleased, I understood well, even though I am not a doctor of this specialty, but I am a dentist, I was interested

Sent by sabah kakeli, më 06 November 2014 në 20:33

YOU ARE VERY clear, you express the desire for patient control with your way of communication, the most important thing is that you give the patient both seriousness and optimism.! Thank you for the accurate information, MJ.STOMAT.S.KAKELI

Sent by sabah kakeli, më 07 November 2014 në 19:57

Thank you for the advice, you are wonderful

Sent by Aevie gaahi, më 20 September 2016 në 15:30

Hello, the goal is for people and many doctors to understand and appreciate things as they should be

Replay from Dr. Shk. Sotiraq Lako, më 22 October 2016 në 03:17

Hello doctor, I wanted an opinion. I am a 23-year-old girl and I am experiencing hair loss. I have done blood tests, and besides an iron deficiency, I haven't done a ferritin analysis and the male hormone level is 1.4 since I did a biopsy from the scalp. I have been treated for 1 year with jasmine, and today the Greek doctor suggested I take a medication that is used for prostate in men (the reason is to lower the male hormone). My question is, how much does the lack of iron influence hair loss? Because the doctor in question claimed that the last medication can sometimes have severe effects on the liver. I would be very grateful if you could advise me on how to proceed

Sent by Marinela, më 17 October 2017 në 16:36

Hello Marinela, one of the reasons hair falls out is due to a lack of iron. The best indicator of iron stores is Ferritinemia. For women, if it is < 20 ng/ml, it is considered an iron deficiency and of course, you need to be treated with iron. Ferritinemia can be affected by Acute Inflammation, therefore in some cases when we suspect an iron deficiency (there are complaints) and ferritinemia appears normal or elevated, we also do Serum Iron and TIBC to determine the Transferrin Saturation which if in a woman is < 15 % it is certain there is an iron deficiency. Iron deficiency is completely correctable, which means if it is corrected and the hair does not fall out anymore, it indicates that their loss was caused by the iron deficiency. But if ferritinemia is corrected and the hair still falls out, it indicates that there are other causes and the specialist doctor is the dermatologist

Replay from Dr. Shk. Sotiraq Lako, më 18 October 2017 në 07:33

Hello,
The information was very valuable.
I wanted to ask about something.
I've been taking medication for 6 months to cover the iron deficiency.
Before I realized I had an iron deficiency, I had nosebleeds.
After treatment and improvement, I didn't have any more nosebleeds. After I got the test results (every element was in the normal range), the nosebleeds started again and before this happened, I felt tired. Do you think there is a connection between the nosebleeds and anemia, and how concerning could this be? I am still taking medication *tot'hema

Sent by ana maria, më 03 January 2018 në 16:56

Hello Ana Maria, anemia is not a cause of bleeding from the nose. You will go to an ENT doctor to clarify the possibility of a local injury to the blood vessels. If there is, you will continue follow-up with the ENT doctor. Repeated hemorrhage can cause iron deficiency and iron deficiency and anemia. But if the amount of blood is large, it can directly cause anemia from blood loss. You will have complete blood count + ferritinemia. If Hb > 12 gr/dl and Ferritinemia around 60 ng/ml, you do not have problems and do not need treatment. If Hb > 12 gr/dl and Ferritinemia < 20 ng/ml, you have iron deficiency and should be treated for 2 months with therapeutic doses of iron (200 mg elemental iron/day). If you also have anemia (Hb < 12 gr/dl), the treatment lasts at least 3 consecutive months. Ferrous sulfate is not a good preparation for replenishing iron stores

Replay from Dr. Shk. Sotiraq Lako, më 04 January 2018 në 02:22

Hello Doctor! I did some tests because I felt a bit tired and uninterested in everything, I feel quite depleted. Also, when I rest, I have a pain or a tickling sensation in all my bones. My hemoglobin was 11.3 and ferritin 5.3. Today, I started taking iron supplements, but I see in your comment that it is not suitable for replenishing iron stores. What do you recommend I take?

Sent by Manjola, më 10 January 2018 në 14:27

Hello Manjola, you currently have Mild Anemia from Iron Deficiency. Treatment with Tot'hema is not optimal (1 ampoule has 50 mg elemental iron). You will take for at least 3 consecutive months: 1-Heferol 350 mg 2 x 1 tablet/day. 2-Vitamin C 100 mg 2 x 1 tablet/day. 3-Vitamin B6 25 mg 2 x 1 tablet/day. Peripheral blood analysis will be repeated after 1 month and at the end of 3 months complete blood treatment and Ferritinemia. Only when ferritinemia is corrected (about 60 ng/ml), anemia should be corrected in about 1-1.5 months of treatment, you have been properly treated. And after ferritinemia is corrected, it is repeated after 5-6 months, to see stability. If it drops, it indicates that there is a cause and generally are blood losses with menstruations (if you have menstruations and use 10-12 tampons for every cycle, it indicates that the blood loss is significant). It could also be any visible or invisible blood loss through urine, feces, etc

Replay from Dr. Shk. Sotiraq Lako, më 11 January 2018 në 09:39

Hello Doctor! I did some tests because I felt a bit tired and I'm kind of uninterested in anything, I feel like I'm listless. Also, I have a pain or like a tingling in all my bones when I rest. Hemoglobin came out 11.3 and ferritin 5.3. Today I started taking pills, but I see in your comment that it is not suitable for replenishing iron stores. What do you recommend I take?

Sent by Manjola, më 11 January 2018 në 08:14

Hello Manjola, you currently have mild Anemia from Iron Deficiency. Treatment with Tot'hema is not optimal (1 ampoule contains 50 mg elemental iron). You will receive for at least 3 consecutive months: 1-Heferol 350 mg 2 x 1 tablet/day. 2-Vitamin C 100 mg 2 x 1 tablet/day. 3-Vitamin B6 25 mg 2 x 1 tablet/day. Peripheral blood analysis will be repeated after 1 month and at the end of 3 months complete blood treatment and Ferritinemia. Only when ferritinemia is corrected (about 60 ng/ml), anemia should be corrected for about 1-1.5 months of treatment, you have been treated properly. And once the ferritinemia is corrected, it is repeated after 5-6 months, to see stability. If it drops, it indicates that there is a cause and generally, it is blood losses with menstruations (if you have menstruations and use 10-12 tampons for each cycle, it indicates that the blood loss is significant). There might also be any visible or invisible blood loss through urine, feces, etc

Replay from Dr. Shk. Sotiraq Lako, më 11 January 2018 në 09:38

Hello Doctor! I ran some tests because I was feeling a bit tired and I'm like uninterested in anything, I feel kind of dulled. Also, I have like a pain or a tickling in all my bones when I rest. Hemoglobin turned out to be 11.3 and ferritin 5.3. Today I started taking pills, but I see in your comment that it's not suitable for replenishing iron stores. What do you recommend I take?

Sent by Manjola, më 11 January 2018 në 11:07

Hello Manjola, you currently have mild Anemia due to Iron Deficiency. Treatment with Tot'hema is not optimal (1 ampoule contains 50 mg elemental iron). You will take for at least 3 consecutive months: 1-Heferol 350 mg 2 x 1 tablet/day. 2-Vitamin C 100 mg 2 x 1 tablet/day. 3-Vitamin B6 25 mg 2 x 1 tablet/day. Peripheral blood analysis will be repeated after 1 month and at the end of 3 months complete blood treatment and Ferritinemia. Only when ferritinemia is corrected (around 60 ng/ml), anemia should be corrected after about 1-1.5 months of treatment, you are treated properly. And once the ferritinemia is corrected, it is repeated after 5-6 months, to see the stability. If it drops, it indicates that there is a cause and generally are blood losses during menstruation (if you have menstruations and use 10-12 tampons for every cycle, it indicates that the blood loss is significant). It can also be any visible or invisible blood loss through urine, feces, etc

Replay from Dr. Shk. Sotiraq Lako, më 12 January 2018 në 03:41

Hello to you, please why is the use of leech therapy not optimal for moderate anemia with hemoglobin 7.4 for example...?

Sent by Lejla, më 12 January 2018 në 05:59

The Daily Medicinal Dose of iron is 200 mg of elemental iron/day, for at least 3 consecutive months. Each vial of iron only contains 50 mg of elemental iron. You need to take 4 vials/day to reach the medicinal dose, but liquids are not very effective in replenishing iron stores. And if you don't replenish the iron stores, the cause of iron deficiency anemia remains, and the anemia will recur

Replay from Dr. Shk. Sotiraq Lako, më 12 January 2018 në 09:45

Hello doctor, I have a question regarding anemia. My iron storage tests showed ferritin at 7 and sideremia at 210, while hemoglobin was at 10. I started treatment with heferol, but it caused me to vomit, and now I am taking tadyferol 80 mg. I stopped using Vitamin C because it seemed to irritate my stomach. They told me to repeat the tests after three months but to stop taking iron 4 days before. However, I feel that I have concerns like dizziness, increased heart rate, and my legs still feel heavy. What do you suggest?

Sent by Ola, më 12 June 2018 në 10:12

Hello Ola, if you support the treatment with Tardyferon 80 mg, you will take it 3 x 1 tablets/day, for at least 3 consecutive months. After 1 month of treatment, you will have a complete blood count, aiming for the correction of hemoglobin (>12 gr/dl) and there is no need to stop taking iron for 4 days, but you can do it 1,2,3 days before. Once the anemia is corrected, the treatment continues for another 2 months with the aim of correcting ferritin levels (at least > 20 ng/ml and the optimal value for females around 60 ng/ml). Only when Ferritin levels are corrected, you are properly treated. And for ferritin, there is no need to stop the medication for a few days and then do the analysis. Vitamin C helps the absorption of iron, but it is not mandatory. The iron medication can be taken in the middle or after meals

Replay from Dr. Shk. Sotiraq Lako, më 12 June 2018 në 12:17

Thank you very much, Dr. Sotiraqi, for the advice. Should I do a complete blood analysis after one month? And the ferritin after 3 months if I understood correctly?

Sent by Ola, më 12 June 2018 në 12:44

Hello Ola, after 1 month complete blood and after 3 months of treatment ferritinemia

Replay from Dr. Shk. Sotiraq Lako, më 13 June 2018 në 02:43

Hello...I have done the tests and the results are: Hb 10.5 g/dl and serum iron 41. I have started taking ferro 3 2x1 per day...I await your opinion...thank you

Sent by DENISA, më 22 June 2018 në 02:24

Hello Denisa, I need the other data as well; RBC, WBC, Plt. How old are you? Ferro 3 is not a preparation that ensures the replenishment of iron stores.

Please write to me the above data and we will discuss the treatment again

Replay from Dr. Shk. Sotiraq Lako, më 22 June 2018 në 07:01

Hello... I am 27 years old. The results of the tests are RBC=5.29, WBC=7.26, PLT=290, HB=10.5, HCT=36.0, sideremia=41... I emphasize that I am a carrier of Mediterranean anemia. Thank you

Sent by DENISA, më 23 June 2018 në 03:56

Hello Denisa, as you know, Thalassemia Minor is not treatable. Given that the serum iron level is low, although we use Ferritinemia as a basis to assess iron stores, you will take for 1 month:

Ironorm 3 x 1 capsule/day and after 1 month you will do: complete blood count + ferritinemia. Depending on the ferritin values, the continuation or discontinuation of the preparation will be discussed

Replay from Dr. Shk. Sotiraq Lako, më 23 June 2018 në 04:54

Hello, doctor. I want to ask and seek advice from you. I have an 11-year-old niece. She had been feeling tired for a while, complaining about leg pain. They attributed it to her growth spurt since she is quite tall for her age, 162 cm tall, with a shoe size of 37. However, what concerned us was her hair thinning. This led to blood tests. Her hemoglobin was 13...but her iron was 6.7. Everything else came back normal. She also had a slight deficiency in vitamin D. Please tell me what should be done about this situation. The parents are doing everything, but I, as her grandmother, am personally seeking your advice on this matter. Please respond to me. With respect, Dasha Gusho!

Sent by Dasha Gusho, më 28 June 2018 në 12:37

Hello Dasha, with an iron level of 6.7, I believe it is the Ferritin analysis. If it is such < 15/20 ng/ml, it is considered iron deficiency. Your niece needs to take iron medication for 1.5-2 months and after 2 months, she will be rechecked with Ferritinemia. I don't know what your niece's weight is and whether she takes the pills or capsules orally?

Replay from Dr. Shk. Sotiraq Lako, më 29 June 2018 në 04:08

Hello doctor! I have been receiving treatment for anemia correction for 8 months. I have been using Feramax 150, 10 months ago my ferritin was 5.4 and hg 10.2, whereas now hg is 14 and ferritin is 35. Everything else has come back normal except for the sediment rate of 35. How should I proceed? Thank you!

Sent by naevi, më 29 June 2018 në 08:24

Hello Naevi, you currently do not have anemia, a ferritin level of 35 ng/ml is a normal value for females, but given that the optimal ferritin levels for females are around 60 ng/ml, I would recommend that you take medication for another 1 month as above. ESR 35 ng/ml is influenced by several factors. Primarily age (age + 10 divided by 2)

Replay from Dr. Shk. Sotiraq Lako, më 29 June 2018 në 10:58

Thank you for the response, doctor. Age 29, should I get a check-up again after a month?

Sent by Naevi, më 29 June 2018 në 11:24

Can you repeat + CRP, Fibrinogen

Replay from Dr. Shk. Sotiraq Lako, më 30 June 2018 në 05:45

Hello doctor. I have a 15-year-old daughter, after blood tests in April she had these results: wbc 6.83, rbc 4.24, hgb 12.3, hct 37, mcv 87.3, mch 29, mchc 33.2, plt 230, eo 6.10%, mxd 13.9%, ferritin 7.23, and sideremia 84. For 2 months she has taken ironorm, 1 capsule per day. It has been 10 days since she stopped taking the medication and we are in doubt about the daily dose and whether she should continue it or not. Last year, in the same period, she had this problem with ferritin at 8.12, she was treated for 6 weeks with intraferr 2×1.5ml and her ferritin rose to 24.95, and according to the doctor, she no longer needed medication. I am waiting for your response, thank you

Sent by Marjeta Zaimi, më 03 July 2018 në 07:57

Hello Marjeta. It was good that the girl had taken Ironorm with 2 capsules/day, for 2 months straight (3 mg/kg body weight of elemental iron and one Ironorm capsule has 61 mg of elemental iron). Anyway, you will do Ferritinemia. The minimal value should be > 20 ng/ml, the optimal value for females should be around 60 ng/ml. Only if these values (around 60 ng/ml) are reached, can the treatment be stopped, if not reached, the treatment continues. You must keep in mind that iron deficiency is a consequence of consuming or losing more iron than necessary (blood is red in color because of the iron). The girl is at an age where she grows quickly and consumes iron, similarly, women have repeated blood losses with menstruations. Therefore, even after ferritinemia has been corrected (around 60 ng/ml), a check-up every 6 months is done, to see if it remains constant or decreases. If it decreases, it indicates that there is cause/causes (consumption/repeated loss of blood) and this is not related to the season of the year

Replay from Dr. Shk. Sotiraq Lako, më 03 July 2018 në 09:39

Doctor, thank you for your response. I had my daughter's ferritin tested, and it came back as 10.18ng/ml. Please, what is the best supplement, or should we continue with Ironorm by increasing the dosage, and for how long should we continue the treatment? Thank you!

Sent by Marjeta, më 06 July 2018 në 04:29

Hello Mirjeta, Ironorm is a good preparation. It should be taken twice a day for at least 2 consecutive months and after 2 months you will do ferritinemia. If it is not corrected, either the preparation is not absorbed properly, or the cause of iron consumption/loss is significant (ideally, to get the best result, the cause/causes should be eliminated and the consequence/ferritinemia corrected). If it is not corrected, the treatment continues using another preparation, always with the proper dose, for at least 1.5 - 2 months

Replay from Dr. Shk. Sotiraq Lako, më 06 July 2018 në 15:19

Hello Dr.
Very meaningful and helpful articles.
My sideremia came out to 32.6, I am 43 years old. I have taken ferro3+vitc medication for 1 month.
WBC and RBC are normal. Hemoglobin 13.2 and hematocrit 38.4. I am worried. I await a response

Sent by Erli, më 09 July 2018 në 17:43

Hello Erli, we rarely use sideremia in practice. You will do Ferritinemia. If it is lower than 20 ng/ml in females and < 30 ng/ml in males, it is considered low and Ferro3 is not the preparation to replenish iron stores

Replay from Dr. Shk. Sotiraq Lako, më 10 July 2018 në 04:09

Hello Doctor,
I am 37 years old and my hemoglobin came out to 12.8 while ferritin was 6.26. I have not had any abnormal bleeding during my cycle, no other illness, and no change in diet. The only thing I felt was fatigue but I justified it because of the busy day. I am a voluntary blood donor and from the tests that were done, the low value of ferritin was discovered. My family doctor recommended Heferol, but I would like your opinion.
Thank you

Sent by Ida, më 22 July 2018 në 11:26

Hello Ilda, you have Iron Deficiency, which is the most common medical problem for women. Blood loss through menstruation is one reason, but also donating blood equals iron loss. You need to replenish your iron stores so that you can maintain normal hemoglobin levels and be able to donate blood again. Heferol is a good supplement, to be taken twice a day + Vitamin C 100 mg 2 x 1 tablets per day. For at least 2 consecutive months and after 2 months you will have a ferritinemia test done. The minimum value for women is at least 20 ng/ml, but normal values should be 60-75 ng/ml

Replay from Dr. Shk. Sotiraq Lako, më 23 July 2018 në 06:16

Hello Doctor,
I am 37 years old and my hemoglobin came out to be 12.8 while my ferritin is 6.26. I have not had any abnormal bleeding during my cycle, no other kind of disease, and no change in my diet. The only thing I felt was fatigue but I justified it because of my busy day. I am a voluntary blood donor and from the tests that were done, the low value of ferritin was discovered. My family doctor recommended Heferol, but I would like your opinion.
Thank you

Sent by Ida, më 22 July 2018 në 23:40

Hello Ilda, you have Iron Deficiency, which is the most common medical problem for women. Blood loss during menstruation is one reason, but also donating blood equals iron loss. You need to replenish your iron stores to maintain the normal value of hemoglobin and to be able to donate blood again. Heferol is a good preparation, it will be taken twice a day + Vitamin C 100 mg 2 x 1 tablets a day. For at least 2 consecutive months and after 2 months you will do a ferritinemia test. The minimal value for women should be at least 20 ng/ml, but normal values should be 60-75 ng/ml

Replay from Dr. Shk. Sotiraq Lako, më 23 July 2018 në 06:16

Hello Doctor,
I am 37 years old and my hemoglobin came out to 12.8 while my ferritin is 6.26. I haven't had any abnormal bleeding during my cycle, no other illness, and no change in diet. The only thing I felt was tiredness but I justified it due to a busy day. I am a voluntary blood donor and from the tests that were done, the low ferritin value was discovered. My family doctor recommended Heferol, but I would like a second opinion from you.
Thank you

Sent by Ida, më 23 July 2018 në 03:45

Hello Ilda, you have Iron Deficiency, which is the most common medical problem for women. Blood loss through menstruation is one cause, but also blood donation = iron loss. You need to correct your iron stores, so you can maintain normal hemoglobin levels and be able to donate blood again. Heferol is a good preparation, it will be taken twice a day + Vitamin C 100 mg 2 x 1 tablets per day. For at least 2 months in a row and after 2 months you will do ferritinemia. The minimum value for females is at least 20 ng/ml, but normal values should be 60-75 ng/ml

Replay from Dr. Shk. Sotiraq Lako, më 23 July 2018 në 06:16

Hello doctor, I am Emirjona, 32 years old. I just received the results of my blood tests. My ferritin level came out to be 11.57 ng/ml, while my hemoglobin level came out to be 13.4 g/dL. I haven't had any problems. I did the tests because I am considering pregnancy. I would like to hear your opinion. Thank you

Sent by Emiriona, më 24 July 2018 në 09:12

Hello Emiriona, you have an iron deficiency, without the presence of anemia. So currently, you do not have anemia. It's the most common problem among women, due to the fact that they have half the iron reserves compared to men and lose blood = iron every month through menstruation. But also, any significant or minor blood loss through urine, feces, can be another cause of iron deficiency. Pregnancy is a period where there will be no menstruation, but the consumption of iron will increase. You will take for 2 consecutive months: Ironorm 3 x 1 capsule/day and after 2 months you will do Ferritinemia and Hemoglobin Electrophoresis. Ironorm contains iron and Vitamin C, B6, and Folic Acid, which are given to pregnant women for the formation of the baby's neural tube

Replay from Dr. Shk. Sotiraq Lako, më 24 July 2018 në 13:04

Hello doctor, I am 19 years old and have difficulty breathing, especially when I sleep, and sometimes I get dizzy, my eyes get blurry and I hear noises, all this lasts about 1 minute. I did blood tests and they told me I have an iron deficiency, I am anemic. My hemoglobin is 10.4g/dl and Erythrocytes are 5.68x10^6 μl. Is this type of anemia very serious and what do you recommend????

Sent by Kleopatra, më 10 Agust 2018 në 02:27

Hello Cleopatra, you have mild anemia and it is suspected that it may be congenital (a carrier of Thalassemia). You will undergo Ferritinemia and Hemoglobin Electrophoresis. Anemia does not cause problems when you are lying down. The problems occur when moving

Replay from Dr. Shk. Sotiraq Lako, më 10 Agust 2018 në 06:32

Yes doctor, you are very accurate because that's what they told me. Tomorrow I will do the tests again. But if it is congenital and I am a carrier of thalassemia, is it something serious and how is it treated?

Sent by Kleopatrq, më 10 Agust 2018 në 08:04

Hello Cleopatra, there are many carriers of Thalassemia, in Albania and abroad. If you have it, at least one of your parents has it, and your brothers and sisters might have it too (they must also undergo the necessary tests that you will do), it comes from the grandmother or grandfather from the parent it was inherited from, etc. It cannot be treated, just diagnosed. It has no impact on your life. You can read more on the article "Hemoglobinopathies"

Replay from Dr. Shk. Sotiraq Lako, më 11 Agust 2018 në 04:15

Hello Dr., I am 23 years old and have been on medication for iron deficiency anemia for almost 5 years.
These are the latest test results: MCV 76.7, HCT 35.0, HGB 118, FeT 6.4 (there are cases when it drops a lot to 2-3), ferritin was done a few months ago 4.07.
Clinically: fatigue, tiredness, difficulty in breathing after a small activity, low blood pressure, fast pulse, cold extremities, hair loss..
What is your advice? Thank you!

Sent by shpetime, më 11 Agust 2018 në 17:14

Hello Shpetime, not every iron supplement that is taken is considered a medicine. You need 180-200 mg of elemental iron/day for at least 2 consecutive months to replenish the iron stores, always when the cause/causes of iron deficiency have been eliminated (usually in women, significant blood loss with menstruation). You will take for 2 months: Heferol 350 mg 2 x 1 capsule/day + Vitamin C 100 mg 2 x 1 tablet/day + Vitamin B6 25 mg 2 x 1 tablet/day. After 2 months you will do: complete blood count + Ferritinemia

Replay from Dr. Shk. Sotiraq Lako, më 12 Agust 2018 në 02:48

Hello Dr., I am 23 years old and have been taking medication for iron deficiency anemia for almost 5 years. These are the latest results of the tests: MCV 76.7, HCT 35.0, HGB 118, FeT 6.4 (there are cases when it drops significantly to 2-3), ferritin was done a few months ago 4.07. Clinic: weakness, fatigue, difficulty in breathing after a minor activity, low blood pressure, fast pulse, cold extremities, hair loss.. What is your advice? Thank you!

Sent by shpetime, më 12 Agust 2018 në 16:05

Hello Shpetime, not every iron supplement taken is considered a medicine. It takes 180-200 mg of elemental iron/day for at least 2 consecutive months to replenish iron stores, always when the cause/causes of the iron deficiency have been eliminated (usually in women, significant blood loss with menstruation). You will take for 2 months: Heferol 350 mg 2 x 1 capsule/day + Vitamin C 100 mg 2 x 1 tablet/day + Vitamin B6 25 mg 2 x 1 tablet/day. After 2 months you will do: complete blood count + Ferritinemia

Replay from Dr. Shk. Sotiraq Lako, më 13 Agust 2018 në 06:01

Hello doctor, I did a ferritin test and electrophoresis, and the ferritin came out to 61.99 μg/L while the electrophoresis shows Hb A 95.79%L Hb A2 4.21%. What do I have???

Sent by kleopatra, më 16 Agust 2018 në 03:51

Hello Cleopatra, you need to present the values of RBC and Hb as well. Anyway, you are with beta-Thalassemia Minor, a carrier of Thalassemia. You might want to read the article on Hemoglobinopathies

Replay from Dr. Shk. Sotiraq Lako, më 16 Agust 2018 në 06:37

The value of RBC 5.68x10^6 /μL and Hb 10.4g/dL

What do you advise me to take, do I have a problem??

Sent by kleopatra , më 16 Agust 2018 në 08:22

Hello Cleopatra, you probably have Thalassemia Minor (carrier of Thalassemia). You will do Ferritinemia and Hemoglobin Electrophoresis

Replay from Dr. Shk. Sotiraq Lako, më 17 Agust 2018 në 14:00

RBC value 5.68x10^6 /μL and Hb 10.4g/dL

What do you advise me to take, do I have a problem??

Sent by kleopatra , më 16 Agust 2018 në 08:22

Hello Cleopatra, you are probably with Thalassemia Minor (carrier of Thalassemia). You will do Ferritinemia and Hemoglobin Electrophoresis

Replay from Dr. Shk. Sotiraq Lako, më 17 Agust 2018 në 14:00

Hello doctor, I did my tests last year and my hb, hct, mch, mcv levels were low. I took heferol for 1 month and then tardyferon. Now, I have repeated the tests and the results are as follows: hct 33.7, hb 10.5, mcv 75.7, mch 23.6, mchc 31.2, rdw 41.5, plt 407, hbA 97.7, hbA2 2.1, ferritin 5.01, serum iron 13.76, transferrin saturation 2.9%, folic acid 2.55. Symptoms include fatigue and leg pain. Is this anemia? What should I take? Thank you

Sent by Kesi Mehmeti, më 22 Agust 2018 në 15:30

Hello Kesi, you currently have Iron Deficiency Anemia, Grade I (mild). Why did you not continue the treatment with Heferol? Did you have any complaints? If you had no complaints, you will use: 1-Heferol 350 mg 2 x 1 capsule/day. 2-Vitamin C 100 mg 2 x 1 tablet/day. 3-Vitamin B6 25 mg 2 x 1 tablet/day. for 3 consecutive months.

You will be re-checked with a complete blood count after 1 month and after 3 months of treatment, complete blood count + ferritinemia. The aim of the treatment is the correction of anemia (Hb > 12 gr/dl which is achieved in 1-1.5 months and the final goal is the correction of ferritin levels 60-75 ng/ml. Do not forget that the cause of anemia is the lack of iron, but the cause of iron deficiency is generally in females, blood loss with menstruation, therefore the possibility of recurrence of iron deficiency and consequently anemia due to iron deficiency, exists.

If you do not take Heferol, you will use, as above (for 3 consecutive months) with the same controls Ironorm 3 x 1 capsule/day, without the need for vitamins and folic acid

Replay from Dr. Shk. Sotiraq Lako, më 23 Agust 2018 në 04:15

Hello doctor! I am Evi, 34 years old. I have been suffering from anemia for years without any results from the treatments I have taken until it was discovered that my iron stores were empty. I have very heavy menstruations. My ferritin level dropped to 1. I have been treated with Ferrosanol duo denal, one pill per day along with vitamin C. After 3 months of treatment, my test results came out as follows: Hb 13. WBC 6700. RBC 4,800,000. Ht 40.2. Platelets 158,000. Neutrophils 62.1. Lymphocytes 30.6. Monocytes 7.3. MCV 83.8. MCH 27.1. MCHC 32.3. Ferritin 6.3. I feel tired and nervous all the time. I also have a problem with spondyloarthrosis. What do you advise? Thank you!

Sent by Evi, më 29 Agust 2018 në 11:02

Hello Evi, you have not been suffering from anemia for a long time, but from increased blood loss during menstruation. Blood is red because of iron; blood loss = iron loss. If you reduce the blood flow, you eliminate the cause. The consequence of the cause is the lack of iron, the consequence of the lack of iron is anemia. Anemia is just the tip of the iceberg of your problem. As you can see, even though you are not taking the proper dose of medication (200 mg of elemental iron/day is needed and Ferro sanol contains only 100 mg of elemental iron), the anemia has been corrected (ferritinemia = 13 ng/ml, which is more than 12 g/dl, the threshold for anemia in women). You should also consider that in 3 months, you will have at least 3 menstrual cycles, thus 3 iron losses. For this reason, the result will not be as expected. If the cause is eliminated (menstrual flow is reduced) and you continue the treatment with Ferro Sanol 2 x 1 capsule/day (the preparation itself contains Vitamin C) for at least 2 more months and Ferritinemia reaches 60-75 ng/ml, the problem you have will be healed. If you do not eliminate the cause (you accept this menstrual cycle that you have) and even if you are treated as above and reach Ferritinemia 60-75 ng/ml, you should be aware that one day the iron deficiency and/or anemia will recur (they can be treated and corrected again, but will recur if the cause persists). It has nothing to do with spondylarthrosis where the problem lies

Replay from Dr. Shk. Sotiraq Lako, më 29 Agust 2018 në 15:46

Hello doctor! I am Evi, 34 years old. I have been suffering from anemia for years without any results from the treatments I have taken until it was discovered that my iron stores were empty. I have very heavy periods. My ferritin level has dropped to 1. I have been treated with Ferrosanol duo denal one pill a day accompanied by vitamin C. After 3 months of treatment, the results of my tests came out as follows: Hb 13. WBC 6700. RBC 4,800,000. Ht 40.2. Platelets 158,000. Neutrophils 62.1. Lymphocytes 30.6. Monocytes 7.3. MCV 83.8. MCH 27.1. MCHC 32.3. Ferritin 6.3. I feel tired and nervous all the time. I also have a problem with spondyloarthrosis. What do you advise? Thank you!

Sent by Evi, më 29 Agust 2018 në 11:03

Hello Evi, you have not been suffering from anemia for a long time, but from increased blood loss during menstruation. Blood is red because of the iron, blood loss = iron loss. If you reduce the blood flow, you eliminate the cause. The consequence of the cause is the lack of iron, the consequence of the lack of iron is anemia. Anemia is just the tip of the iceberg of your problem. As you can see, even though you are not taking the proper dose of medication (200 mg of elemental iron/day is required and Ferro sanol only contains 100 mg of elemental iron), the anemia has been corrected (ferritinemia = 13 ng/ml, meaning more than 12 gr/dl which is the threshold for anemia in women). You should also keep in mind that in 3 months you will have at least 3 menstrual cycles, hence 3 iron losses. For this reason, the result will not be as expected. If the cause is eliminated (the menstrual flow is reduced) and you continue the treatment with Ferro Sanol 2 x 1 capsules/day (the preparation itself contains Vitamin C) for at least 2 more months and the ferritinemia reaches 60-75 ng/ml, the problem you have will be cured. If you do not eliminate the cause (you accept this menstrual cycle you have) and if treated as above and you achieve ferritinemia of 60-75 ng/ml, you should be aware that one day the iron deficiency and/or anemia will recur (they will be treated and corrected again, but will repeat if the cause persists). It has nothing to do with spondyloarthrosis where the problem lies

Replay from Dr. Shk. Sotiraq Lako, më 29 Agust 2018 në 15:46

Hello doctor! It's Evi again, thank you very much for the answer. I wanted to ask if a second pregnancy affects the improvement of the menstrual cycle (I have heavy menstrual flow). Thank you!

Sent by Evi, më 30 Agust 2018 në 05:13

Hello Evi, it is possible, but you need to discuss this with the obstetrician-gynecologist. Regarding iron reserves, it is beneficial to have as little blood loss as possible. Pregnancy is a solution for the temporary elimination (about 1 year or a little more) of menstruation, and this itself affects the iron reserves

Replay from Dr. Shk. Sotiraq Lako, më 30 Agust 2018 në 07:21

Hello doctor. I am a 27-year-old girl. When I did my blood tests, I had hemoglobin at 7, sideremia at 25, and ferritin at 3. I used Ferrum Hausmann 50mg once a day. After 2 months, my ferritin had increased to 13, and hemoglobin to 11. I continued the treatment for another 3 months and did the tests again. Hemoglobin was 12 and ferritin was 10. How is this possible?

Sent by Vilma, më 06 September 2018 në 15:05

Hello Vilma, you have read that the daily medicinal dose is 200 mg of elemental iron per day and the treatment continues for at least 3 months in a row, to correct anemia (1-1.5 months) and then ferritinemia + 1.5 more months. You have taken only 50 mg of iron/day, very little. The anemia has barely been corrected, you cannot claim the replenishment of stores. You will take for 2 months: Heferol 350 mg 2 x 1 capsule/day + Vitamin C 100 mg 2 x 1 tablet/day. After 2 months you will do the ferritinemia

Replay from Dr. Shk. Sotiraq Lako, më 07 September 2018 në 06:47

Hi doctor, I am a 25-year-old girl. I have been experiencing hair loss in patches for 5 months, alopecia. I have done tests, ferritin 18 while sideremia 48. Are these values the cause of the hair loss in patches? Thank you

Sent by Jona, më 09 October 2018 në 05:50

Hello Jona, as a rule, hair does not fall out in patches due to iron deficiency. They become thin, brittle, and fall out generally. However, you need to correct the iron deficiency, and correcting it will confirm whether or not the hair loss is related to the deficiency

Replay from Dr. Shk. Sotiraq Lako, më 10 October 2018 në 06:05

Hello doctor, my niece is 10 years old, she has been experiencing loss of appetite and fatigue for a few days, today we did the tests and her hemoglobin came out to 11.7 and ferritin 21.9, she has been taking Tardyferon 80 2*1, vitamin C, and vitamin B6 2*1, are these enough?

Sent by Alba , më 12 October 2018 në 10:42

Hello, the values that it has for the age are good. To have even more iron reserves, if there has been no concern from the treatment, it can continue for 1 more month

Replay from Dr. Shk. Sotiraq Lako, më 13 October 2018 në 04:47

Hello doctor! I am 27 years old and once every two years I get treated for anemia. I did the tests today and my Hemoglobin came out 9.9 and Sideremia 31.66 from 50-170 which were the normal values. What do you suggest, please? Thank you!

Sent by Manjola, më 12 October 2018 në 11:20

Hello Manjola, You have mild anemia. However, you need to write down the number of erythrocytes (RBC) and whether you have done Ferritinemia. Low Ferritinemia is the best indicator of iron deficiency (< 20 ng/ml). If treated properly, anemia and iron deficiency are fully corrected. If they recur, it indicates that there are cause(s), where the main cause is the loss of iron through blood (in females, menstruation plays a key role. If you do not react to the medications, you will take for at least 3 months in a row: Ironorm 3 x 1 capsule/day. After 1 month complete blood test and after 3 months of treatment complete blood test + ferritinemia. The goal of the treatment is: after 1 - 1.5 months the correction of anemia (Hb > 12 gr/dl) should be achieved and after 3 months also the correction of ferritinemia (60-70 ng/ml). Only when these values are reached you are properly treated. But the lack of iron and/or anemia from iron deficiency can recur, when there is cause(s) = loss of iron

Replay from Dr. Shk. Sotiraq Lako, më 13 October 2018 në 04:52

Hello doctor! I'm 27 years old and every two years I get treated for anemia. I did the tests today and my Hemoglobin came out 9.9 and Ferritin 31.66 from 50-170 which were the normal values. What do you suggest, please? Thank you!

Sent by Manjola, më 12 October 2018 në 11:59

Hello Manjola, You have mild anemia. But you need to write down the number of erythrocytes (RBC) and if you have done Ferritinemia. Low Ferritinemia is the best indicator of iron deficiency (< 20 ng/ml). If treated properly, anemia and iron deficiency can be completely corrected. If it recurs, it indicates that there is a cause/causes, where the main cause is iron loss through blood (in women, menstruation plays a key role. If you do not respond to the medications you will take for at least 3 months in a row: Ironorm 3 x 1 capsules/day. After 1 month complete blood count and after 3 months of treatment complete blood count + ferritinemia. The aim of the treatment is: after 1 - 1.5 months, the correction of anemia (Hb > 12 gr/dl) should be achieved and after 3 months the correction of ferritinemia (60-7- ng/ml). Only when these values are reached have you been treated properly. But iron deficiency and/or anemia from iron deficiency can recur, when there is a cause/causes = iron loss

Replay from Dr. Shk. Sotiraq Lako, më 13 October 2018 në 04:52

Hello doctor! I am 27 years old and once every two years I get treated for anemia. I did the tests today and my Hemoglobin came out 9.9 and Serum Iron 31.66 from 50-170 which were the normal values. What do you suggest, please? Thank you!

Sent by Manjola, më 12 October 2018 në 13:28

Hello Manjola, You have mild anemia. But you need to write down the number of erythrocytes (RBC) and if you have done Ferritinemia. Low Ferritinemia is the best indicator of iron deficiency (< 20 ng/ml). If treated properly, anemia and iron deficiency are completely corrected. If they recur, it indicates that there is a cause/causes, where the main cause is the loss of iron through blood (in women, menstruation plays a major role. If you do not respond to the medications, you will take for at least 3 months in a row: Ironorm 3 x 1 capsules/day. After 1 month complete blood test and after 3 months of treatment complete blood test + ferritinemia. The goal of the treatment is: after 1 - 1.5 months, the correction of anemia (Hb > 12 gr/dl) should be achieved and after 3 months the correction of ferritinemia (60-70 ng/ml). Only when these values are reached, you are properly treated. But the deficiency of iron and/or anemia from iron deficiency can recur, when there is a cause/causes = loss of iron

Replay from Dr. Shk. Sotiraq Lako, më 13 October 2018 në 04:53

Hello doctor! I am 27 years old and once every two years I get treated for anemia. I did the tests today and my Hemoglobin came out 9.9 and Serum Iron 31.66 from the normal values of 50-170. What do you suggest, please? Thank you!

Sent by Manjola, më 12 October 2018 në 23:55

Hello Manjola, You have mild anemia. But you need to write down the number of erythrocytes (RBC) and whether you have done Ferritinemia. Low Ferritinemia is the best indicator of iron deficiency (< 20 ng/ml). If treated properly, anemia and iron deficiency are completely corrected. If they recur, it indicates that there is a cause/causes, where the main cause is the loss of iron through blood (in women, menstruation plays a key role. If you do not respond to the medications, you will take for at least 3 months in a row: Ironorm 3 x 1 capsule/day. After 1 month complete blood count and after 3 months of treatment complete blood count + ferritinemia. The goal of the treatment is: after 1 - 1.5 months the correction of anemia should be achieved (Hb > 12 gr/dl) and after 3 months the correction of ferritinemia (60-70 ng/ml). Only when these values are reached, you are treated properly. But the lack of iron and/or anemia due to iron deficiency can recur, when there is a cause/causes = loss of iron

Replay from Dr. Shk. Sotiraq Lako, më 13 October 2018 në 04:53

I have an RBC of 3.63 from the normal value of 4.0 - 6.20. Apparently, I have ferritin deficiency

Sent by Manjola, më 13 October 2018 në 11:52

Hello Manjola, you have mild anemia, probably due to iron deficiency. Start treatment with Ironorm 3 x 1 capsules/day. After 1 month of treatment, you will do a complete blood count + ferritinemia. Depending on the values, the continuation of the treatment will be discussed

Replay from Dr. Shk. Sotiraq Lako, më 14 October 2018 në 08:41

I have an RBC of 3.63 out of 4.0 - 6.20 which is the normal value. I apparently have low ferritin

Sent by Manjola, më 13 October 2018 në 12:10

Hello Manjola, you have mild anemia, probably due to iron deficiency. Start treatment with Ironorm 3 x 1 capsules/day. After 1 month of treatment, you will have a complete blood count + ferritinemia. Depending on the values, the continuation of the treatment will be discussed

Replay from Dr. Shk. Sotiraq Lako, më 14 October 2018 në 08:42

My RBC is 3.63 from the normal value of 4.0 - 6.20. It seems I have ferritin deficiency

Sent by Manjola, më 13 October 2018 në 15:37

Hello Manjola, you have mild anemia, probably due to iron deficiency. Start treatment with Ironorm 3 x 1 capsule/day. After 1 month of treatment, you will undergo a complete blood + ferritinemia test. Depending on the values, the continuation of treatment will be discussed

Replay from Dr. Shk. Sotiraq Lako, më 14 October 2018 në 08:42

I have an RBC of 3.63 out of the normal range of 4.0 - 6.20. It seems I have low ferritin

Sent by Manjola, më 14 October 2018 në 01:10

Hello Manjola, you have mild anemia, probably due to lack of iron. Start treatment with Ironorm 3 x 1 capsule/day. After 1 month of treatment, you will do a complete blood count + ferritinemia. Depending on the values, the continuation of the treatment will be discussed

Replay from Dr. Shk. Sotiraq Lako, më 14 October 2018 në 08:42

Hello doctor! I have a 21-month-old son who has been on iron treatment since August. In August, his blood test values were
RBC 5.14. HCT 23.1. HGB 7.6 MCV 44.9 MCH 14.8 MCHC 32.9 RDW-SD 38 RDW-CV 24.7 FERRITIN 4.81 and Serum Iron 16.15. Meanwhile, with iron treatment during these 2 months, he now has these values. RBC 5.78 HCT 28.7 HGB 9.8 MCV 49.7 MCH 17 MCHC 34 RDW-SD 42 RDW-CV 25. FERRITIN 25.3 and SERUM IRON 97.4 please doctor can you tell me if my child has anemia due to iron deficiency or maybe it is inherited. Neither my spouse nor I suffer from anemia at least as reflected in normal blood tests. The boy is active, eats well, weighs 14 kg, and is 88 cm tall! All the best to you!
Thank you in advance

Sent by Erjona, më 20 October 2018 në 06:02

Hello Erjona, the data indicate that there might be inherited mild anemia. But for children, it is good that Hemoglobin Electrophoresis is done after the age of 4 years. But you and your spouse will do: complete blood count, ferritinemia, hemoglobin electrophoresis. And we will discuss it again. Hemoglobin Electrophoresis reveals beta forms of hemoglobinopathies, but not alpha forms

Replay from Dr. Shk. Sotiraq Lako, më 20 October 2018 në 08:55

Hello doctor! I had a blood test two weeks ago and my ferritin level was 5.2. I started taking liquid iron and vitamin C and today when I repeated the tests to see if it started to increase, it was 3.7. Is it possible that by taking iron, the ferritin level goes from 5.2 to 3.7? What should I do? Should I change my medication? Thank you in advance

Sent by ana, më 24 October 2018 në 06:35

Hello Ana, if the hemoglobin has been > 12 gr/dl and ferritin 5.2 ng/ml, you are considered to have an iron deficiency. The therapeutic dose is 180-200 mg of elemental iron/day, for at least 2 consecutive months. The supplement you have taken is definitely not providing this amount. After 2 months of treatment with the proper dose, the ferritin levels should be re-evaluated. The minimum target is 20 ng/ml and the optimal value for females is 60-70 ng/ml

Replay from Dr. Shk. Sotiraq Lako, më 25 October 2018 në 05:31

Hello, doctor
I have been experiencing hair thinning and lately, I have had scalp pain. After an examination by the gynecologist, it turned out that I have a fibroid. Doctor, could the consequence of the fibroid be the reason my hair is falling out? My blood analysis turned out well, but I also consulted with a dermatologist and after the examination, he advised me to use a Pantelon solution every day to strengthen the roots, starting with vitamin C, Magnesium, and iron, but there has been no result. Please, doctor, give me an answer

Sent by lea, më 26 October 2018 në 02:48

Hello Lea, hair loss from fibroids does not occur. If the fibroid causes excessive blood loss and this is accompanied by iron deficiency, the lack of iron can cause hair loss. Iron deficiency is assessed with Ferritinemia < 20 ng/ml. If this is the case, the appropriate treatment with iron supplements should be taken

Replay from Dr. Shk. Sotiraq Lako, më 26 October 2018 në 07:51

Hello doctor! I am a 24-year-old girl, I have a problem with hair loss. I've done all the necessary tests, everything is fine, but the doctor gave me an iron deficiency treatment. After I finished the iron treatment, it increased significantly, the doctor told me that my iron depot has not increased. The results are these: serum iron 64, ferritin 26.3. I want to know, does my hair fall out because of this?

Sent by Anita shehaj, më 08 November 2018 në 10:25

Hello Anita, the minimum normal value of ferritin in females is 20 ng/ml and the normal value that should be maintained is 60-70 ng/ml. You will continue the treatment until it is completely corrected. If the Ferritin reaches 60-70 ng/ml and the hair stops falling out, it indicates that it was related to the iron deficiency; if the hair continues to fall out even with this value, there are other factors involved

Replay from Dr. Shk. Sotiraq Lako, më 09 November 2018 në 08:08

Hello doctor!! I am a 24-year-old girl, I have a problem with hair loss. I have done all the necessary tests, everything is fine but the doctor prescribed me an iron supplement treatment. After I finished the iron treatment, it significantly increased, the doctor told me that my iron stores have not increased. The results are these: sideremia 64, ferritin 26.3. I want to know, does my hair fall out because of this?

Sent by Anita shehaj, më 09 November 2018 në 02:32

Hello Anita, the minimum normal value of ferritin in women is 20 ng/ml, and the normal value that should be maintained is 60-70 ng/ml. You will continue the treatment until its complete correction. If Ferritin reaches 60-70 ng/ml and the hair stops falling out, it indicates that it was related to an iron deficiency; if the hair continues to fall out even with this value, there are other factors involved

Replay from Dr. Shk. Sotiraq Lako, më 09 November 2018 në 08:08

Hello doctor!! I am a 24-year-old girl, I have a problem with hair loss. I have done all the necessary analyses, everything is fine, but the doctor prescribed me an iron deficiency treatment. After I finished the iron treatment, my iron levels significantly increased, but the doctor told me that my iron stores had not increased. The results are these: sideremia 64, ferritin 26.3. I want to know, is my hair falling out because of this?

Sent by Anita shehaj, më 09 November 2018 në 02:32

Hello Anita, the minimum normal value of ferritin in females is 20 ng/ml, and the normal value they should have is 60-70 ng/ml. You will continue the treatment until it is completely corrected. If the Ferritin reaches 60-70 ng/ml and the hair no longer falls out, it indicates that it was related to the lack of iron, if they continue to fall out even with this value, there are other factors involved

Replay from Dr. Shk. Sotiraq Lako, më 09 November 2018 në 08:08

Hello doctor !! I am a 24-year-old girl, I have a problem with hair loss. I have done all the necessary tests, everything is fine, but the doctor gave me an iron deficiency treatment. After I finished the iron treatment, it increased significantly, the doctor told me that your iron store has not increased. The results are these: sideremia 64, ferritin 26.3 .. I want to know, is this the reason my hair is falling out?

Sent by Anita shehaj, më 16 November 2018 në 08:01

Hello Anita, the value of Ferritin is above the lower limit of normal (20 ng/ml), but the optimal value for women should be 60-75 ng/ml. If you do not have allergies to the medications in use: 1-Heferol 350 mg 2 x 1 capsule/day. 2-Vitamin C 100 mg 2 x 1 tablet/day, for 1.5 to 2 months and then we repeat the analysis. One of the reasons why hair falls out is also due to the lack of iron

Replay from Dr. Shk. Sotiraq Lako, më 16 November 2018 në 09:48

Hello doctor. I am 23 years old, I did the tests and the results are these: erythrocytes 4.95, HGB 12, HEMATOCRIT 36.4, MCV 73.5, MCH 24.2, MCHC 33.0. While my ferritin is very low at 4.3. Today I will start treatment with floradix fe 2×1 tab per day. Is it the right treatment?

Sent by Xhesilda, më 27 November 2018 në 08:31

Hello Xhesi, you have mild Anemia due to iron deficiency. If you are | 15 years old: Heferol 350 mg 2 x 1 capsule/day + Vitamin C 100 mg 2 x 1 tablet/day, Vitamin B6 25 mg 2 x 1 tablet/day. For 2 consecutive months. After 2 months of treatment: complete blood count + ferritinemia + Hemoglobin Electrophoresis

Replay from Dr. Shk. Sotiraq Lako, më 29 November 2018 në 02:36

Hello, I am Bruna and I have been having a problem with ferritin for years. I take tardeferon B9 twice a day on the 10th of the month, but actually, after two months all the iron comes out through my stools. What should I do because I feel very tired and very pale in the face?

Sent by Brunilda , më 10 December 2018 në 13:46

Hello Brunilda, iron deficiency = low Ferritinemia is the most common problem in medical practice. I don't know what your value is, but the best preparation to replenish iron stores in Albania is Heferol 350 mg 2 x 1 capsule/day + Vitamin C 100 mg 2 x 1 tablet/day. For at least 2 consecutive months. The normal minimum value is 20 ng/ml for females and the optimal value is 60-70 ng/ml. Treatment is discontinued once this value is reached. The cause of iron deficiency is almost always iron loss and in females, the main role is played by blood loss during menstruation. But any significant or invisible blood loss through urine or feces is evaluated and must be treated

Replay from Dr. Shk. Sotiraq Lako, më 11 December 2018 në 03:54

Hello, I am 22 years old. I have significant hair loss in the central part of my head. I use shampoo and capsules for hair loss, but they haven't done anything for me. The hair loss has become problematic. My hemoglobin is 14. Ferritin is 14.8. I have used Ironorm capsules for a month but haven't redone the tests. My blood tests are normal. What do you suggest? Thank you!

Sent by Eduina, më 28 December 2018 në 03:43

Hello Eduina, iron deficiency assessed with ferritinemia < 20 ng/ml is one of the causes of hair loss. Hair loss is not related to anemia. Ironorm is a good preparation to refill the stores. If you are an adult (> 15 years old), Ironorm should be continued for another month with 3 x 1 capsules/day and after 1 month you will do a complete blood count + ferritinemia. If ferritinemia reaches 60-70 ng/ml and the hair continues to fall, there are certainly other causes. If it is from iron deficiency, by correcting it, the hair loss will also be corrected

Replay from Dr. Shk. Sotiraq Lako, më 28 December 2018 në 08:41

Hello, doctor. I am 33 years old, I did blood tests because I had dizziness and rapid fatigue without any high physical activity. HB 11, ferritin 3.7, serum iron 52. I drank for 3 days Heferol 350 (2×1), vitamin C 100 (2×1), vitamin B6 (2×1). I started having stomach pain and stopped the medication. Please, what should I do?

Sent by Ana, më 23 February 2019 në 08:56

Hello Ana, if Heferol bothers you, another preparation will be considered. You can use Ironorm 3 x 1 capsules/day, during or after meals. If this also causes discomfort, a stomach examination with gastroscopy will be performed, and the taking of iron intravenously will be evaluated

Replay from Dr. Shk. Sotiraq Lako, më 25 February 2019 në 08:28

Hello Doctor, I am 55 years old. I got my blood tests done and my iron stores came out at 1.6. I am taking ferrous sulfate duodenal twice a day, vitamin C three times a day, and vitamin B6 three times a day. During the treatment, I have been experiencing some itchiness that stops bothering me after ten minutes. The marks look like I've been bitten by a mosquito. Could you please tell me what might be causing this itchiness, and should I stop the medication? Thank you

Sent by Margarita Janko, më 18 March 2019 në 13:56

Hello Margarita, Ferrosanol duodenal is a good preparation. It contains vitamin C itself. It is possible that this preparation or accompanying vitamins may cause allergies. Use only Ferrosanol, without the vitamins, if your concern does not occur, it is related to the vitamins and you continue with iron, without the vitamins as they are not necessary. If it happens again, it is related to Ferrosanol and you should switch to another preparation

Replay from Dr. Shk. Sotiraq Lako, më 20 March 2019 në 10:12

Hello Doctor, I am 55 years old, I did blood tests and my iron stores came out at 1.6. I am taking ferrous sulfate duodenally twice a day, vitamin C three times a day, and vitamin B6 three times a day. During the treatment, I have a concern about some itching that bothers me for about ten minutes and then doesn't bother me anymore. The marks look like I've been bitten by a mosquito. Could you please tell me what might be causing this itching, and should I perhaps stop the medication? Thank you

Sent by Margarita Janko, më 18 March 2019 në 13:57

Hello Margarita, Ferrosanol duodenal is a good product. It contains vitamin C itself. It's possible that this product or the accompanying vitamins could cause allergies. Use only Ferrosanol, without the vitamins, if your concern does not persist, it means it is related to the vitamins and you can continue the iron without the vitamins as they are not necessary. If it happens again, it is related to Ferrosanol and you should switch to a different product

Replay from Dr. Shk. Sotiraq Lako, më 20 March 2019 në 10:12

Thank you, Doctor, I haven't used vitamin C for two days and I have no concerns

Sent by Margarita janko, më 21 March 2019 në 02:13

You can continue only with Ferro Sanol Duodenal, which itself contains vitamin C

Replay from Dr. Shk. Sotiraq Lako, më 22 March 2019 në 12:13

Hello Doctor,
My daughter is 18 months old and the values of the analyses are: Ferritin 8.08 ng/mL, Serum Iron: 274 µg/dl, WBC 11.2, LYM 6.2, MON 0.5, GRA 4.5, LYM% 55.0, MON% 4.8, GRA% 40.2, RBC 4.87, HGB 11.8, HCT 37.9, MCV 77.8, MCH 24.2, MCHC 31.1, RDW 14.9, PLT 512, MPV 8.2, PCT 0.420, PDW 13.2
Please can you tell me what these values indicate and if my daughter needs any medication. Thank you

Sent by Eni, më 26 March 2019 në 06:45

Hello Eni, the hemoglobin value is normal for the child. Ferritin 8 ng/ml, is a good value for the age. He/She can take iron supplements, due to the fact that children grow quickly and growth needs iron. You will discuss with the pediatrician about the adjunctive treatment

Replay from Dr. Shk. Sotiraq Lako, më 29 March 2019 në 08:30

Hello Dr. Lako!
I am 42 years old. I just did my tests and I am worried about the low level of Ferritin (4.03 ng/ml) and Serum Iron (14 ug/dL). Hemoglobin is 9 g/dL.
Besides the menstrual cycle, I have also had problems for more than 2 years with chronic diarrhea that we cannot find the cause to treat (despite the tests, colonoscopy, and various treatments).
What could you suggest for me to meet my iron needs, please?
Thank you in advance for your time and thoughts!

Sent by Vali, më 27 March 2019 në 08:12

Hello Vali, you have iron deficiency anemia, between mild and moderate. The cause is the lack of iron (for the anemia) and the cause of the iron deficiency is blood loss. In females, more so with menstruation. Diarrhea can play a role in the poor absorption of iron. Gluten has been evaluated as a cause of diarrhea? We prefer oral treatment for anemia and iron deficiency, but generally, they cause diarrhea. The correction with intravenous iron can be discussed, but it also has its own problems

Replay from Dr. Shk. Sotiraq Lako, më 29 March 2019 në 08:38

Hello doctor, I am worried because my husband has Triglycerides 454 and ferritin 4789 ng/ml. Please, how can we act in this case, what diet should he follow to lower them. Thank you very much! I am waiting for a reply. Good luck with your work!

Sent by Adela, më 05 April 2019 në 03:04

Hello Adela, One of the reasons for the increase in ferritin is also lipid disorder. You will also do serum iron and TIBC. If the ratio between them, times 100, is less than 50%, it is considered a reactive increase in ferritin. It does not affect the organism and diet is not of importance. The lipid disorder must be treated, the use of alcohol must be eliminated, and the hyperferritinemia must be corrected

Replay from Dr. Shk. Sotiraq Lako, më 05 April 2019 në 03:58

Hello doctor, I am worried because my husband has Triglycerides of 454 and ferritin of 4789 ng/ml. Please, how should we act in this case, what diet should he follow to lower them. Thank you very much! I am waiting for a reply. Good work!

Sent by Adela, më 05 April 2019 në 03:04

Hello Adela, One of the causes of increased ferritin levels is also fat dysregulation. You will also undergo sideremia and TIBC tests. If the ratio between them, multiplied by 100, is less than 50%, it is considered a reactive increase in ferritin levels. It has no impact on the body and has no nutritional value. The fat dysregulation must be treated, alcohol use must be eliminated, and hyperferritinemia must also be corrected

Replay from Dr. Shk. Sotiraq Lako, më 05 April 2019 në 03:56

Hello doctor, I am worried because my husband's Triglycerides are 454 and ferritin 4789 ng/ml. Please, how can we act in this case, what diet should he follow to lower them. Thank you very much! I am waiting for an answer. Good luck!

Sent by Adela, më 05 April 2019 në 03:07

Hello Adela, One of the causes of increased ferritin is also lipid disorder. You will also do serum iron and TIBC tests. If the ratio between them, multiplied by 100, is less than 50%, it indicates a reactive increase in ferritin. It has no impacts on the body and is not related to diet. The lipid disorder should be treated, alcohol use should be eliminated, and hyperferritinemia should be corrected

Replay from Dr. Shk. Sotiraq Lako, më 05 April 2019 në 03:56

Hello Doctor, I am writing to you again because after using Ferro Sanol, I have experienced some issues, namely some itching on my legs. If I change the preparation, could you recommend me another one? Thank you

Sent by Margarita , më 05 April 2019 në 07:32

Hello Margarita, can you tell me which preparations you have used so far?

Replay from Dr. Shk. Sotiraq Lako, më 05 April 2019 në 13:14

Hello doctor, I am worried because my husband's Triglycerides are 454 and ferritin is 4789 ng/ml. Please, how can we act in this case, what diet should he follow to lower them. Thank you very much! I am waiting for an answer. Good work!

Sent by Adela, më 05 April 2019 në 12:05

Hello Adela, One of the causes of increased ferritin levels is also the dysregulation of fats. You will also undergo sideremia and TIBC tests. If the ratio between them, multiplied by 100, is less than 50%, it is considered a reactive increase in ferritin levels. It has no effects on the body and has no nutritional value. The dysregulation of fats needs to be treated, the use of alcohol eliminated, and hyperferritinemia corrected

Replay from Dr. Shk. Sotiraq Lako, më 05 April 2019 në 13:13

Hello Doctor, I have used Ferro Sanol without the accompanying vitamins as I was told I have Vit B6 and Vit C.
Thank you

Sent by Margarita , më 09 April 2019 në 07:24

Hello Margarita, if you truly have anemia due to iron deficiency (Hb < 12 gr/dl and ferritin < 20 ng/ml), you need to take iron preparations. You can use Heferol 350 mg 2 x 1 capsule/day, or Tardyferon 80 mg 3 x 1 tablet/day

Replay from Dr. Shk. Sotiraq Lako, më 09 April 2019 në 11:45

Hello Doctor, I have used Ferro Sanol without the accompanying vitamins as I had vitamin B6 and Vit C.
Thank you

Sent by Margarita , më 09 April 2019 në 07:24

Hello Margarita, if you truly have anemia from iron deficiency (Hb < 12 gr/dl and ferritinemia < 20 ng/ml), you need to take iron supplements. You can use Heferol 350 mg 2 x 1 capsule/day, or Tardyferon 80 mg 3 x 1 tablet/day

Replay from Dr. Shk. Sotiraq Lako, më 09 April 2019 në 11:45

Hello Doctor, I have used Ferro Sanol without the accompanying vitamins as I had been told, B6 and Vit C.
Thank you

Sent by Margarita , më 09 April 2019 në 07:24

Hello Margarita, if you truly have anemia due to iron deficiency (Hb < 12 gr/dl and ferritinemia < 20 ng/ml), you need to take iron supplements. You can use Heferol 350 mg 2 x 1 capsule/day, or Tardyferon 80 mg 3 x 1 tablet/day

Replay from Dr. Shk. Sotiraq Lako, më 09 April 2019 në 11:45

Hello doctor, I am 26 weeks pregnant, my ferritin is 10, Hgb 12, serum iron 160, and RBC 4000... should I take iron supplements in addition to the iron that the multivitamins contain, and does the low ferritin value harm the fetus?

Sent by Mirela, më 17 April 2019 në 07:31

Hello Mirela, you are considered to have an iron deficiency (ferritinemia < 20 ng/ml and the optimal value for females is 60-70 ng/ml). You will receive 2 months of iron preparations with a medicinal dose. Iron supplements are insufficient to replenish the stores

Replay from Dr. Shk. Sotiraq Lako, më 17 April 2019 në 07:44

Hello doctor, I am 27 years old, I did the tests and the results are: rbc-4.44, hgb-12.4, hct-37.2, sediment 54, and ferritin 5.48. I feel continuous fatigue, drowsiness, leg fatigue I can't stand on my feet, and frequent heartbeats. Today I will start the treatment with Tot'hema 3×1, vit C 100 mg 3×1, and Vitadep 1/day.
Is this the right treatment, what do you suggest?

Sent by Dorina, më 21 April 2019 në 01:56

Hello Dorina, you have an iron deficiency. Tothema is not the appropriate preparation. Use Heferol 350 mg 2 x 1 capsule/day + Vitamin C 100 mg 2 x 1 tablet/day, for at least 2 consecutive months. After 2 months of treatment, you will do complete bloodwork + ferritinemia. The optimal value of ferritinemia in adult females is 60-70 ng/ml

Replay from Dr. Shk. Sotiraq Lako, më 21 April 2019 në 05:30

Hello doctor, I am 34 years old. I did the tests and the results are ferritin 5.04, sideremia 41, and hemoglobin 11. I feel fatigued, drowsy, and lack concentration. I have been taking heferol 350 mg once a day and folic acid 5mg once a day for 2 days. Also, for a vaginal infection, I have been taking miraclin twice a day but I don't know if I can take it while I am on the anemia treatment and whether the treatment I use for anemia is the right one. Can foods like coffee and tea be consumed during the period of anemia treatment, what do you suggest?
THANK YOU!

Sent by Matilda, më 22 April 2019 në 09:09

Hello Matilda, you have mild anemia due to iron deficiency. The treatment with Heferol is twice a day, for at least 3 consecutive months. After 1 month, the hemoglobin is checked and after 3 months the ferritinemia, with the aim of their complete correction. There is no problem if you also take antibiotics, they can be taken about 1 hour apart from each other

Replay from Dr. Shk. Sotiraq Lako, më 22 April 2019 në 09:17

Hello doctor, I am 27 years old, I did my tests and the results are: rbc-4.44, hgb-12.4, hct-37.2, sediment 54, and ferritin 5.48. I feel continuous fatigue, drowsiness, leg fatigue, I can't stand on my legs, and frequent heartbeats. Today I will start treatment with Tot'hema 3×1, Vit C 100 mg 3×1, and Vitadep 1/day.
Is this the right treatment, what do you suggest?

Sent by Dorina, më 22 April 2019 në 16:03

Hello Dorina, you have an iron deficiency. Tothema is not the appropriate preparation. Use Heferoli 350 mg 2 x 1 capsule/day + Vitamin C 100 mg 2 x 1 tablet/day, for at least 2 consecutive months. After 2 months of treatment, you will do a complete blood count + ferritinemia. The optimal ferritin value in adult females is 60-70 ng/ml

Replay from Dr. Shk. Sotiraq Lako, më 23 April 2019 në 04:22

Hello doctor! My 25-year-old daughter gets tired quickly, has a temperature of 99.5°F (37.2°C) or slightly below for over a year, complains of a headache which makes her feel nauseous and occasionally vomit. The corners of her lips are cracked, and she has been losing weight for some months now, sometimes appearing as if she might faint. For this reason, she has had several complete blood tests, and the results from a week ago are as follows:
HGB 12.4, RBC 4.83, PLT 236, WBC 7.53, Iron 105, Ferritin 12.71.
I emphasize that the Ferritin level has been decreasing over the years, from 45.3 in 2016, to 18.7 last year, and to 12.71 a few days ago.
So far, she has not taken any medication as I think a diagnosis should be established first. What do you suggest, doctor? Thank you

Sent by Gleda Toli, më 27 May 2019 në 04:52

Hello Gleda, your daughter has an iron deficiency. She will take for 2 months 1-Feroli 350 mg 2 x 1 capsule/day plus 2-Vitamin C 100 mg 2 x 1 tablet/day and after the treatment, you will do: complete blood count + ferritin + Hemoglobin Electrophoresis. The optimal value of ferritin in adult females is about 60-70 ng/ml

Replay from Dr. Shk. Sotiraq Lako, më 27 May 2019 në 07:15

Hello doctor, I did the blood tests because I was feeling unwell, I could hardly catch my breath, and I had total exhaustion and numbness in the joints, and my ferritin came out 8.7, below normal

Sent by Amarilda, më 28 May 2019 në 12:26

Hello Amarilda, you have an iron deficiency (ferritinemia in women < 20 ng/ml). Iron deficiency causes fatigue, weakness, numbness of limbs, loss of balance. If Hb is > 12 gr/dl, you will be treated for at least 2 months, if you also have anemia due to iron deficiency for at least 3 consecutive months, with the goal of correcting the anemia and the iron deficiency itself (*60-70 ng/ml for adult females)

Replay from Dr. Shk. Sotiraq Lako, më 28 May 2019 në 12:47

Hello! I did some tests 2 months ago, and these were the blood values: total leukocytes 5.4, erythrocytes 4.61, hb 11.7, platelets 200, mcv 76, mch 25.2, ferritin 2.31 ng/dl. I took Folifer for 2 months. I wanted to ask for guidance on what to do next? Which tests should I repeat? Thank you!

Sent by Kejsi, më 01 June 2019 në 17:15

Hello Kejsi: complete blood count + ferritinemia + Hemoglobin Electrophoresis. Depending on the value of ferritinemia, the continuation or discontinuation of treatment with iron preparations is evaluated

Replay from Dr. Shk. Sotiraq Lako, më 02 June 2019 në 03:21

Hello! I did some tests 2 months ago and these were my complete blood values leukocytes 5.4 erythrocytes 4.61 hb 11.7 platelets 200 mcv 76 mch 25.2 ferritin 2.31 ng/dl. I took Folifer medication for 2 months. I wanted to ask for your guidance on what to do next? Which tests should I repeat? Thank you!

Sent by Kejsi, më 01 June 2019 në 17:26

Hello Kejsi: complete blood count + ferritin + Hemoglobin Electrophoresis. Depending on the value of the ferritin, the continuation or not of treatment with iron supplements is assessed

Replay from Dr. Shk. Sotiraq Lako, më 02 June 2019 në 03:22

Hello! I had some tests done 2 months ago and these were the results of the complete blood count: leukocytes 5.4, erythrocytes 4.61, hb 11.7, platelets 200, MCV 76, MCH 25.2, ferritin 2.31 ng/dl. I took Folifer for 2 months. I wanted to ask for guidance on what to do next? What tests should I repeat. Thank you!

Sent by Kejsi, më 01 June 2019 në 17:41

Hello Kejsi: complete blood count + ferritinemia + Hemoglobin Electrophoresis. Depending on the value of ferritinemia, the continuation or not of treatment with iron preparations is assessed

Replay from Dr. Shk. Sotiraq Lako, më 02 June 2019 në 03:21

Hello! I did some tests 2 months ago and these were the complete blood count values: leukocytes 5.4, erythrocytes 4.61, hb 11.7, platelets 200, MCV 76, MCH 25.2, ferritin 2.31 ng/dl. I took Folifer as a treatment for 2 months. I would like your guidance on what to do? Which tests should I repeat? Thank you!

Sent by Kejsi, më 01 June 2019 në 17:51

Hello Kejsi: complete blood count + ferritin + Hemoglobin Electrophoresis. Depending on the value of ferritin, the continuation or discontinuation of treatment with iron supplements is evaluated

Replay from Dr. Shk. Sotiraq Lako, më 02 June 2019 në 03:21

Good day, doctor, I need advice from you. My hemoglobin is 6.9. Can I improve it with food?

Sent by elmedin, më 06 June 2019 në 01:31

Hello, Elmedin, as you may have read, it is impossible to correct anemia with food, even more so iron deficiency. Only by taking the right preparation, the right dose, for the right amount of time can you correct the anemia and its cause, iron deficiency (if the anemia is due to iron deficiency)

Replay from Dr. Shk. Sotiraq Lako, më 06 June 2019 në 06:27

Good day, doctor. I want advice from you. My hemoglobin is 6.9. Can I improve it with food?

Sent by elmedin, më 06 June 2019 në 01:31

Hello, Elmedin, as you might have read, it is impossible to correct anemia with food, even more so iron deficiency. Only by taking the right preparation, the right dosage, for the right amount of time can you achieve the correction of anemia and its cause, iron deficiency (if the anemia is due to a lack of iron)

Replay from Dr. Shk. Sotiraq Lako, më 06 June 2019 në 06:27

Hello doctor, first of all, thank you for the detailed writing as I secured a lot of information (about which I was completely unaware).
I have felt powerless for some time, tired, it seems as if my legs are "snapping" from the knees down, and my arms from the elbows down. I also feel drowsy and sweaty.
I do not have these symptoms all the time but occasionally and all together start with fatigue, drowsiness, and sweating...
I did a blood analysis and my hemoglobin turned out to be 11.7 today I also did IRON which turned out to be 34.7 from 59-158 which is the norm (I’m not sure if I’m right about the norm). Before going to the doctor, what do you suggest I do (not that I don't trust the doctors here but from my family's experience the treatment given has not solved my mother's anemia problems.)
Thank you in advance
Have a good day

Sent by Blerta, më 14 June 2019 në 07:17

Hello Blerta, you currently have mild anemia. In most cases, it is due to iron deficiency. You have had your sideremia tested, which is low and supports the iron deficiency, but Ferritinemia is what matters to us. When it is < 20 ng/ml in females, it is considered an iron deficiency and you will be treated with iron for at least 2 consecutive months and after 2 months of treatment, a complete blood + Ferritinemia test will be repeated. For adults, I often use Heferol 350 mg 2 x 1 capsule/day + Vitamin C 100 mg 2 x 1 tablet/day + Vitamin B6 25 mg 2 x 1 tablet/day. Take in the middle or after a meal

Replay from Dr. Shk. Sotiraq Lako, më 15 June 2019 në 07:37

Hello doctor. Thank you for the detailed information. After completing the tests, these are the results I got:
HGB 9.1
HCT 29.8
MCV 73.4
HBA 98.1
HBA2 1.9
TIBC 478.4
FERRITIN 2.3
IRON 2
I am experiencing fatigue and insomnia.
What do you recommend for me? Thank you

Sent by Alma, më 29 June 2019 në 15:29

Hello Ana, you should have also recorded the values of RBC, WBC, Plt. The data are in favor of Anemia due to Iron Deficiency.
1-Heforol 350 mg 2 x 1 capsule/day – 60 capsules/month.
2-Vitamin C 100 mg 2 x 1 tablet/day – 60 tablets/month.
3-Vitamin B6 25 mg 2 x 1 tablet/day – 60 tablets/month.
4-Folic Acid 5 mg 2 x 1 tablet/day – 60 tablets/month.

For at least 3 consecutive months, after 1 month a complete blood count is done and after 3 months of treatment, ferritinemia is checked

Replay from Dr. Shk. Sotiraq Lako, më 30 June 2019 në 11:55

Hello doctor. Thank you for the detailed information. After completing the tests, these were the results:
HGB 9.1
HCT 29.8
MCV 73.4
HBA 98.1
HBA2 1.9
TIBC 478.4
FERRITIN 2.3
IRON 2
I have fatigue and insomnia.
What do you recommend for me? Thank you

Sent by Alma, më 29 June 2019 në 15:41

Hello Ana, you should have also recorded the value of RBC, WBC, Plt. The data are in favor of Iron Deficiency Anemia.

1-Heferol 350 mg 2 x 1 capsule/day – 60 capsules/month.
2-Vitamin C 100 mg 2 x 1 tablet/day – 60 tablets/month.
3-Vitamin B6 25 mg 2 x 1 tablet/day – 60 tablets/month.
4-Folic Acid 5 mg 2 x 1 tablet/day – 60 tablets/month.

For at least 3 consecutive months, after 1 month a complete blood count is done and after 3 months of treatment, ferritinemia as well

Replay from Dr. Shk. Sotiraq Lako, më 30 June 2019 në 11:55

Thanking you for the response, I am also providing the other data. RBC 4.06 WBC 6.3 PLT 387 Age 43 years. Thank you once again

Sent by Alma, më 30 June 2019 në 12:18

Hello Ana, continue as I have written to you before

Replay from Dr. Shk. Sotiraq Lako, më 01 July 2019 në 12:59

Hello doctor, I am Mira, 42 years old. After doing blood tests, my hemoglobin came out to 9.9 and ferritin to 3.33. I have been following the doctor's prescription for 3 days now, taking Tot Hema ampoules three times a day and Vitamin C and a complete Vitamin B, but I noticed in your comments that Tot Hema is not a good preparation. Could you advise me on what to do? I wanted to add that I am experiencing side effects with this medication: nausea, vomiting, headaches, and loss of appetite. I don't feel like eating anything. I would greatly appreciate it if you could answer me

Sent by Mira , më 04 July 2019 në 12:17

Hello Mira, if there is no reaction from the medications, I often use 1-Heferol 350 mg 2 x 1 capsule/day – 60 capsules/month.
2-Vitamin C 100 mg 2 x 1 tablet/day – 60 tablets/month.
3-Vitamin B6 25 mg 2 x 1 tablet/day – 60 tablets/month.
either during or after meals. The treatment will continue for at least 3 consecutive months. The patient will be rechecked with a complete blood count after 1 month and after 3 months of treatment with a complete blood count + ferritinemia + Hemoglobin Electrophoresis

Replay from Dr. Shk. Sotiraq Lako, më 05 July 2019 në 11:02

Hello doctor, I have a 2-year-old daughter with these test results:
Hgb 98. gra% 26.9. Lym% 67.9. Lym 6.7. Mpv 7.7. Plt 437. During the blood examination, some values came out elevated and some low, also iron 13. What could my daughter have since the doctor did not take the case very seriously and the girl got sick again two more times from the throat and now she has lost all her appetite, I have to trick her to feed her when before she used to eat with pleasure and without fuss, thank you

Sent by Njomza, më 08 July 2019 në 09:43

Hello Njomza, the girl has a mild anemia, but it is not the cause of the concerns the girl has (throat illness). You will discuss with the pediatrician, so that iron treatment can be added as well

Replay from Dr. Shk. Sotiraq Lako, më 10 July 2019 në 07:21

Hello, doctor. I am 45 years old, I have done the tests and had these results: RBC 9, MCV 66, MCH 21, serum iron 16, ferritin 4.2. I have never been treated, what do you suggest? Thank you

Sent by brunilda, më 11 July 2019 në 08:16

Hello Brunilda, you need to record the other values as well: RBC, WBC, Plt. Anyway, the data indicates Iron Deficiency Anemia, for which you should be treated with iron supplements, therapeutic dose (200 mg elemental iron), for at least 3 consecutive months, with the aim of initially correcting the anemia (Hb > 12 gr/dl) and ferritinemia (about 60 ng/ml in adult females)

Replay from Dr. Shk. Sotiraq Lako, më 11 July 2019 në 13:04

It is also important to discover the cause of iron deficiency, which generally is repeated blood loss and in women who menstruate, it is the menstruations

Replay from Dr. Shk. Sotiraq Lako, më 11 July 2019 në 13:05

Hello Doctor, I am Margarita, 56, from Korça. I had communicated with you in April 2019. I have done blood tests and have received these results:
WBC 5.4*10, RBC +5.59*10, HGB 14.0s/dL, HCT 43.6%, MCV-78.0fL, MCH -25.0ps, MCHC 32.1s/dL, PLT 148*10, IRON2 15.5ng/dL, Calcium 9.87 mg/dl, LYM 33.3% MXD 10.7% NEUT 56% RDW -SD 44.7 fL RDW CV 14.9% PDW 11.4fL MPV -8.3 fL P LCR 14.8%. Ferritin 18.15ng/mL. I had my ferritin level at 1.6 on March 10, 2019. After treatment with Tardyferon for two months, I have the above results. Please, what do you advise me? Can I take the medication again? I feel that I have improved but sometimes my vision is accompanied by small black spots or one ear makes a little noise some days. Thank you for your willingness and good intention that you always have for your very valuable advice. Wishing you health and all the best

Sent by Margarita Janko, më 13 July 2019 në 07:58

Hello Margarita, you still have iron deficiency. For 2 months take 1-Heferol 350 mg 2 x 1 capsule/day – 60 capsules/month. 2-Vitamin C 100 mg 2 x 1 tablet/day – 60 tablets/month. And after 2 months do ferritinemia

Replay from Dr. Shk. Sotiraq Lako, më 13 July 2019 në 08:39

Hello Doctor, and I would like to ask for a clarification. The use of Duodenal Ferosanol and Tardyferon caused me a reaction, and I'm worried that Heferol might also cause me concern. The concern was in the form of urticaria only on the legs and occasionally on the body or face, more so with Ferosanol and slightly less with Tardyferon. Thank you for your advice

Sent by Margarita Janko, më 15 July 2019 në 13:10

Hello Margarita, you need to try the preparation to evaluate it. If it gives you hives, it's very simple, stop it and other preparations will be evaluated

Replay from Dr. Shk. Sotiraq Lako, më 16 July 2019 në 13:22

Hello Doctor, I would like to ask for a clarification. The use of Duodenal Ferosanol and Tardyferon caused me reactions, I'm worried that Heferol might also cause me trouble. The concern was in the form of urticaria only on the legs and occasionally on the body or face, more so from Ferosanol and slightly less from Tardyferon. Thank you for your advice

Sent by Margarita Janko, më 15 July 2019 në 13:15

Hello Margarita, you need to try the product to assess it. If it gives you hives, it's very simple, stop using it and other products will be evaluated

Replay from Dr. Shk. Sotiraq Lako, më 16 July 2019 në 13:22

Hello Doctor,
I have been experiencing a lot of hair loss.
I have done blood tests and my ferritin came out to be 30.
Vitamin D 12.
My TSH came out to be 4.2 (I have been treated with Euthyrox for 3 months and the value came out to 3.5).
What can I do?

Sent by Anela, më 29 July 2019 në 15:35

Hello Anela, one of the reasons that hair falls out is the lack of iron, which is assessed by the low value of ferritinemia (< 20 ng/ml). A value of 30 ng/ml is a good value and as a rule, it does not justify hair loss. Since we aim for a ferritinemia value for adult females around 60 ng/ml, you can take for 1 month (if you do not have a reaction): 1- Heferol 350 mg 2 x 1 capsule/day – 60 capsules/month. 2- Vitamin C 100 mg 2 x 1 tablet/day – 60 tablets/month. And after 1 month reevaluate both the ferritinemia and the change or not in the hair loss

Replay from Dr. Shk. Sotiraq Lako, më 30 July 2019 në 05:31

Hello Doctor,
I am experiencing a lot of hair loss.
I have done blood tests and my ferritin came out as 30.
Vitamin D 12.
My TSH was 4.2 (I have been treated with eutirox for 3 months and the value came out as 3.5.
What can I do?

Sent by Anela, më 29 July 2019 në 15:35

Hello Anela, one of the reasons for hair loss is the lack of iron, which is assessed with a low ferritin value (< 20 ng/ml). A value of 30 ng/ml is considered normal and usually does not justify hair loss. Since we aim for a ferritin value for adult females around 60 ng/ml, you can take for 1 month (if you do not have a reaction): 1-Heferol 350 mg 2 x 1 capsule/day – 60 capsules/month. 2-Vitamin C 100 mg 2 x 1 tablet/day – 60 tablets/month. And after 1 month re-evaluate both the ferritin and the change or lack thereof in hair loss

Replay from Dr. Shk. Sotiraq Lako, më 30 July 2019 në 05:31

Hello Doctor,
I hope you are well!
I have a 4-year-old daughter and her test values are serum iron 66 ug/dl, ferritin 24.9 ng/ml, hemoglobin 11.8g/dl, and hematocrit 37.7%. She looks pale but is very active as a child; she does not consume meat as she does not like it except in soup broth. Does she need to take any supplements or are these values normal for children of this age?
Thank you,
Marsida

Sent by Marsida , më 01 Agust 2019 në 08:35

Hello Marsida, the values are very good. There is no need for anything and let it play

Replay from Dr. Shk. Sotiraq Lako, më 02 Agust 2019 në 03:15

Hello Dr. Lako. I am very worried because I have received my blood test results and my ferritin is 3; iron 14 while the thyroid hormone is 9.81. I am 44 years old, I have been suffering from iron-deficiency anemia for years and almost every year I have used iron for 2-3 months. Please, what do you recommend? Thank you and good luck with your work!

Sent by Enkeleda, më 14 September 2019 në 03:16

Hello Enkeleda, you certainly have an Iron Deficiency, and it could also be iron deficiency anemia (hemoglobin is not presented), but this is very common. Indeed, you are being treated, but it depends on what kind of preparation you take, what dose, the correction of anemia and iron deficiency is achieved. And what is most important is the cause of the iron deficiency (usually in females it is the Menstrual Cycle), that if it exists and is fully corrected, hematological problems will recur. You can come in one day to discuss and properly address this issue. For TSH, = Hypothyroidism and you will discuss with the endocrinologist, but these are two problems that are not related to each other

Replay from Dr. Shk. Sotiraq Lako, më 15 September 2019 në 04:18

Hello Dr. Lako. I am very worried after receiving my blood test results; my ferritin level is 3; iron 14, while my thyroid hormone is 9.81. I am 44 years old, and have been suffering from iron deficiency anemia for years and nearly every year I have used iron for 2-3 months. Please, what do you recommend? Thank you and good luck with your work!

Sent by Enkeleda, më 14 September 2019 në 03:56

Hello Enkeleda, indeed you have Iron Deficiency, and possibly anemia from iron deficiency (hemoglobin has not been presented), but this is very common. Indeed, you are being treated, but it depends on what preparation you take, what dose, the correction of the anemia and the iron deficiency is achieved. And what is most important is the cause of the iron deficiency (usually in females it is the Menstrual Cycle), if it exists and is fully corrected, hematological problems will recur. You can come one day to discuss and properly treat this problem. For TSH, = Hypothyroidism and you will discuss with the endocrinologist, but these are two problems that have no connection to each other

Replay from Dr. Shk. Sotiraq Lako, më 15 September 2019 në 04:18

Hello Dr. Lako. I am very worried because I have received the results of the blood tests and my ferritin came out as 3; iron 14 while the thyroid hormone 9.81. I am 44 years old, I have been suffering from iron deficiency anemia for years and almost every year I have used iron for 2-3 months. Please, what do you recommend? Thank you and good luck with your work!

Sent by Enkeleda, më 14 September 2019 në 05:25

Hello Enkeleda, certainly you have an Iron Deficiency, and possibly anemia from iron deficiency (hemoglobin is not presented), but this is very common. Indeed, you are treated, but it depends on what preparation you take, what dose, the correction of the anemia and the iron deficiency is achieved. And what is most important is the cause of the iron deficiency (usually in women it is the Menstrual Cycle), which if it exists and is fully corrected, hematological problems will recur. You can come one day to discuss and properly address this problem. For TSH, = Hypothyroidism and you will discuss with the endocrinologist, but these are two problems that have no connection to each other

Replay from Dr. Shk. Sotiraq Lako, më 15 September 2019 në 04:18

Hello Dr. Lako. I am very worried because I have received the results of my blood tests and my ferritin is 3; iron 14 while the thyroid hormone is 9.81. I am 44 years old, I have been suffering from iron deficiency anemia for years and almost every year I have used iron for 2-3 months. Please, what do you recommend? Thank you and good luck with your work!

Sent by Enkeleda, më 14 September 2019 në 07:19

Hello Enkeleda, indeed you have Iron Deficiency, it could also be anemia from iron deficiency (hemoglobin is not presented), but this is very common. Indeed, you are being treated, but it depends on what preparation you are taking, what dose, the correction of the anemia and the iron deficiency is achieved. And what is most important is the cause of the iron deficiency (usually in females it is the Menstrual Cycle), that if it exists and is fully corrected, the hematological problems will recur. You can come one day to discuss and properly treat this problem. For TSH, = Hypothyroidism and you will discuss with the endocrinologist, but these are two problems that have no connection between them

Replay from Dr. Shk. Sotiraq Lako, më 15 September 2019 në 04:18

Hello Dr. Lako. I am very worried because I have received the results of my blood tests and my ferritin turned out to be 3; iron 14 while the thyroid hormone 9.81. I am 44 years old, I have been suffering from iron-deficiency anemia for years and almost every year I have used iron for 2-3 months. Please, what do you recommend? Thank you and good work!

Sent by Enkeleda, më 14 September 2019 në 12:50

Hello Enkeleda, certainly you have Iron Deficiency, it might be anemia from iron deficiency (hemoglobin is not presented), but this is very common. Indeed, you receive treatment, but it depends on which preparation you take, what dose, the correction of anemia, and iron deficiency is achieved. And what's most important is the cause of the iron deficiency (usually in women it is the Menstrual Cycle), which if it exists and is fully corrected, hematological problems will recur. You can come one day to discuss and properly address this issue. For TSH, = Hypothyroidism and you will discuss with an endocrinologist, but these are two problems that have no connection with each other

Replay from Dr. Shk. Sotiraq Lako, më 15 September 2019 në 04:17

Hello Dr. Lako. I am very worried because I have received the results of my blood tests and my ferritin is at 3; iron at 14 while the thyroid hormone is at 9.81. I am 44 years old, I have been suffering from iron deficiency anemia for years and almost every year I have used iron for 2-3 months. Please, what do you recommend? Thank you and good luck with your work!

Sent by Enkeleda, më 14 September 2019 në 14:18

Hello Enkeleda, you definitely have Iron Deficiency, and possibly anemia from iron deficiency (hemoglobin is not presented), but this is very common. Indeed, it can be treated, but it depends on what preparation you take, what dosage, the correction of the anemia and the iron deficiency can be achieved. And what matters most is the cause of the iron deficiency (usually in females it is the Menstrual Cycle), if it exists and is fully corrected, the hematological problems will repeat. You can come one day to discuss and properly address this problem. For TSH, = Hypothyroidism and you will discuss with the endocrinologist, but these are two issues that are not related to each other

Replay from Dr. Shk. Sotiraq Lako, më 15 September 2019 në 04:17

Hello Dr. Lako. I am very worried because I have received the results of my blood tests and my ferritin is 3; iron 14 while the thyroid hormone is 9.81. I am 44 years old, I have suffered from iron deficiency anemia for years and almost every year I have used iron for 2-3 months. Please, what do you recommend? Thank you and good luck with your work!

Sent by Enkeleda, më 14 September 2019 në 17:29

Hello Enkeleda, you certainly have Iron Deficiency, it might also be anemia due to iron deficiency (hemoglobin is not presented), but this is very common. Indeed, it can be treated, but it depends on what preparation you take, what dose, the correction of the anemia and the iron deficiency is achieved. And what is most important is the cause of the iron deficiency (usually in women it is the Menstrual Cycle), if it exists and is completely corrected, the hematological problems will recur. You can come one day to discuss and properly address this issue. For TSH, = Hypothyroidism and you will discuss with the endocrinologist, but these are two problems that are not related to each other

Replay from Dr. Shk. Sotiraq Lako, më 15 September 2019 në 04:17

Thank you for your reply and your time, Dr. Lako! So, I am writing below the missing values: HGB 8.2 (it has been at this level for about 10 years even after the use of iron in various forms; IRONORM, SIDERAL...etc.) WBC 7650; RBC 4,760,000. The menstrual cycle, (which has been seen as the main cause of iron deficiency anemia) has been heavy from the beginning and continues. Should the blood or the thyroid be prioritized, (since due to the busy schedules, my appointment with Dr. Ylli for the thyroid has been set for October 3rd)? And sorry for the question, but what happens if ferritin becomes 0? I thank you once again and wish you good work!

Sent by Enkeleda, më 16 September 2019 në 06:56

Hello Enkeleda, Iron Deficiency Anemia, from the name itself you can understand that the cause of Anemia is Iron Deficiency. So, initially, you must have an iron deficiency (ferritinemia 0-20 ng/ml) and then iron deficiency anemia can develop. So, if there is anemia, ferritinemia is 0 ng/ml. Iron Deficiency = Blood Loss. It has nothing to do with the thyroid. If you do not regulate menstrual blood flow, the treatment will not be successful, and if it is, it will soon be lost, as happens to you. The basic problem = Repeated blood losses, a consequence of this = iron deficiency, a consequence of iron deficiency is anemia

Replay from Dr. Shk. Sotiraq Lako, më 16 September 2019 në 07:49

Hello doctor, I am Adnan from Kosovo, I am 30 years old. I wanted to ask, I have problems with iron, I feel dizzy, I have difficulty breathing sometimes during sleep, and I feel very weak in my body. My hair is falling out, and I am noticing some black spots in my vision. I went to the doctor, had a hemogram test, and was told that I have mild anemia. I was given a therapy to use Heferol once a day for 2 weeks, along with vitamins B2, B6, B12. My hemoglobin level was 124. I wanted to ask for your advice, doctor. Thank you

Sent by Adnan , më 20 September 2019 në 03:57

Hello Adnan, Hb 124 gr/l is considered mild anemia. To see if you have iron deficiency, you should get a ferritin test. If Ferritin < 30 ng/ml, you are considered to have an iron deficiency and should take iron medication. Heferol is the right preparation, but with 2 x 1 capsules/day, for at least 3 months in a row. And it is accompanied by Vitamin C and Vitamin B6, without the need for other vitamins

Replay from Dr. Shk. Sotiraq Lako, më 20 September 2019 në 04:15

Hello doctor, I am Alketa. My mother, who is 80 years old, suffers from heart and thyroid issues. I have done her blood tests, and the results came out with erythrocytes 7.41, Hgb 12.9, MCV 58.2. I did the ferritin, and it turned out to be low. Now, she has started to experience itching in her extremities; she had it before, but now it has become more pronounced and bothersome. What do you advise me to do, please?

Sent by Alketa, më 21 September 2019 në 04:59

Hello Alketa, the number of erythrocytes is increased. With this data, there is a suspicion of Thalassemia Minor, for this you need to do Hemoglobin Electrophoresis. If there is also itching, even more so if ferritin has decreased, it can be suspected of a combination of Thalassemia Minor with Polycythemia. LDH, uricemia, erythrocyte sedimentation rate, Vitamin B12 will also be done. Probably also a JAK2V679F genetic mutation

Replay from Dr. Shk. Sotiraq Lako, më 22 September 2019 në 11:27

Hello doctor. My ferritin has come out to be 259 ng, if the analysis has been done correctly. And my hemoglobin has come out to be 124

Sent by Adnan , më 26 September 2019 në 05:30

What is the value of red blood cells (RBC)?

Replay from Dr. Shk. Sotiraq Lako, më 26 September 2019 në 06:44

Hello Doctor!
In July, my tests showed 60 iron, 10.2 ferritin.
In August, my tests showed 21 iron, 8.8 ferritin.
In September, my tests showed 30.2 iron, 212 ferritin.
I have been treated with Ferrotec, Ferrovit, again Ferrotec 2 tablets + vitamin C 200mg

Sent by ediola, më 26 September 2019 në 06:46

Hello Ediola, the ferritin level is 212 ng/ml or 21.2 ng/ml. It is impossible with the iron supplements you are taking for the ferritin level to go to 212 ng/ml. If it is so, the most likely possibility is that it is a laboratory error

Replay from Dr. Shk. Sotiraq Lako, më 27 September 2019 në 05:07

5.1 Doctor RBC

Sent by Adnan , më 26 September 2019 në 06:49

You probably have Thalassemia Minor. You will do Hemoglobin Electrophoresis

Replay from Dr. Shk. Sotiraq Lako, më 27 September 2019 në 05:05

Thank you, Doctor! So, it turns out to be 21.2 ferritin because this number 212 seemed strange to me, in fact, and I got a little worried!!
And according to these results, is the cause of the decrease in iron and ferritin a gastrointestinal disease or another reason?

Sent by ediola, më 27 September 2019 në 06:32

Hello Ediola, the value of 21.2 ng/ml is a good value, but the optimal value for females is around 60 ng/ml. You need to continue treatment with the right preparation, the right dose, and furthermore, to achieve the desired result. The main cause of iron deficiency is repeated blood loss, where the main role in females is losses with the Menstrual Cycle (95% of the cause) and 5% noticeable or invisible losses with urine, feces, presence of parasites

Replay from Dr. Shk. Sotiraq Lako, më 27 September 2019 në 08:50

Thank you, Doctor! So, the right preparation is ferrotec2+2vitamin C(100mg) or do you have any other suggestion?

Sent by ediola, më 27 September 2019 në 10:12

Ferrotec is an iron supplement. 1-Heferoli 350 mg 2 x 1 capsule/day - 60 capsules/month. 2-Vitamin C 100 mg 2 x 1 tablet/day - 60 tablets/month, during or after meals, for at least 2 consecutive months

Replay from Dr. Shk. Sotiraq Lako, më 28 September 2019 në 07:55

Hello doctor. I have done the tests and my ferritin is 6.06 ng/ml. While the ESR is 35. I have done one venous session, they told me that I need to do 4 more sessions (once every two days) and then continue the treatment once a day with Feramax 100 mg pills and vitamin C. I wanted to ask if you recommend the venous sessions, and if the medication dosage is correct or if it should only be through the pills.
Thank you

Sent by Djana, më 29 September 2019 në 08:52

Hello Djana, we are using intravenous medication. A woman has about 600 mg of iron reserves and if you only have an iron deficiency, without iron deficiency anemia, it would require 6 ampoules of Sol. Ferrovin (which is used here in Albania) 100 mg of iron. After the infusion is complete, ferritin levels are usually measured after 1 week. The intravenous route is the best way to correct it, but we don’t use it often because sometimes there are allergic reactions, phlebitis/local thrombophlebitis. If these do not occur, it always remains the best option. There is no need to continue treatment afterwards with oral preparations, especially since Feramax, in our experience, is not an effective preparation. Furthermore, once the iron stores are replenished, the orally taken iron supplement is no longer absorbed

Replay from Dr. Shk. Sotiraq Lako, më 29 September 2019 në 12:34

Hello, I am Gena, 39 years old...20 days ago, I had complete blood tests including ferritin. The results are Hgb 11.6, Rbc 4.85, Hct 37.58, ferritin 3. I have been taking a medicine called Emacrit and B vital once a day. What do you recommend I do? Should I continue with these medications? I noticed that the name of neither of these medications I am taking was mentioned. Thank you

Sent by Migena Boci, më 01 October 2019 në 03:52

Hello Migena, you have mild Anemia due to Iron Deficiency. The treatment you are using are iron supplements. The most frequently used preparation: Heferol 350 mg 2 x 1 capsule/day plus Vitamin C 100 mg 2 x 1 tablet/day plus Vitamin B6 25 mg 2 x 1 tablet/day for at least 3 consecutive months. After 1 month you will do a complete blood count and after 3 months a complete blood count and ferritinemia

Replay from Dr. Shk. Sotiraq Lako, më 06 October 2019 në 11:58

Hello, doctor. I have done the tests and these are the results:
WBC 9.21, HGB 5.5, RBC 3.72, MCV 52.2, RDWs 47.7, RDWc 23%, HCT 19.43%, MCH 14.8, MCHC 28.3, PLT 257, PCT 0.27%. I have not done FERRITIN

Sent by roi, më 03 October 2019 në 07:31

Hello Roi, you have Iron Deficiency Anemia, even though you haven't done the ferritin test. You need to start treatment to fully correct the anemia and the iron deficiency itself. If possible, also do the ferritin test

Replay from Dr. Shk. Sotiraq Lako, më 06 October 2019 në 12:05

Hello Doctor, please a ferritin of 534, in a 60-year-old man what does it indicate? All the tests done, Complete Blood Count, transaminases, biochemical panel are within normal ranges. Thank you very much for your answer

Sent by Eri, më 03 October 2019 në 10:44

Hello Eri, there is a slight increase in ferritinemia. It doesn't always mean that you have iron overload. Generally, these values are related to alcohol use, liver fattening (ultrasound), rheumatic disease, lipid disorders. Some tests and examinations are done

Replay from Dr. Shk. Sotiraq Lako, më 06 October 2019 në 12:07

Hello doctor, I am 33 years old, I did tests and my ferritin level came out at 2.87. I haven't had problems before and I have never been treated for this. I feel fatigued from time to time, have frequent headaches, meanwhile, I have checked my eyes, teeth, and sinuses and those were okay. Also, I am experiencing a lot of hair loss. Could you recommend any treatment? Thank you in advance!

Sent by Erinda, më 04 October 2019 në 14:00

Hello Erinda, it is certain that you have an iron deficiency and that may justify your complaints. It is the biggest medical problem in the world, especially for women. What is the value of Hemoglobin? Of course, you need to be treated to bring the value of ferritin around 60 ng/ml

Replay from Dr. Shk. Sotiraq Lako, më 06 October 2019 në 12:10

Hello doctor, I did a ferritin test and it turned out to be 2.04. I took medication with heferol and vitamin C for 3 months. My question is, will these medications help me or might I need another medication? Thank you!

Sent by Roi, më 07 October 2019 në 11:38

Hello Roi, is the current ferritin value before or after treatment? If it is before treatment, Heferol is a very good preparation for correction. 2 x 1 capsule/day for 2 months in a row + Vitamin C. After 2 months, it will be repeated

Replay from Dr. Shk. Sotiraq Lako, më 08 October 2019 në 06:22

For health's sake, I am Manuela, I have done blood tests and my ferritin level came out to be 5.8 while sideremia and other tests are normal. I had concerns like tingling in the fingers, severe headaches, arrhythmia (extrasystole), I took medication oviron three times a day, and in the last two months, I took it twice a day (because I still had a bit of anemia) but not regularly (my negligence), in total, I was treated for 5 months, now I have almost three months without medication and the extrasystoles are starting again (the structure of the heart is ok), but I am feeling cold, have flies before my eyes, headache, not continuous. Is it possible that these signs are related to anemia, maybe it has started again, I have encountered hemato but I don't know if this also has a connection with anemia, thank you and good work

Sent by Manuela, më 08 October 2019 në 13:15

Hello Manuela, after receiving treatment, control analyses are also done: complete blood count and ferritin. If Hb is above 12 gr/dl, you do not have anemia, and if your ferritin is around 60 ng/ml, you do not have an iron deficiency either. If these are your values, there is no hematological justification for your complaints

Replay from Dr. Shk. Sotiraq Lako, më 09 October 2019 në 05:01

For health, I am Manuela, I have done blood tests and my ferritin came out at 5.8 while sideremia and other tests are normal. I had concerns such as tingling in the fingers, severe headaches, arrhythmia (extrasystole), I took medication oviron 3 times a day, and in the last two months, I took it twice a day (because I still had a bit of anemia) but not regularly (my negligence), in total, I was treated for 5 months, now I have been almost three months without medication and the extrasystoles are starting again (the structure of the heart is ok), but I am feeling cold, flies in front of my eyes, headache, not continuous. Is it possible that these signs are related to anemia, maybe it has started again, I have encountered hemato but I don't know if this is also related to anemia, thank you and good work

Sent by Manuela, më 08 October 2019 në 13:33

Hello Manuela, after taking the medication, control analyses are also performed: complete blood count and ferritinemia. If Hb is above 12 gr/dl, you do not have anemia, and if your ferritinemia is around 60 ng/ml, you do not have an iron deficiency either. If these are the values, there is no hematological justification for your complaints

Replay from Dr. Shk. Sotiraq Lako, më 09 October 2019 në 05:01

Hello doctor, I am 37 years old and have been dealing with anemia for 3 years. I have been treated for 2 months and stopped today. I did blood tests and my results were WBC 6.62, RBC 3.96, Ferritin 11.2. My family doctor advised me to take ironorm twice a day because my total bilirubin was high but within normal limits. What do you suggest, please? Thank you

Sent by Pranvera , më 11 October 2019 në 11:54

Hello Spring, you need to write ferritinemia as well. If Hb is 12 g/dl and above, you are considered only with iron deficiency and will take Ironorm 3 x 1 capsules/day, for at least 2 consecutive months. If Hb < 12 g/dl you will take it but 3 x 1 capsules/day, for at least 3 consecutive months. The ultimate goal of treatment, Ferritinemia around 60 ng/ml. Bilirubin is not related to iron deficiency, anemia from iron deficiency

Replay from Dr. Shk. Sotiraq Lako, më 13 October 2019 në 04:34

Hello doctor, I am 37 years old and I have been suffering from anemia for 3 years. I have been treated for 2 months and I stopped today. I did the tests and my results were Wbc 6.62 Rbc 3.96 Ferritin 11.2. My family doctor advised me to take ironorm twice a day after my total bilirubin came out high but within the norms. What do you recommend, please? Thank you

Sent by Pranvera , më 11 October 2019 në 11:55

Hello Spring, you need to also check ferritinemia. If Hb is 12 gr/dl and higher, you are considered to only have an iron deficiency and will take Ironorm 3 x 1 capsules/day, for at least 2 months consecutively. If Hb < 12 gr/dl, you will take it but 3 x 1 capsules/day, for at least 3 months consecutively. The ultimate goal of the treatment, Ferritinemia around 60 ng/ml. Bilirubin is not related to iron deficiency, anemia from iron deficiency

Replay from Dr. Shk. Sotiraq Lako, më 13 October 2019 në 04:35

Hello Doctor. I thank you very much for the answers you give to our questions. I have done some routine analyses for my 9-year-old son and his eosinophils came out at 4.8 with a reference value of <3, and monocytes at 5.9 with a reference value of < 5, RDW - SD 33.6 with a reference of 37. Should I follow up on this? Does he need further tests? Thank you!

Sent by Eri, më 12 October 2019 në 11:02

Hello Eri, up to 4% is considered normal for Eosinophils. And if we evaluate it as slightly elevated (as a rule, we do a second check), we assess allergies and parasitoses. There is no blood disease in this case. Monocytes are normal (2-10%)

Replay from Dr. Shk. Sotiraq Lako, më 13 October 2019 në 04:36

Hello doctor!
I did the tests and these are the results HGB 10.8 RBC 4.55•10^12 HCT 29.32 MCH 23.8 MCHC 36.9. The symptoms are: I feel tired, ringing in the ears, flies before my eyes, and what terrifies me the most is that I have stinging in the side of the heart and mild pain in the left shoulder and arm. I have done an echo for the heart and all four valves are normal.
Could the stinging be a result of anemia or should I check back with the cardiologist? Thank you

Sent by Diana, më 12 October 2019 në 14:24

Hello Diana, you have mild anemia, probably due to Iron Deficiency (you should check ferritin levels). As a rule, this level of anemia does not cause heart concerns. However, this anemia has a good side, if treated properly it is entirely correctable, and once corrected, your concerns should be re-evaluated. Also consider the possibility of muscle or stomach pains

Replay from Dr. Shk. Sotiraq Lako, më 13 October 2019 në 04:39

Hello doctor!
I did the tests and have these results HGB 10.8 RBC 4.55• 10 to the power of 12 HCT 29.32 MCH 23.8 MCHC 36.9. The symptoms are as follows: I feel tired, ringing in the ears, I see spots before my eyes, and what terrifies me the most is that I have stabbing pains in the area of the heart and slight pain in the left shoulder and arm. I have had an echo done for the heart and all four valves are normal.
Could the stabbings be a result of anemia or should I check again with the cardiologist? Thank you

Sent by Diana, më 12 October 2019 në 14:33

Hello Diana, you have mild anemia, probably due to Iron Deficiency (you should get ferritinemia). As a rule, this level of anemia does not cause heart problems. However, the good thing about this anemia is that if treated properly, it is completely correctable, and once corrected, your concerns should be re-evaluated. Also consider the possibility of muscle or stomach pain

Replay from Dr. Shk. Sotiraq Lako, më 13 October 2019 në 04:39

Hello doctor!
I did the tests and have these results HGB 10.8 RBC 4.55• 10 to the power of 12 HCT 29.32 MCH 23.8 MCHC 36.9 The symptoms are as follows: I feel tired, ringing in my ears, I have flies before my eyes, and what terrifies me the most is that I have stabbing pain on the side of my heart and slight pain in the left shoulder and arm. I have done an echo for the heart and all four valves are normal.
Could the stabbing pains be a result of anemia or should I check again with the cardiologist? Thank you

Sent by Diana, më 12 October 2019 në 14:33

Hello Diana, you have mild anemia, probably due to Iron Deficiency (you should get a ferritin test). As a rule, this level of anemia does not cause heart concerns. However, this anemia has a positive side, if treated properly it is completely correctable, and once corrected, your concerns should be reassessed. Also consider the possibility of muscle or stomach pains

Replay from Dr. Shk. Sotiraq Lako, më 13 October 2019 në 04:39

Hello, I am interested to know what precautions we should take when we have high iron in the blood, please what should we eat..

Sent by Romario Tafa, më 15 October 2019 në 04:46

Hello Romario, as a rule it is impossible to avoid iron from foods. A certain amount of iron is also absorbed from the metal pots where the food is cooked. If you have increased ferritin with Transferrin Saturation 20-50%, there is no need to follow a diet to limit iron. It might only make sense in cases with increased ferritin and Transferrin Saturation > 50%

Replay from Dr. Shk. Sotiraq Lako, më 15 October 2019 në 06:12

Hello doctor.
I have had
Some concerns about low blood pressure 80 over 50, frequent palpitations like tearing. I want to start a pregnancy. And some tests showed ferritin 12.6ng/ml
Rbc 3.76 L, hgb 12.2 g/dl, hct 38, pct 160, plt 204. Iron 102.
Is anything else necessary besides folic acid?

Sent by Ina, më 16 October 2019 në 05:35

Hello Ina, currently you have an Iron Deficiency, it is not an obstacle for pregnancy, but you need to correct it. If you live in Albania, you will take for 2 consecutive months: Ironorm 3 x 1 capsules/day and after 2 months you will have a complete blood test, ferritinemia. Once in a lifetime, hemoglobin electrophoresis is also done

Replay from Dr. Shk. Sotiraq Lako, më 16 October 2019 në 07:04

Hello doctor. Thank you for your answers. I have discovered almost a year ago that my Ferritin level is low. In November, my Ferritin was 4.3. I drank "FeroSac" which dissolves in water for 3 months. It seemed to somewhat stabilize me. Since I was experiencing dizziness, loss of balance... In March, I measured my Ferritin and it turned out to be 14. Now I did the tests again and my Ferritin is 24.1ng m/L but every time I get my period, the symptoms start again: loss of balance, a bad feeling, bad mood. I live in Athens and I will present the analyses as they have written them in medical language.
(WBC) 7.09 K/ μl
(NEUT) 59.9%
(LYM) 28,4%
(MONO)10.3%
(EOS) 0,7%
(BAS0),07%
(RBC) 4,67 M/μL
(HGB) 14,5 gr/dL
(HTC) 42,9%
(MCV) 91,fl
(MCH) 30,9 pg
(MCHC) 33,7gr/dL
(RDW-CV) 12,2%
(PLT) 384K/μl
(PCT) 0,35%
(MPV) 9,0 fl
(PDW) 15,1%
FERRITIN 24.1ng m/L
I would like to know your opinion on the treatment... Thank you for your time

Sent by Elda Shehaj, më 16 October 2019 në 06:25

Hello Elda, you are currently classified only with Iron Deficiency. In Greece, you will receive treatment for 1.5-2 months with Ferro sanol duodenal 100 mg 2 x 1 capsule/day and after the treatment, ferritinemia will be repeated. The goal is around 60 ng/ml in females

Replay from Dr. Shk. Sotiraq Lako, më 16 October 2019 në 07:02

Hello doctor. It seems that this medication is not available in Greece. Can you give me a suggestion? Or another medication that we could replace it with, so that my people can buy it for me in Albania? Thank you..

Sent by Elda Shehaj, më 16 October 2019 në 08:15

Hello Elda, that drug is in Greece, because patients from Greece take it

Replay from Dr. Shk. Sotiraq Lako, më 17 October 2019 në 02:42

Hello doctor. It seems to me that this medication is not available in Greece. Can you give me a suggestion? Or another medication that we can replace it with so that my people can buy it for me in Albania? Thank you..

Sent by Elda Shehaj, më 16 October 2019 në 08:21

Hello Elda, that preparation is in Greece, because patients from Greece take it

Replay from Dr. Shk. Sotiraq Lako, më 17 October 2019 në 02:42

Hello Doctor,

I did the blood analysis with formula today and my hemoglobin came out to be 8.8

I will be doing the ferritin and sideremia analysis in the following days.

Is this anemia curable?

Sent by anisa decka, më 16 October 2019 në 09:18

Hello Anisa, what we can say with the value that you have presented, is the existence of anemia, its degree (moderate). Other values of the analysis (RBC, WBC, Plt) should also be written, and even better, the value of ferritin. If it is anemia due to iron deficiency, if you take the right preparation, the right dose for the right period, it is usually completely correctable

Replay from Dr. Shk. Sotiraq Lako, më 17 October 2019 në 02:44

Hello Doctor,

I did a blood analysis with formula today and my hemoglobin came out 8.8

I will do the ferritin and sideremia analysis in the following days.

Is this anemia curable?

Sent by anisa decka, më 16 October 2019 në 09:54

Hello Anisa, what we can say about the value you have presented is the existence of anemia, its degree (moderate). Other values of the analysis (RBC, WBC, Plt) should also be written, and even better, the value of ferritin. If it is anemia due to iron deficiency, by taking the right preparation, the right dose for the right time, it is usually completely correctable

Replay from Dr. Shk. Sotiraq Lako, më 17 October 2019 në 02:44

Hello doctor, 3 years ago I was treated for iron deficiency, recently I have had hair loss and for this reason I did an iron test, ferritin came out 5.3, while hemoglobin 12.2. I am 20 years old. Today I went to the general practitioner and he suggested a TSH hormone test and also did not recommend iron medication. I wanted to know your opinion

Sent by Marjana, më 16 October 2019 në 14:50

Hello Marjana, you currently do not have anemia, but you have an iron deficiency (ferritinemia < 20 ng/ml). You will be treated for 2 months to correct it (the minimum standard is 20 ng/ml and the optimal value for adult females is about 60 ng/ml).

1-Heferol 350 mg 2 x 1 capsule/day – 60 capsules/month.
2-Vitamin C 100 mg 2 x 1 tablet/day – 60 tablets/month.

Iron deficiency is one of the reasons why hair falls out, but the good news is that it's correctable with treatment (it does not self-correct). If ferritinemia reaches values close to 60 ng/ml and the hair continues to fall out, there are certainly other causes, including a decrease in thyroid gland function (elevated TSH)

Replay from Dr. Shk. Sotiraq Lako, më 17 October 2019 në 02:48

Hello doctor, 3 years ago I was treated for iron deficiency, lately I have been experiencing hair loss and for this reason, I did an iron analysis; ferritin came out at 5.3, while hemoglobin at 12.2. I am 20 years old. Today I went to the general practitioner and they suggested a TSH hormone analysis and also did not recommend iron medication. I wanted to know your opinion

Sent by Marjana, më 16 October 2019 në 15:01

Hello Marjana, you currently do not have anemia, but you have an iron deficiency (ferritinemia < 20 ng/ml). You will be treated for 2 months to correct it (the minimum standard is 20 ng/ml and the optimal value for adult females is about 60 ng/ml). 1-Heferol 350 mg 2 x 1 capsule/day – 60 capsules/month. 2-Vitamin C 100 mg 2 x 1 tablet/day – 60 tablets/month.
Iron deficiency is one of the reasons why hair falls out, but the good news is that it is correctable with treatment (it does not self-correct). If ferritinemia reaches values close to 60 ng/ml and the hair continues to fall out, there are certainly other causes, including a decrease in thyroid gland function (elevated TSH)

Replay from Dr. Shk. Sotiraq Lako, më 17 October 2019 në 02:48

Hello Doctor, I did a blood test on my son and these values came out WBC 13.9 RBC 4.53 HCT 35.3 HB 12.2 Ferritin 26.04 Sideremia 25.4 should we start iron treatment?

Sent by aurora, më 17 October 2019 në 12:23

Hello Aurora, how old is the child?

Replay from Dr. Shk. Sotiraq Lako, më 18 October 2019 në 03:58

6 years old Doctor

Sent by aurora, më 18 October 2019 në 04:40

Hello, currently there are good values and there is no need to take iron medication

Replay from Dr. Shk. Sotiraq Lako, më 18 October 2019 në 05:05

6 years old Doctor

Sent by aurora, më 18 October 2019 në 04:41

Hello, currently has good values and does not need to take iron medication

Replay from Dr. Shk. Sotiraq Lako, më 18 October 2019 në 05:05

Hello Doctor! Currently, I am taking ferrous sulfate (heferol) 2x1 & vitamin C 2x1, but I have had some concerns with my glands perhaps I don't know but I feel enlargement under the armpit, difficulty swallowing, severe pain during the menstrual cycle, hair loss, nail breakage, should I do something about this?

Sent by ediola, më 18 October 2019 në 05:23

Hello Ediola, the lack of iron causes the concerns you have. The treatment you are receiving is appropriate for addressing the correction of Iron Deficiency

Replay from Dr. Shk. Sotiraq Lako, më 18 October 2019 në 05:51

Hello doctor. I am Mrs. Elda from Athens. Ferro sanol duodenal 100 mg, 2 x 1 capsules/day that you prescribed for me to take. It is missing in all of Athens, they have removed it from circulation. What do you recommend? Since I am also traveling to the USA in a few days and I have a loss of balance... I can ask my relatives from Albania for another medicine that you know of. Thank you

Sent by Elda Shehaj, më 20 October 2019 në 06:49

Hello Elda, here in Albania we use Ironorm 3 x 1 capsules/day, for at least 3 consecutive months

Replay from Dr. Shk. Sotiraq Lako, më 20 October 2019 në 14:04

I did the tests because my hemoglobin came out as 8.8 (perhaps because my cycle repeated twice last month) for ferritin and sideremia and the values were:

Ferritin 4.4

Sideremia 13.8

I want to take the medicine in Italy because I have been told that the anemia medication we have here causes problems with the teeth, what do you advise me to take?

Thank you

Sent by anisa decka, më 20 October 2019 në 09:51

Hello Anisa, any iron preparation, if chewed in the mouth, causes blackening of the teeth, regardless of its origin. In Italy, you will receive: Ferrograd 105 mg 2 x 1 tablets/day plus Vitamin C 100 mg 2 x 1 tablets/day plus Vitamin B6 25 mg 2 x 1 tablets/day, for at least 3 consecutive months. It's not the fault of one or two menstrual cycles, many cycles have caused the iron deficiency (ferritinemia 4.4 ng/ml) and as a consequence of the iron deficiency, anemia might occur (which in your case has set in) and this is a problem that has existed for a long time

Replay from Dr. Shk. Sotiraq Lako, më 20 October 2019 në 14:07

Hello doctor, I am Elda from Athens. Ironorm 3×1 capsule per day, 3 pills per day if I'm not mistaken??? Before food or after???.. Thank you very much, doctor, for providing your answers. You are a humane person. All the best

Sent by Elda Shehaj, më 21 October 2019 në 12:48

Hello Elda, yes, the treatment is as it is written. Usually, iron is taken before meals, but since it irritates the stomach lining, we give it during or after meals. The timing of taking it doesn't matter, what's important is taking the correct daily dose and the duration of the treatment

Replay from Dr. Shk. Sotiraq Lako, më 22 October 2019 në 10:52

Hello Doctor! I need an urgent solution, it seems that as soon as my body's iron stores are replenished, I start having diarrhea, and it seems worse when I consume a lot of fruits and vegetables, and my stool is like black tar. This happened to me two or three months ago when my stores were replenished, I started having diarrhea, but now with ferrous sulfate 2x1 and vitamin C 2x1 it started faster than with the supplements!!

Sent by ediola, më 22 October 2019 në 03:14

Hello Ediola, I don't know what you refer to as filling the reserves. Indirectly, we consider them filled when ferritinemia in women is 60-70 ng/ml. Iron stores are generally with the liver. So, we do not wait for ferritinemia to reach 60-70 ng/ml for you to have diarrhea or other problems. This is the value that Nature has set, and Nature is perfect. But when you take iron supplements, you might experience epigastric pain, diarrhea, stools are always dark colored, but this is a result of irritation of the gastrointestinal mucosa by the iron preparation and the need for dark color is all the excess iron

Replay from Dr. Shk. Sotiraq Lako, më 22 October 2019 në 10:56

And one more thing Doctor, my hands are always cold!

Sent by ediola, më 22 October 2019 në 03:16

Hello Ediola, one of the reasons for feeling cold is the lack of iron. But if this is corrected (ferritin at least 20 ng/ml and the optimal value around 60 ng/ml) and they are still such, other causes are looked into

Replay from Dr. Shk. Sotiraq Lako, më 22 October 2019 në 10:57

Hello doctor. My iron level is low at 24.1. I don't have anemia, just an iron deficiency. I am 42 years old. Every time I get my periods, my resistance drops. I wanted to know if the lack of iron can cause a loss of balance and all the body weight falls on the legs and I keep my toes tightly clenched to the floor because it feels like I will fall as if my leg muscles won't hold me, dizziness, Are these signs of iron deficiency or is there something else? Thank you

Sent by Edlira, më 23 October 2019 në 03:29

Hello Edlira, iron deficiency causes concentration loss, sometimes even fainting spells. Currently, it is still within the lower range of normal. You will take for 1.5 months 1-Heferol 350 mg 2 x 1 capsule/day – 60 capsules/month. 2-Vitamin C 100 mg 2 x 1 tablet/day – 60 tablets/month. and you will repeat the ferritin test (optimal value around 60 ng/ml). By improving/correcting the iron deficiency, your complaints will be assessed to see if there is improvement or not

Replay from Dr. Shk. Sotiraq Lako, më 24 October 2019 në 10:31

I read all the comments and got a lot of information, but I wanted to ask about something. With these indicators:
HGB 13.0, MCV 68.0fl, MCH 22.9pg, RDW SD 16.3%, PLT. 301, PDW 14.7, PCT 0.319, and FERRITIN 19.21. IS THERE A NEED FOR MEDICATION? AND IF SO, WHAT KIND AND DOSAGE PLEASE. AGE 16 YEARS. IT IS A BOY. THANK YOU

Sent by Anila, më 25 October 2019 në 08:02

Hello Anila, you also need to write down the value of the erythrocytes. The minimal normal ferritin level for males is 30 ng/ml, so there is a need to take iron supplements. The patient's weight should also be known for the medication. The treatment is used for 2 consecutive months and after 2 months a complete blood check, ferritin levels, and hemoglobin electrophoresis should be reevaluated

Replay from Dr. Shk. Sotiraq Lako, më 25 October 2019 në 08:11

Hello doctor, I have a 15-year-old daughter with a hemoglobin level of 10.5 and ferritin 2.7. It has been a week since I started treating her with Tardyfer 80mg one vial per day

Sent by Klodiana, më 30 October 2019 në 11:15

Hello Klodiana, the girl has mild anemia, probably due to iron deficiency. Tardyferon 1 tablet/day is insufficient. Depending on the weight, 2 or 3 tablets/day will be taken, for at least 3 consecutive months. The patient will be rechecked with complete blood count after 1 month and after 3 months of treatment with complete blood count + ferritinemia + Hemoglobin Electrophoresis

Replay from Dr. Shk. Sotiraq Lako, më 31 October 2019 në 06:20

Hello. Two days ago, I had blood tests done and they showed that I have pronounced anemia. I was given Ferro3 forte and a vitamin, but I don't feel any improvement. Could you give me more information on what I should use?

Sent by Safete merkaj , më 13 November 2019 në 12:31

Hello Safete, severe anemia is considered when the hemoglobin value is < 7 gr/dl. Only Iron Deficiency Anemia is treated with iron, so in addition to the hemoglobin value, you need to write the value of the red blood cells (RBC) and the value of ferritinemia. If it is indeed Iron Deficiency Anemia, you will be treated with iron, but Ferro3 is not the right preparation (it has only 30 mg of elemental trivalent iron). We treat with divalent iron (ferrous sulfate and ferrous fumarate salts are the best and cheapest) and the therapeutic dose is 180-200 mg of elemental iron per day, for at least 3 consecutive months

Replay from Dr. Shk. Sotiraq Lako, më 13 November 2019 në 13:44

Hello doctor! Please, I have done a ferritin test and it came back 1.7, what should I take, thank you

Sent by Ina, më 15 November 2019 në 00:04

Hello Ina, for women the normal minimum value of ferritin is 20 ng/ml and the optimal value is around 60 ng/ml. Of course, you will receive medication. It depends on age, weight. In principle, treatment lasts at least 2 months in a row, with the goal of reaching the above values

Replay from Dr. Shk. Sotiraq Lako, më 15 November 2019 në 06:13

Hello doctor!
I am 22 years old, have Hb 11.4, and ferritin 19.91. I am being treated with FerroDep, but I have stomach pain, stabbing pain in the left side, and after I eat, I get a bitter taste in my mouth. Also, the pain extends to my back. I should mention that because of the stomach pain, I am also taking Nolpaza. Could FerroDep be the cause of these pains, or should I consider some other concern?

Sent by Dajana, më 19 November 2019 në 01:49

Hello Dajana, iron supplements can cause gastrointestinal issues. It is easy to verify, just interrupt it for a few days (nothing happens, especially since it is not the right preparation to correct iron stores, iron supplement) and assess the concerns. If they go away, they are related to it; if not, there are other causes

Replay from Dr. Shk. Sotiraq Lako, më 19 November 2019 në 07:39

Hello doctor!
I wanted to ask a question that is off your topic, but at least to get an opinion on where to turn. For almost two weeks, I have had only stinging on the side of my heart, a feeling like burning or stabbing, and sometimes I also have it on the right side; these are not related to activity at all. I am 22 years old, have had an echocardiogram, and the 4 valves are normal. I emphasize that sometimes when I stand up or change position, I have a pain only in one spot as if it were injured. I don't know if I should go again to a cardiologist or assess some neuromuscular pain. I also have stings in my back and neck.
I would be immensely grateful for an answer

Sent by Eliona, më 28 November 2019 në 11:43

Hello Eliona, more muscle pain. If no injuries are found, you will be referred to a rheumatologist and a physiotherapist

Replay from Dr. Shk. Sotiraq Lako, më 29 November 2019 në 11:51

Hi! I am Gena, I have written to you before about the problem of anemia. I took the medication prescribed by the doctor for 3 consecutive months and then did the tests again. I did the hematocrit and B vitamins tests. The results are HGB 12.0, RBC 4.59, and HCT 35.9... WBC 7.74 and ferritin 5.79. What do you recommend I should continue to use? Thank you!

Sent by Migena Boci, më 30 November 2019 në 06:54

Hello Migena, you have taken iron supplements. It is impossible to replenish the iron stores with them. You need to take the right medication (to increase 180-200 mg of elemental iron per day) for at least 2 consecutive months, aiming for a minimum ferritin level of 20 ng/ml and an optimal value of around 60 ng/ml

Replay from Dr. Shk. Sotiraq Lako, më 30 November 2019 në 14:08

Hello doctor, I did the tests and the results were these: hemoglobin 12.4 and ferritin 4.4ng/mL. I have been given ferro dep 2x1 for 3 months, please can you tell me if this is enough or should I take any other medication?

Sent by stela, më 30 November 2019 në 08:25

Hello Stela, you currently have an Iron Deficiency and it cannot be corrected with supplements, such as the one you have taken. It is necessary to take 180-200 mg of elemental iron per day, for at least 2 consecutive months with the goal of reaching a minimum ferritin level for women of 20 ng/ml and an optimal value of about 60 ng/ml. The supplement you are taking is not the right one

Replay from Dr. Shk. Sotiraq Lako, më 30 November 2019 në 14:10

Hello doctor, I did the tests and the results were these: hemoglobin 12.4 and ferritin 4.4ng/mL. I have been given ferro dep 2x1 for 3 months, please can you tell me if this is enough or should I take some other medication?

Sent by stela, më 30 November 2019 në 08:28

Hello Stela, you currently have an Iron Deficiency and it cannot be corrected with supplements, such as the one you have been taking. You need 180-200 mg of elemental iron per day, for at least 2 consecutive months with the goal of reaching a minimum ferritin level for females of 20 ng/ml and an optimal value of around 60 ng/ml. The preparation you are taking is not the right one

Replay from Dr. Shk. Sotiraq Lako, më 30 November 2019 në 14:14

Hello doctor! I have done the tests and the results are: hemoglobin 11.9, ferritin 68.67, erythrocytes 466, leukocytes 630. All the others - hematocrit, MCV, MCH, are within normal ranges. RDW 48.8 showed to be high. What prompted me to do the tests was massive hair loss. Until the child turned 5 months old, I was taking vitamins, afterward, I neglected and did not take them. The feeding was breastmilk. That's when the problem started. Could this have been the cause? Do I need iron supplements? I emphasize that I have sent the tests to the doctor. They told me I don't need it. Thank you!

Sent by Brunilda, më 03 December 2019 në 14:08

Hello Brunilda, one of the reasons why hair falls out is the lack of iron. But you don't have this issue, your ferritin is 68 ng/ml. You will consult with the dermatologist

Replay from Dr. Shk. Sotiraq Lako, më 06 December 2019 në 11:01

Hello doctor! I have done the tests and my Ferritin came out 4, Hgb 12.5, serum iron 110, leukocytes 456, erythrocytes 423, MCV 91.7. What medication should I take to correct it?

Sent by Alba, më 07 December 2019 në 09:55

Hello Alba, you are currently classified with Iron Deficiency. The minimum value of ferritin in women is 20 ng/ml and the optimal value is about 60 ng/ml. To prescribe treatment, you need to know the age, the causes of iron deficiency, weight, etc. The principle is 180-200 mg of elemental iron per day, for at least 2 months of treatment

Replay from Dr. Shk. Sotiraq Lako, më 07 December 2019 në 12:11

Hello doctor! I have done the tests and my Ferritin came out 4, Hgb 12.5, serum iron 110, leukocytes 456, erythrocytes 423, MCV 91.7. What medication should I take to correct it?

Sent by Alba, më 07 December 2019 në 09:55

Hello Alba, you are currently classified with Iron Deficiency. The minimum value of ferritinemia in females is 20 ng/ml and the optimal value is around 60 ng/ml. To prescribe medication, you need to know the age, the causes of the iron deficiency, weight, etc. The principle is 180-200 mg of elemental iron per day, for at least 2 months of treatment

Replay from Dr. Shk. Sotiraq Lako, më 07 December 2019 në 12:11

Hello Doctor! I have been treated for two months with heferol 2+2 vitamin c. But last night I had blood from the mouth and I do not understand whether it is from the mucous membranes of the mouth or the stomach. I have vomited blood 2-3 times!

Sent by ediola, më 14 December 2019 në 00:25

Hello Ediola, Heferol does not cause such concerns. The origin of the bleeding, from the nose and from behind going into the mouth, from the stomach, the mucosa of the mouth, the lung, must be evaluated. You should consult the respective specialists

Replay from Dr. Shk. Sotiraq Lako, më 14 December 2019 në 10:10

Hello!
I have some kind of allergy or reaction that mostly appears on my hands and feet but also on my body. They appear as swellings and I feel a bit of itching and mostly as if they are stings, the area where it appears becomes like a long swollen stripe, meaning not like pimples. Could this be related to an iron deficiency or something else? Thank you

Sent by Elda, më 15 December 2019 në 12:55

Hello Elda, if confirmed, iron deficiency can cause skin dryness and cracking at the corners. Iron deficiency is fully treatable with medication, which means that the complaints should be eliminated. Probably, your complaints are very likely to be allergy-based, and you will be referred to a dermatologist, allergist

Replay from Dr. Shk. Sotiraq Lako, më 16 December 2019 në 07:55

Hello Doctor! My test results came out as follows: Leukocytes 5.37, Erythrocytes 4.37, Hemoglobin 13.6, Hematocrit 40.1, Platelets 276, Erythrocyte Sedimentation Rate 20.

Sedimentation 20

Serum Iron 97.5
Ferritin 103

Sent by ediola, më 18 December 2019 në 05:28

Hello Ediola, the current analysis of the peripheral blood is normal. The ferritin levels are also normal. There is no need to continue with the Heferoli treatment. The ferritin levels will be retested in 6 months

Replay from Dr. Shk. Sotiraq Lako, më 18 December 2019 në 09:27

Hello, my child is 13 months old and has these blood test results: WBC=H 10.8, LYMF 3.8, GRAN 6.2, MID 0.8, LYMF 35.4, GRAN 57.6, MID 7.0, HCT=L 30.2, MCV L 68.1, RBC 4.43, HGB L 95, MCH L 21.4, MCHC 314, RDW %15.6, RDEa 48.5, PCT 0.28, MPV 8.5, PLT 308, PDW 11.0, LPCR W3.2. Please provide me with an answer, thank you for your understanding

Sent by Dona, më 19 December 2019 në 14:48

Hello Dona, your child has mild anemia, probably due to iron deficiency. You will discuss with the pediatrician to get the iron supplements, with the right dose for the child's weight. If possible, ferritinemia should also be done

Replay from Dr. Shk. Sotiraq Lako, më 19 December 2019 në 15:44

Hello Doctor! I wanted to ask if it's possible that a few months ago I had a ferritin level of 8, hemoglobin (Hgb) 13, iron 100, whereas now, after undergoing a three-month iron treatment, my ferritin result is 20 and Hgb 14.5? From the leukocyte formula, my eosinophils# is 0.11

Sent by Alba, më 20 December 2019 në 12:20

Hello Alba, yes, it is very possible. It can only be achieved with medication. However, the dosage of the preparation and the preparation itself that you use are important. A ferritin level of 20 ng/ml is the minimum normal value for females, with the optimal value being around 60 ng/ml. Typically, treatment with the right preparation and dosage continues for about 2 more months

Replay from Dr. Shk. Sotiraq Lako, më 22 December 2019 në 10:34

Doctor, hello
I have muscle and joint pain on the left side of my body, while on the right side less often, sometimes I also have stabbing pain in the middle of my back. I am 22 years old, I have done a complete blood count and I have been found to have mild anemia Hg 11.4 while leukocytes 6000. For what reason might I have joint pain and is it necessary to do more detailed analysis for the white cells or is it something not worrying that is only related to the muscles?
Thank you

Sent by Sindi, më 05 January 2020 në 16:01

Hello Sindi, you have mild anemia. You will also note the value of erythrocytes. For anemia, you will also do ferritinemia and Vitamin B12. For muscle pain, Vitamin D and electrolytes in the blood

Replay from Dr. Shk. Sotiraq Lako, më 06 January 2020 në 07:14

Sindi is writing to you again. I have 4,400,000 red blood cells, ferritin 19.91. I haven't done the B12 analysis yet. Thank you

Sent by Sindi, më 06 January 2020 në 07:38

Hello Sindi, you have mild Anemia, probably due to iron deficiency. You need to be treated for at least 2 months with iron supplements, a therapeutic dose until the correction of ferritinemia (optimal value for women is about 60 ng/ml)

Replay from Dr. Shk. Sotiraq Lako, më 07 January 2020 në 03:24

Hello doctor, I am 40 years old and here are the results of my blood tests: wbc-7.3; rbc-4.2; hgb-10.1; hct-32; mcv-75.8; mch-23.9; rdw_sd 46.7; rdw_cv 15.8. My current medication is TARDYFERON 80mg / 3 times a day. Please give me your opinion. Thank you

Sent by Ornela, më 09 January 2020 në 08:59

Hello Ornela, you have mild anemia, probably due to iron deficiency. The treatment you have received is correct. It should continue for at least 3 months, with the aim of correcting the anemia after 1 month of treatment and ferritinemia after 3 months of treatment (minimum 20 ng/ml, optimal value around 60 ng/ml)

Replay from Dr. Shk. Sotiraq Lako, më 11 January 2020 në 12:47

Meanwhile, ferritinemia is 3.36

Sent by Ornela, më 09 January 2020 në 09:03

Hello Ornela, you have mild anemia, probably due to iron deficiency. The treatment you have received is correct. It needs to continue for at least 3 months, with the goal of correcting the anemia after 1 month of treatment and the ferritin levels after 3 months of treatment (minimum 20 ng/ml, optimal value around 60 ng/ml)

Replay from Dr. Shk. Sotiraq Lako, më 11 January 2020 në 12:48

Hello doctor. I am 37 years old. I have done tests, my ferritin is 7ug/L and leukocytes: 3580/mm³. I have a lot of hair loss and pain, I don't understand if it's back pain or the kidneys. Can I please have an explanation?

Sent by Albina, më 04 February 2020 në 12:58

Hello Albina, you have an iron deficiency which needs to be treated and could be the reason why your hair is falling out, but not for the pain. You also have a slight decrease in leukocytes, which is not the cause of hair loss and pain. There could be one or several reasons why these values are like this

Replay from Dr. Shk. Sotiraq Lako, më 05 February 2020 në 08:17

Hello doctor. I am 37 years old. I have done tests, ferritin is 7ug/L and leukocytes: 3580/mm³. I am experiencing a lot of hair loss and pain, I don't understand if it's back pain or the kidneys. Can I have an explanation, please?

Sent by Albina, më 04 February 2020 në 13:39

Hello Albina, you have an iron deficiency, which needs to be treated and might be the reason for hair loss, but not for the pain. You also have a slight decrease in leukocytes, which is not the cause of hair loss and pain. There might be one or several reasons why these values are like this

Replay from Dr. Shk. Sotiraq Lako, më 05 February 2020 në 08:17

Hello doctor.
My daughter is 6 years old, she has thalassemia minor. After Ferro-Duretter treatment, we repeated the blood tests and HGB is 10.0gr/dl while ferritin is 5.8 ng/ml. She is an active child and has no health problems.
Thank you

Sent by Edona , më 10 February 2020 në 11:31

Hello Edona, Thalassemia Minor is not treated. Iron is used to treat Iron Deficiency, Anemia from Iron Deficiency. Currently, the girl has Iron Deficiency. You will be directed to a pediatrician to receive the appropriate treatment, for the complete correction of ferritin levels

Replay from Dr. Shk. Sotiraq Lako, më 11 February 2020 në 11:29

Hello doctor.
My daughter is 6 years old, she has thalassemia minor. After the ferrodue treatment, we repeated the blood tests and HGB 10.0gr/dl while ferritin 5.8 ng/ml. She is an active child and does not have health problems.
Thank you

Sent by Edona , më 10 February 2020 në 11:36

Hello Edona, Thalassemia Minor is not treated. Iron is used to treat Iron Deficiency, Iron Deficiency Anemia. Currently, the girl has Iron Deficiency. You will be directed to a pediatrician to receive the proper medication for the full correction of ferritin levels

Replay from Dr. Shk. Sotiraq Lako, më 11 February 2020 në 11:29

Hello!
I am 45 years old, I had a vaginal hysterectomy a month ago. Now I have a little bleeding.
I did the tests and the results are these: PT 64%
INR 1.23, Hemoglobin 10.2, Hematocrit 30, Erythrocytes 3.60.
Please can you briefly tell me what I should do. Should I use aspirin to stop any thrombus since my PT values are 64%.
Please evaluate my message, I am very worried

Sent by Matina, më 11 February 2020 në 10:58

Hello Matina, you currently have mild anemia. PT 64%, if it's okay, it isn't associated with hemorrhage or thrombosis and there's no need to take anything. You will do: ferritinemia, repeat PT/INR, APTT, Fibrinogen

Replay from Dr. Shk. Sotiraq Lako, më 11 February 2020 në 11:33

Hello!
I am 45 years old, I had a vaginal hysterectomy a month ago. Now I have some bleeding.
I did the tests and the results are these: PT 64%
INR 1.23, Hemoglobin 10.2, Hematocrit 30, Red blood cells 3.60.
Please can you briefly tell me what I should do. Should I use aspirin to prevent any clots since my PT values are 64%.
Please evaluate my message, I am very worried

Sent by Matina, më 11 February 2020 në 11:01

Hello Matina, you currently have mild anemia. PT 64%, if it is normal, is not associated with bleeding or thrombosis and there is no need to take anything. You will do: ferritinemia, repeat PT/INR, APTT, Fibrinogen

Replay from Dr. Shk. Sotiraq Lako, më 11 February 2020 në 11:33

Hello!
I am 45 years old, I had a vaginal hysterectomy a month ago. Now I have some bleeding.
I did the tests and the results are these: PT 64%
INR 1.23, Hemoglobin 10.2, Hematocrit 30, Erythrocytes 3.60.
Please, can you briefly tell me what I should do. Should I use aspirin to prevent any clots since my PT values are 64%.
Please evaluate my message, I am very worried

Sent by Matina, më 11 February 2020 në 12:15

Hello Matina, you currently have mild anemia. PT 64%, if it's okay, is not associated with bleeding or thrombosis and there's no need to take anything. You will do: ferritinemia, repetition of PT/INR, APTT, Fibrinogen

Replay from Dr. Shk. Sotiraq Lako, më 13 February 2020 në 08:28

Thank you for your response, doctor. I will do the tests tomorrow. A PT of 64% is concerning, can you please tell me if there's something urgent that needs quick action?

Sent by Matina, më 11 February 2020 në 12:46

It is a value close to the norm and does not cause anything. It needs to be repeated

Replay from Dr. Shk. Sotiraq Lako, më 13 February 2020 në 08:29

Thank you for your response, doctor. I will do the tests tomorrow. A PT of 64% is concerning, please tell me, is there something urgent that needs quick action?

Sent by Matina, më 11 February 2020 në 13:18

It is a value close to the norm and does not cause anything. It needs to be repeated

Replay from Dr. Shk. Sotiraq Lako, më 13 February 2020 në 08:29

Hello. I am in the 7th month of pregnancy and have been diagnosed with anemia. FERRITIN 4.20, SERUM IRON 78, hemoglobin 11.1, leukocytes 7.7, RBC 3.58, HCT 32.7, MCV 91.3, MCH 31, MCHC 33.9, PLT 222, LYM% 17.2, MXD% 7, Neut% 75.8, Lym 1.3, MXD 0.5, Neut 5.9, RDW-CV 12.7, RDW-SD 45.6, PDW 15.1, MPV 10.9, P-LCR 32.8

Sent by ina , më 12 February 2020 në 07:25

Hello Ina, you have mild Anemia, probably due to Iron Deficiency. You need to be treated with iron preparations (divalent iron, capsules or tablets. Ferrous sulfate or ferrous fumarate are the best, for at least 3 consecutive months)

Replay from Dr. Shk. Sotiraq Lako, më 13 February 2020 në 08:30

Doctor, I am Bledi. Every 6 months, I get tests done to know where I stand. Last July, my hemoglobin was 14, hematocrit 41, ferritin 30. I took iron for 3 months, and now my results are ferritin 41, hemoglobin 15.2, and hematocrit 43.2. How are the values, doctor?

Sent by Bledi, më 13 February 2020 në 09:42

Hello Bledi, the value of the hemoglobin is very good. Ferritin is within normal values, but at the lower limit of the norm. In men, the optimal value is around 100 ng/ml. There is room to continue the proper iron treatment for 1-1.5 months

Replay from Dr. Shk. Sotiraq Lako, më 14 February 2020 në 09:46

Hello Doctor. I have repeated the tests, and the Prothrombin values have gone from 64 to 54, INR 49 without medication, ferritin 80 while fibrinogen 410, Sedimentation rate 55. Are these values a consequence of the surgery I had (Vaginal Hysterectomy), or is the low level of prothrombin a result of something else? Please tell me what I should do to normalize the prothrombin level

Sent by Matina, më 17 February 2020 në 10:33

Hello Matina, no, it has nothing to do with the surgery you had. What was the APTT?

Replay from Dr. Shk. Sotiraq Lako, më 18 February 2020 në 06:15

Hello!
APTT was 30.5
What could be the cause of a PT of 54, what other tests should I do?
Thank you for the responses and accuracy

Sent by Matina, më 19 February 2020 në 07:16

APTT is normal. We evaluate the factors of the Internal Coagulation Pathway. Their quantity and function - dependent on Vitamin K

Replay from Dr. Shk. Sotiraq Lako, më 19 February 2020 në 09:19

Thank you, Doctor
Should I do the coagulation factor tests?

Sent by Matina, më 19 February 2020 në 11:05

Hello Matina, we cannot solve much more through this communication. A complete medical history needs to be taken, to discuss the possible causes, the possibility of correction

Replay from Dr. Shk. Sotiraq Lako, më 20 February 2020 në 07:21

Doctor, and another question regarding anemia. I have done an anemia panel for my daughter and got these results: Ferritin 9.95. Whereas Serum Iron 54. Could you tell me something about this? Thank you for the answer

Sent by ami, më 20 February 2020 në 07:32

Hello Ami, the value of ferritin is within the normal range for children, but at the lower limits. It is possible to use iron supplements, with a recommendation from the pediatrician

Replay from Dr. Shk. Sotiraq Lako, më 21 February 2020 në 09:07

Doctor, and another question regarding anemia: I have conducted an anemia panel for my daughter, and these are the results: Ferritin 9.95. While the Serum Iron is 54. Could you tell me something about this? Thank you for your answer

Sent by ami, më 20 February 2020 në 08:33

Hello Ami, the ferritin level is within the normal range for children, but at the lower limits. Iron supplements can be used, with a recommendation from the pediatrician

Replay from Dr. Shk. Sotiraq Lako, më 21 February 2020 në 09:08

Doctor, and another question regarding anemia. I have done an anemia panel for my daughter, and these are the results: Ferritin 9.95. While Serum Iron is 54. Could you tell me something about this? Thank you for the answer

Sent by ami, më 21 February 2020 në 02:56

Hello Ami, the ferritin level is within the normal range for children, but at the lower limits. Iron supplements can be used, with a recommendation from the pediatrician

Replay from Dr. Shk. Sotiraq Lako, më 21 February 2020 në 09:07

HELLO DOCTOR. I HAVE DONE THE ANEMIA TESTS. Fe-Iron 30 mcg/dl AND FERRITIN 31.2 ng/ml. What do you suggest, doctor?

Sent by Erisa, më 06 March 2020 në 10:24

Hello Erisa, the ferritinemia value is good. The minimum normal value for females is 20 ng/ml and the optimal value is around 60 ng/ml

Replay from Dr. Shk. Sotiraq Lako, më 07 March 2020 në 09:20

Hello doctor, I had a cesarean section 6 weeks ago. I experienced numbness in my fingers, legs, and a bit on my face. I did blood tests and it turned out my iron level was low at 16, while all other parameters were normal. I have been taking iron supplements and post-pregnancy vitamins recommended by my gynecologist. For the past 3 days, I have had black-colored stools. Does this come from taking the supplements? I emphasize that I continue to have slight bleeding after giving birth. Thank you for your answer

Sent by Julia, më 07 March 2020 në 04:56

Hello Julia, iron supplements do cause the feces to have a dark color. 16 is sideremia or feritinemia. Similarly, the iron preparation you are taking should be evaluated. In my experience, obstetrician-gynecologists prescribe suboptimal medication in terms of dosage and duration

Replay from Dr. Shk. Sotiraq Lako, më 07 March 2020 në 09:21

Hello doctor, I had a cesarean section 6 weeks ago. I had numbness in the fingers, legs, and a bit on the face. I did blood tests and my iron level was low at 16, while all other parameters were normal. I have taken iron supplements and post-pregnancy vitamins recommended by my gynecologist. I have had black stool for 3 days. Does this come from taking the supplements? I emphasize that I continue to have some bleeding after giving birth. Thank you for your answer

Sent by Julia, më 07 March 2020 në 05:18

Hello Julia, iron supplements do cause the feces to have a dark color. 16 is sideremia or ferritinemia. Similarly, the iron preparation you are taking should be evaluated. In my experience, obstetrician-gynecologists prescribe suboptimal medication in terms of dosage and duration

Replay from Dr. Shk. Sotiraq Lako, më 07 March 2020 në 09:20

Hello Doctor! I have an 8.5-month-old daughter and I did the tests (complete blood + sideremia) recommended by the pediatrician. The result was this: WBC 13.8h / lym 10.5 h / RBC 5.10 / hgb 10.0 L / HCT 32.9 L / PLT 384 Sideremia 15

Sent by Albana, më 07 March 2020 në 07:24

Hello Albana, instead of sideremia, ferritinemia should be done. Low ferritinemia is 100% indicative of iron deficiency. Low sideremia is also an indicator of inflammation. Anyway, your daughter has mild anemia, probably due to iron deficiency. You will discuss with the pediatrician the use of iron supplements

Replay from Dr. Shk. Sotiraq Lako, më 07 March 2020 në 09:11

Hello doctor, I underwent a cesarean section 6 weeks ago. I have had numbness in my fingers, legs, and slightly in my face. I did blood tests and my iron levels were low at 16, while all other parameters were normal. I have been taking iron supplements and post-pregnancy vitamins recommended by my gynecologist. For the past 3 days, I have had black stools. Does this come from taking the supplements? I emphasize that I continue to have some bleeding after giving birth. Thank you for your response

Sent by Julia, më 07 March 2020 në 09:09

Hello Julia, iron supplements do give the feces a dark color. 16 is sideremia or ferritinemia. Likewise, the iron preparation you are taking should be assessed. In my experience, obstetrician-gynecologists prescribe suboptimal medication in terms of dose and duration

Replay from Dr. Shk. Sotiraq Lako, më 07 March 2020 në 09:12

Hello doctor, I had a cesarean section 6 weeks ago. I experienced numbness in my fingers, feet, and a bit in my face. I did blood tests and my iron level was low at 16 while all other parameters were normal. I have taken iron supplements and post-pregnancy vitamins recommended by my gynecologist. For the past 3 days, I have had black stools. Does this come from taking the supplements? I should note that I continue to have a bit of bleeding after giving birth. Thank you for your response

Sent by Julia, më 14 March 2020 në 16:55

Hello Julia, the black stools come from iron supplements. For their need, ferritinemia is done. If it is < 20 ng/ml, it is considered an iron deficiency and you need to take the appropriate medication for the required duration

Replay from Dr. Shk. Sotiraq Lako, më 15 March 2020 në 12:50

Hello Dr. Sotiraqi, the one writing to you is Esmeralda, 36 years old. I have had 3 births by surgery at the ages of 21, 25, and 33. Ever since the first birth, because I did not continue taking iron supplements, I experience anemia twice a year due to my period lasting 7 days and being very heavy. My recent tests showed bad results: Hemoglobin 7.5 g/dl, RBC 3.53, HCT 22.4%. I have been using Ferro 3, 2 vials a day for a week and then 1 vial a day for 2 weeks. Now, I have been taking Tardyferon 2×1 and vitamin C 2×1. What should I do? How long should I continue the treatment, or should I change it? Thank you

Sent by Esmeralda , më 15 March 2020 në 06:23

Hello Esmeralda, you have anemia, moderate grade. Ferro 3 is a waste of time and money. Tardyferon 3 x 1 tablet per day, for at least 3 months straight. Vitamin C, Vitamin B6, Folic Acid

Replay from Dr. Shk. Sotiraq Lako, më 15 March 2020 në 12:51

Hello doctor, I had a cesarean section 6 weeks ago. I have had numbness in my fingers, legs, and a bit on my face. I did blood tests and it turned out my iron was low, at a value of 16, while all other parameters were normal. I have taken iron supplements and post-pregnancy vitamins recommended by the gynecologist. I have had black stools for 3 days. Does this come from taking the supplements? I emphasize that I continue to have some bleeding after giving birth. Thank you for your answer

Sent by Julia, më 18 March 2020 në 07:23

Hello Julia, the black-colored feces come from iron supplements. Their need is determined by ferritinemia. If it is < 20 ng/ml, it is considered an iron deficiency, and you need to take the appropriate medication for the necessary period

Replay from Dr. Shk. Sotiraq Lako, më 18 March 2020 në 10:30

Hello
I am a 21-year-old girl, since September I felt a strong heartbeat that I could feel throughout my chest. It happened from time to time, making me scared. I have had thyroid checks which were fine, 2 ECGs which were okay, blood tests where the ferritin levels were low at 8, where the doctor told me that it is normal for frequent heartbeats. I have been taking iron supplements and vitamin C all of March, but I still experience that strong heartbeat more frequently, thinking as if it is a bad sign. Reading different symptoms makes me think of bad things and puts me in great anxiety. I think that the ECGs I have done might not have shown if there is a problem.
Could it be from the low iron value or could it be something else?

Sent by Sara, më 26 March 2020 në 17:18

Hello Sara, the lack of iron has no connection with heartbeats, neither increased nor decreased. Ferritinemia of 8 ng/ml is low. You need treatment. Iron supplementation is not sufficient. The therapeutic dose is 180-200 mg of elemental iron per day, for 1.5 - 2 months

Replay from Dr. Shk. Sotiraq Lako, më 27 March 2020 në 12:30

Hello doctor. I'm Ana, 22 years old. I have been suffering from anemia for 4 years... Initially, I had severe bleeding for almost a month and as a result, I became anemic. My hemoglobin was 4.6, and I had no iron stores; they were empty. I received treatment for 3 months. But after some time, the anemia reappeared. I underwent treatment again for 2-3 months, and my tests came back good... but these days, I did a check-up, and again my hemoglobin was low and my iron stores were empty. I did a specific test, and it turned out that my anemia was not congenital but acquired. The problem is that the medications I am taking are damaging my stomach and liver since I was diagnosed with hepatitis A. For my periods, I took a medicine called Diane-42 (if I'm not mistaken), and for over a year, my periods come once every 3-4-6 months... I have done check-ups, but the gynecologist told me that I had some microcysts... I took medication, but my periods are still irregular. Whenever I take medication, it normalizes, but after it ends, they become irregular again... I don't understand my condition. The anemia recurs, my cycle continues irregularly, my liver with hepatitis... I am tired both physically and psychologically

Sent by Ana, më 26 March 2020 në 18:05

Hello Ana, when you are treated, there are two objectives: 1) correction of anemia Hb > 12 gr/dl and then continuing treatment for the correction of ferritin levels to a minimum of 20 ng/ml, but the optimal value is around 60-70 ng/ml. If you don't achieve these objectives, you have not been treated properly. We are not interested in CM of rares. It has nothing to do with Hepatitis A. Oral medication can harm the stomach, but not the liver

Replay from Dr. Shk. Sotiraq Lako, më 27 March 2020 në 12:29

Hello doctor. My 17-year-old daughter had blood tests in February and had these values: wbc 4.94 rbc 4.64 hgb 13.30 hct 39.20 mcv 84.40 mch 28.70 mchc 34.00 plt 266.00
serum iron 97.88 and ferritin 10.63. I was concerned about the ferritin value, but the doctor said with this hemoglobin value we do not give iron. Now my daughter has started to lose her hair and has a lack of appetite, her hands and feet are always cold. Please, does she need medication and what treatment? Thank you!

Sent by Jeta, më 08 April 2020 në 16:39

Hello Jeta, your daughter does not have anemia, but she has an iron deficiency (ferritinemia < 20 ng/ml). Hair loss is due to the lack of iron. She needs to take 180-200 ng/ml of elemental iron per day, for 1.5-2 months, aiming to achieve a ferritinemia > 20 ng/ml as the minimum and an optimal value of about 60 ng/ml

Replay from Dr. Shk. Sotiraq Lako, më 10 April 2020 në 09:40

doctor. I have bone and leg pain, headache after the cycle. I am 50 years old with hemoglobin 11.7 and ferritin 24. Do the leg pains especially at the ankles come from anemia?

Sent by drita, më 16 April 2020 në 15:17

Hello Drita, as a rule when we talk about Iron Deficiency Anemia, it is not possible to have Hb < 12 gr/dl if you have ferritinemia 24 ng/ml, so when you do not have an iron deficiency, you cannot have anemia. As a rule, your complaints are not related to hematological changes

Replay from Dr. Shk. Sotiraq Lako, më 17 April 2020 në 12:31

doctor. I have bone and leg pain, headache after my cycle. I am 50 years old with hemoglobin 11.7 and ferritin 24. Do leg pains especially in the calf come from anemia?

Sent by drita, më 16 April 2020 në 15:21

Hello Drita, as a rule, when we talk about Iron Deficiency Anemia, it is not possible to have Hb < 12 gr/dl if you have ferritinemia 24 ng/ml, meaning when you do not have an iron deficiency, you cannot have anemia. As a rule, your complaints are not related to hematological changes

Replay from Dr. Shk. Sotiraq Lako, më 17 April 2020 në 12:31

Hello, doctor. I am in the 7th month of pregnancy and my tests came out like this: WBC-9.3, RBC-3.43, HGB-9.5, HCT-29.5, and ferritin 18.21. The doctor recommended I stop the prenatal vitamins and told me to increase the dose of the strong birvit pill I was taking with the vitamins from 1 to 2 pills per day for 1 month. 1 birvit strong pill contains 245mg of ferrous fumarate, vitamin C 150mg, folic acid 400mcg, and vitamin B12 3.75 mcg. Is this dosage correct? Is it too much or within the normal range? Thank you

Sent by Redian, më 25 April 2020 në 05:33

Hello Redian, you have anemia, probably from a lack of iron. I would recommend Ironorm 3 x 1 capsules/day, for at least 2 months straight. After 1 month of treatment, a complete blood count will be done and after 2 months, ferritin levels will be checked

Replay from Dr. Shk. Sotiraq Lako, më 26 April 2020 në 06:57

Hello doctor! Two years ago, I was diagnosed with iron deficiency, with a ferritin level of 8. After two rounds of treatment with 350mg heferol 2×1 and 80mg tardyferon 2x1, my ferritin level reached 13. After that, I went a year without taking any medication, and in my most recent tests, my ferritin level has dropped back down to 8. Please doctor, what do you suggest? Thank you

Sent by Alda, më 23 May 2020 në 13:08

Hello Alda, and when ferritinemia has reached 13 ng/ml, it is not considered corrected. Correction = the minimal normal value for females 20 ng/ml and the optimal value around 60 ng/ml. Heferol is the best preparation for replenishing the stores, and the dose is twice a day for at least 2 consecutive months. If you have done so and it has reached 13 ng/ml, another preparation will be evaluated, or you will switch to treatment with intravenous preparations

Replay from Dr. Shk. Sotiraq Lako, më 24 May 2020 në 13:49

Hello doctor! I did the tests and I have an iron deficiency, ferritinemia 7.8. They advised me folifer 2×1 and vitamin C. What do you suggest, is this the right treatment? Thank you

Sent by Lizeta, më 30 May 2020 në 05:39

Hello Lizeta, it is one of the preparations that is used. You can continue it for 1.5 - 2 months and repeat the ferritinemia. The minimum normal value is 20 ng/ml and the optimal value is around 60 ng/ml

Replay from Dr. Shk. Sotiraq Lako, më 30 May 2020 në 09:51

Hello doctor! I have a 1-year-old daughter who, after some tests, was found to have mild iron-deficiency anemia. The pediatrician advised me to give her an Iron vit supplement, 1ml, and vitamin C. I have been giving it to her continuously for three months because, due to the pandemic, I could not repeat the tests. My concern is that her first two upper teeth have started to darken at the roots. What could be the cause, and is it related to the iron? Thank you in advance!

Sent by Albana, më 08 June 2020 në 14:44

Hello Albana, oral iron supplements can cause teeth staining. Before continuing, you will undergo a complete blood test, ferritinemia, and depending on those, the need for taking them will be assessed

Replay from Dr. Shk. Sotiraq Lako, më 09 June 2020 në 09:02

Hello doctor! I have been diagnosed with a uterine fibroid and as a consequence, I had problems with iron. Hemoglobin 6, ferritin 3, serum iron 25. I was treated for 1 month with Tot'hema (4 ampoules per day) and for 3 months with Heferol 2 per day. I have been taking vitamin C and Vitadep for 4 months. Now I have reached values of hemoglobin 12, ferritin 40, and serum iron 11. I don't know if I should continue the treatment because the serum iron is a bit low. Thank you

Sent by Gerta, më 11 June 2020 në 16:11

Hello Gerta, currently you have normal values. If you no longer have increased menstrual bleeding, you can stop the medication. After 6 months, recheck with ferritinemia

Replay from Dr. Shk. Sotiraq Lako, më 12 June 2020 në 07:53

Thank you for the response. These days I am undergoing surgery for fibromyoma hoping to solve the problem of hemorrhaging. I wanted to ask you something last. Even though my values are normal now, my hair continues to fall out in a frightening manner (I have been taking biotin for hair for 3 months). Maybe the continuation of hair loss is a consequence of the fibromyoma? Given that the blood values have stabilized? Thank you for the help

Sent by Gerta, më 12 June 2020 në 08:18

Hello, hair does not fall out from fibroids. By resolving the bleeding, you will save yourself from iron losses. One of the reasons why hair falls out is the lack of iron. There are other reasons as well, the specialist is a dermatologist

Replay from Dr. Shk. Sotiraq Lako, më 13 June 2020 në 02:27

Hello doctor! I did some tests a month ago and my hemoglobin, calcium, and vitamin D came back low. Meanwhile, my HDL is normal and my LDL is low. My C protein came out very good. My concern is the terrifying hair loss and I am really alarmed. Currently, following the advice of the local doctor, I am taking Ironorm-1 capsule per day and Vitamin D 4400 IU-1 capsule per day. What tests do you suggest I do to further investigate the situation? Thank you and good work!

Sent by Bujare, më 08 July 2020 në 16:18

Hello Bujare, the only hematological reason for hair loss is iron deficiency. For this, you will do a ferritin test and if it is < 20 ng/ml, you are considered to have an iron deficiency and you will receive the appropriate iron treatment for at least 2 months in a row until the ferritin level is corrected (about 60 ng/ml)

Replay from Dr. Shk. Sotiraq Lako, më 10 July 2020 në 10:25

Hello doctor. 8 months ago, from a routine checkup, the results of the tests were vitamin D 10.7 and ferritin 12.26. I have been taking ironorm 1 tablet/day and vitamin D 1 tablet/day 2000 IU. A week ago, I repeated the tests and the results were vitamin D 35.26 and ferritin 37.45. I wanted to emphasize the fact that I am trying to get pregnant. My question was whether I should continue with ironorm or should I stop it since it has been a long time that I have been taking it? Can it bring side effects or can I continue to use it since it helps with the pregnancy I am planning for? Thank you and I wish you good work

Sent by Ina, më 19 July 2020 në 02:34

Hello, currently you have optimal levels of ferritin and vitamin D. There is no obstacle for you to become pregnant. The medicinal dose of Ironorm is 3 x 1 capsules/day, for at least 3 consecutive months, but even as you have taken it, what matters is that the correction of ferritin has been achieved. You may discontinue it, by rechecking ferritin after 2-3 months. Pregnancy eliminates the menstrual cycle, but increases iron consumption for both the mother and the child

Replay from Dr. Shk. Sotiraq Lako, më 19 July 2020 në 08:18

Hello doctor. Eight months ago, from a routine check-up, the result of the tests was vitamin D 10.7 and ferritin 12.26. I have been taking Ironorm 1 tablet/day and vitamin D 1 tablet/day 2000 IU. A week ago, I repeated the tests and the results were these: vitamin D 35.26 and ferritin 37.45. I wanted to emphasize the fact that I am trying to get pregnant. My question was whether I should continue Ironorm or should I stop it since I have been taking it for a long time? Can it bring side effects or can I continue to use it since it helps with the pregnancy I am planning? Thank you and I wish you good work

Sent by Ina, më 19 July 2020 në 06:09

Hello, currently you have optimal levels of ferritinaemia and vitamin D. There is no obstacle for you to become pregnant. The medicinal dose of Ironorm is 3 x 1 capsules/day, for at least 3 consecutive months, but even as you have taken it, what matters is that the correction of ferritinaemia has been achieved. You can stop it, by rechecking the ferritinaemia after 2-3 months. Pregnancy eliminates the menstrual cycle, but increases the iron consumption for both the mother and the child

Replay from Dr. Shk. Sotiraq Lako, më 19 July 2020 në 08:18

Hello doctor. About 8 months ago, from a routine checkup, the result of the tests was vitamin D 10.7 and ferritin 12.26. I have taken 1 ironorm tablet/day and vitamin d 1 tablet/day 2000 IU. A week ago, I repeated the tests and the results were these: vitamin D 35.26 and ferritin 37.45. I wanted to emphasize the fact that I am trying to get pregnant. My question was whether I should continue the ironorm or should I stop it since it has been a long time that I have been taking it? Can it bring side effects or can I continue to use it since it helps with the pregnancy I am planning for? Thank you and I wish you good work

Sent by Ina, më 19 July 2020 në 07:18

Hello, currently you have optimal levels of ferritin and vitamin D. There is no obstacle for you to become pregnant. The medicinal dose of Ironorm is 3 x 1 capsule/day, for at least 3 months in a row, but even as you have taken it, what matters is that the correction of ferritin has been achieved. You can stop it, by checking the ferritin again after 2-3 months. Pregnancy eliminates the menstrual cycle, but increases the iron consumption for the mother and child

Replay from Dr. Shk. Sotiraq Lako, më 19 July 2020 në 08:18

Hello doctor!
I have done the tests and I have these results WBC-4.90 RBC-3.49 HGB-10.97 hematocrit-33.07 MCV-94.83 MCH-31.46 erythrocyte sedimentation rate 13. The analyses before these were done in February with these values WBC-5.7 RBC-3.98 HGB-11.9 HCT-35.6 MCV-89.4 MCH-30. I am 43 years old, without a cycle for 1 year, I have been treated with methotrexate for arthritis for 5 months. Please, I would like your opinion on these results. Thank you

Sent by Jeta, më 20 July 2020 në 07:32

Hello Jeta, you have mild anemia. It could be inflammatory due to arthritis, it could also be an effect of treatment with MTX. We measure ferritinemia, folic acid. In case of their deficiency, you will be treated with the respective preparations

Replay from Dr. Shk. Sotiraq Lako, më 21 July 2020 në 08:21

Hello doctor, I wanted to ask you something. My 3-month-old baby is vomiting blood, and I brought him/her to the hospital. They told me to get admitted, and I have been here for a week now. I want to know what could cause my baby to vomit blood. I give him/her formula because I don't have breast milk. Initially, I fed him/her Hipp Organic 1, but now I am feeding Nan lactose-free

Sent by Arbesa Bucolli, më 25 July 2020 në 06:41

Hello Arbesa, This is a problem that you will continue to discuss with the pediatrician. I treat patients > 14 years old

Replay from Dr. Shk. Sotiraq Lako, më 27 July 2020 në 06:29

Hello, I am 25 years old. I had ferritin and serum iron tests done 4 months ago, and my ferritin level was 9.98 and serum iron was 23. I was treated with kentoferro for three months, and my ferritin level is 11.98, which means I have not seen any improvement. What should I do?

Sent by Maria Agolli, më 12 Agust 2020 në 12:12

Hello Maria, you have an iron deficiency, and the treatment you are using is not appropriate, so you cannot expect improvement. For 1.5-2 months, a bivalent iron preparation should be taken (Ferrous Sulfate or Ferrous Fumarate), the therapeutic dose is 200 mg of elemental iron per day, the minimum goal is ferritinemia 20 ng/ml

Replay from Dr. Shk. Sotiraq Lako, më 13 Agust 2020 në 08:31

Hello, I am 25 years old. I had ferritin and serum iron tests done 4 months ago and my ferritin level was 9.98 and serum iron was 23. I have been treated with Kentopherro for three months and my ferritin is now 11.98, which means I have not seen any improvement. What should I do?

Sent by Maria Agolli, më 12 Agust 2020 në 15:16

Hello Maria, if you have an iron deficiency, the treatment you are using is not appropriate, so you cannot expect improvement. For 1.5-2 months, a bivalent iron preparation should be taken (Ferrous Sulfate or Ferrous Fumarate), the therapeutic dose is 200 mg of elemental iron per day, with a minimum goal being a ferritin level of 20 ng/ml

Replay from Dr. Shk. Sotiraq Lako, më 13 Agust 2020 në 08:31

Hello Doctor, I am 23 years old and today I did a ferritin test and it came out to be 4.87 ng/ml. What do you recommend I take? I have been suffering from hair loss, fatigue, frequent heart palpitations, weakness, and irritability for 5 years. Thank you if you respond. I have often taken various vitamins for hair, zinc, etc., but without effect

Sent by Denisa, më 15 Agust 2020 në 03:08

Hello Denisa, you have an iron deficiency which is one of the reasons why hair falls out. You need to do a complete blood count to see if you have anemia or not. If you don't have anemia (Hb>12 gr/dl), you will be treated for 2 months with Heferol 350 mg 2 x 1 capsule/day + Vitamin C 100 mg 2 x 1 tablet/day, if you also have anemia, for at least 3 months in a row and at the end of the treatment, the analyses will be rechecked

Replay from Dr. Shk. Sotiraq Lako, më 17 Agust 2020 në 02:43

Hello doctor, I am 23 years old. I did a ferritin test and it came out to 4.87 ng/ml. I have been suffering from hair loss and hair breakage for 5 years. I feel tired, irritated, and have frequent heart palpitations. I have taken various vitamins for hair but without effect. Thank you if you can answer what I can take?

Sent by Denisa, më 15 Agust 2020 në 08:29

Hello Denisa, you have an iron deficiency which is one of the reasons why hair falls out. You need to also do a complete blood count, to see whether or not you have anemia. If you do not have anemia (Hb>12 gr/dl), you will be treated for 2 months with Heferol 350 mg 2 x 1 capsule/day + Vitamin C 100 mg 2 x 1 tablet/day, if you also have anemia, for at least 3 consecutive months and at the end of the treatment, the analyses will be rechecked

Replay from Dr. Shk. Sotiraq Lako, më 17 Agust 2020 në 02:43

Hello doctor. I have a 17-year-old daughter. From the tests done, it resulted that she had anemia and iron deficiency, with ferritin lower than 10. I treated her with ironorm twice a day, one pill for 3 consecutive months. I repeated the tests and the results are as follows:
WBC:5.0, RBC 4.47, HGB 11.9, HCT 39, MCV 87, MCH 26.7, MCHC 30.5, RDW 13.4, MPV 8.8 AND FERRITIN 20.33. I am continuing ironorm again, because the doctor told me that ferritin needs to be up to 90 before stopping, but she is having some problems with the use of ironorm, like stomach pain, and feeling nauseous. I don't know whether to continue with ironorm or to change the treatment

Sent by ETLEVA, më 09 September 2020 në 07:19

Hello Etleva, the medicinal dose of Ironorm is 3 x 1 capsules/day, for at least 3 consecutive months. Currently, continue for 1 more month with 3 x 1 capsules per day and after 1 month of treatment, you will repeat: complete blood count, ferritinemia, hemoglobin electrophoresis

Replay from Dr. Shk. Sotiraq Lako, më 09 September 2020 në 11:12

Hello doctor. I am 21 weeks pregnant. The latest blood test results are: wbc 8.63, hgb 10.4, mch 30.7 mchc 39.3, rbc 3.37, mcv 78.1, mpv 6.2. Ferritin was 10.7. The gynecologist has given me Globifer Forte for 2 consecutive months and it has not had an effect. Now, after these test results and the ferritin, which was the first time I tested it, he prescribed Ferra Max 150 twice a day. Should I continue with Ferra Max, what should I do?

Sent by Sibora , më 15 September 2020 në 04:47

Hello Sibora, you have mild anemia, probably from iron deficiency. None of the preparations you are using are effective. Ironorm 3 x 1 capsule/day, after 1 month recheck with complete blood and after 3 months of treatment and ferritinemia

Replay from Dr. Shk. Sotiraq Lako, më 15 September 2020 në 07:59

Hello doctor! I am a 25-year-old girl, and due to some problems and concerns I had with my stomach, I visited a gastroenterologist and was diagnosed with mild gastritis. I also experienced fatigue, weakness, and rapid heartbeats, and had blood tests done. My hemoglobin was 12.3, while my Ferritin was 8. During this time, I did not take any iron supplements, since I was taking another medication for gastritis. Now that I have finished the treatment, I have started to use Tardyferon 80mg accompanied by vitamin C 100mg. Is this the right treatment considering the problems I have with my stomach?

Sent by Brisi, më 07 October 2020 në 08:18

Hello Brisi, you have an iron deficiency. Take Tardyferon 80 mg 3 x 1 tablets/day + Vitamin C 100 mg 3 x 1 tablets/day, for 2 consecutive months. After 2 months, have a complete blood recheck, ferritinemia (minimum 20 ng/ml and the optimal value for females is around 60 ng/ml)

Replay from Dr. Shk. Sotiraq Lako, më 08 October 2020 në 07:23

Hello doctor! I am a 25-year-old girl, and due to some problems and concerns I had with my stomach, I visited a gastroenterologist and was diagnosed with mild gastritis. I also experienced fatigue, weakness, and rapid heartbeats, and I had blood tests done. My hemoglobin was 12.3, while my Ferritin was 8. During this time, I did not take any iron medication because I was taking another medication for gastritis. Now that I have finished the course, I have started to use Tardyferon 80mg accompanied by vitamin C 100mg. Is this the right treatment considering the problems I have with my stomach?

Sent by Brisi, më 07 October 2020 në 08:19

Hello Brisi, you have an iron deficiency. Tardyferon 80 mg 3 x 1 tablets/day + Vitamin C 100 mg 3 x 1 tablets/day, for 2 consecutive months. After 2 months, a complete blood recheck, ferritinemia (minimum 20 ng/ml and the optimal value for females around 60 ng/ml)

Replay from Dr. Shk. Sotiraq Lako, më 08 October 2020 në 07:23

Hello, doctor! I am a 25-year-old girl, and due to some problems and concerns I had with my stomach, I visited a gastroenterologist and was diagnosed with mild gastritis. I also had fatigue, weakness, and rapid heartbeats, and I did blood tests. Hemoglobin was 12.3, while Ferritin was 8. During this time, I did not take any iron medication, since I was taking another medication for gastritis. Now that I have finished the course, I have started to use Tardyferon 80mg accompanied by vitamin C 100mg. Is this the right treatment considering the problems I have with my stomach?

Sent by Brisi, më 07 October 2020 në 08:19

Hello Brisi, you have an iron deficiency. Tardyferon 80 mg 3 x 1 tablets/day + Vitamin C 100 mg 3 x 1 tablets/day, for 2 months straight. After 2 months, a complete blood recheck, ferritinemia (minimum 20 ng/ml and the optimal value for females around 60 ng/ml)

Replay from Dr. Shk. Sotiraq Lako, më 08 October 2020 në 07:23

Hello, doctor! I am 31 years old. I have done the tests and the results are: RBC-5.37 / HGB-9.6 / HCT-30.6 / MCV-56.8 / MCH-17.8 hemoglobin electrophoresis HbA1 98.1 HbA2 1.9. please doctor, what type of anemia do I have, and can it go away with food. I have been recommended Ferro 3 Forte. Regards

Sent by Besjana, më 19 October 2020 në 14:25

Hello Besjana, Mild Anemia, probably due to Iron Deficiency. You will undergo ferritinemia testing and will be treated. Ferro3 is not the appropriate preparation.
1-Ironorm 3 x 1 capsule/day – 90 capsules/month.

Either during or after meals, for at least 3 consecutive months. The patient will be rechecked with a complete blood count after 1 month and after 3 months of treatment with a complete blood count + ferritinemia

Replay from Dr. Shk. Sotiraq Lako, më 20 October 2020 në 05:19

Hello doctor, I have a 12-year-old son and from a routine check-up, we did blood tests for him and the results were erythrocytes 4.84; hemoglobin 13; and hematocrit 39.5, however, iron 36.2 and ferritin 8.31. The doctor advised us to give him Ferro 3 Forte (those small bottles) for a month and see how it goes, because she said he doesn't have anemia but his iron and iron stores are very low. Meanwhile, during the summer, my son also complained about leg pains, as if they were shaking after he finished playing, and his hair falls out easily too. Do you think Ferro 3 Forte is good for my son or should we try something else? Thank you

Sent by Ornela, më 06 November 2020 në 10:58

Hello Ornela, the boy has a reduction in iron reserves. Ferro3 is not a preparation that replenishes iron stores. You can use it for 1.5 months, Ironorm 2 x 1 capsules/day, if it is possible for him to take the capsules orally

Replay from Dr. Shk. Sotiraq Lako, më 07 November 2020 në 10:18

Hello doctor, I have a 12-year-old son and from a routine check-up, we did blood tests for my son and the results were erythrocytes 4.84; hemoglobin 13; and hematocrit 39.5, but iron 36.2 and ferritin 8.31. The doctor advised us to give him Ferro 3 Forte, (those small bottles) for a month and to see how it goes, because she said he doesn't have anemia but very low iron and iron stores. Meanwhile, during the summer, my son also complained about leg pains, as if they were trembling after he finished playing and his hair falls out easily. Do you think Ferro 3 Forte is good for my son or is something else needed? Thank you

Sent by Ornela, më 06 November 2020 në 11:07

Hello Ornela, the boy has a reduction in iron reserves. Ferro3 is not a preparation that replenishes iron stores. You can use it for 1.5 months, Ironorm 2 x 1 capsule/day, if he is able to take the capsules orally

Replay from Dr. Shk. Sotiraq Lako, më 07 November 2020 në 10:18

Hello doctor, my son is 12 years old and from a routine check-up, we did his blood tests and it showed erythrocytes 4.84; hemoglobin 13; and hematocrit 39.5, but iron 36.2 and ferritin 8.31. The doctor advised us to give him Ferro 3 Forte, (those small bottles) for a month and see how it goes, because she said he doesn't have anemia but very low iron and iron stores. Meanwhile, during the summer, my son also complained about leg pains, as if they were shaking after he finished playing and his hair falls out easily too. Do you think Ferro 3 Forte is good for my son or is something else needed? Thank you

Sent by Ornela, më 06 November 2020 në 11:08

Hello Ornela, the boy has a reduction in iron reserves. Ferro3 is not a preparation that replenishes iron stores. It can be used for 1.5 months, Ironorm 2 x 1 capsule/day, if it's possible for him to take the capsules orally

Replay from Dr. Shk. Sotiraq Lako, më 07 November 2020 në 10:18

Hello doctor. I have a 14-year-old daughter. She has been suffering from dizziness for 6 weeks. From the tests done, her ferritin was 5.25, while her hemoglobin was 12.2. Can the low level of iron cause dizziness? Which medication do you recommend to correct the low level of ferritin?

Sent by Bruna, më 07 November 2020 në 07:48

Hello Bruna, the girl has an iron deficiency and it is one of the reasons for the complaint. She will take iron supplements, a medicinal dose of 180-200 mg of elemental iron per day, for 1.5-2 months in a row. If it is related to the iron deficiency, the iron deficiency is always corrected by treatment, for this reason, the girl's complaint should be eliminated. If the iron stores are replenished and the complaint continues, you will discuss with the neurologist, the ENT doctor

Replay from Dr. Shk. Sotiraq Lako, më 07 November 2020 në 10:23

Hello doctor. I have a 14-year-old daughter. She has been suffering from dizziness for 6 weeks. From the tests conducted, her ferritin level was found to be 5.25, while her hemoglobin was 12.2. Can the low level of iron cause dizziness? Which medication do you recommend to correct the low ferritin level?

Sent by Bruna, më 07 November 2020 në 07:49

Hello Bruna, the girl has an iron deficiency and it is one of the reasons for her complaint. She will take iron supplements, a medicinal dose of 180-200 mg of elemental iron per day, for 1.5-2 consecutive months. If it is related to the iron deficiency, the lack of iron is always corrected by the medication, for this reason, the girl's complaint should be eliminated. If the iron stores are replenished and the complaint continues, you will discuss with the neurologist, ENT doctor

Replay from Dr. Shk. Sotiraq Lako, më 07 November 2020 në 10:23

Hello doctor! I am a 25-year-old girl, 43 kg! A month ago, I had blood tests and ferritin checked. Hemoglobin was 12.3 while Ferritin was 8. During this time, I have been taking Tardyferon 80 mg once a day accompanied by 10mg of vitamin C. My family doctor has told me to continue this medication for 2 months in a row. I wanted to ask if this is the right medication? (I continue to feel very tired, hair loss, and arrhythmia) Thank you

Sent by Brisi, më 11 November 2020 në 08:04

Hello Brisi, the preparation is good, take it for your weight twice a day, for 2 months in a row

Replay from Dr. Shk. Sotiraq Lako, më 13 November 2020 në 07:29

Hello doctor! I am a 31-year-old girl, suffering from ulcerative colitis. Today I did the tests and the results are: WBC 3.36, RBC 4.11, HGB 11.9, FERRITIN 3.38 AND SERUM IRON 29. I feel tired, powerless, and constantly drowsy. I wanted your opinion on what you suggest for the correction of iron

Sent by Ana, më 16 November 2020 në 12:43

Hello Ana, you have mild anemia due to iron deficiency. As a rule, we prefer treatment with oral iron supplements, but in a patient with ulcerative colitis, we try to avoid it and apply treatment with injections. In Albania, there are 2 options: Ferrovin 100 mg 5-6 infusions or Ferrinject 500 mg a single infusion. Treatments with injections are only done in hospital conditions, because iron preparations, rarely, cause allergic reactions

Replay from Dr. Shk. Sotiraq Lako, më 17 November 2020 në 07:13

Hello doctor and thank you for your answers! I am 25 years old and since childhood, I have shown problems with iron deficiency! I have been treated and the moment I stop iron, my ferritin levels drop again, going down to 6. What would you suggest I use, I have used ironorm and most recently tadyferon, but not for a very long time, my negligence... thank you!

Sent by klaudia, më 17 November 2020 në 10:58

Hello Klaudia, in Albania, the best preparation for replenishing iron stores is Heferol. 1-Heferol 350 mg 2 x 1 capsule/day – 60 capsules/month. 2-Vitamin C 100 mg 2 x 1 tablet/day – 60 tablets/month. During or after meals, for at least 2 consecutive months. Ferritinemia (the minimum normal value for adult females for ferritinemia is 20 ng/ml and the optimal value is around 60 ng/ml)

Replay from Dr. Shk. Sotiraq Lako, më 19 November 2020 në 03:21

Hello Doctor, I have hair loss, ferritin 5.2, what do you advise?

Sent by Edisona, më 12 December 2020 në 17:10

Hello Edisona, one of the reasons for hair loss is also iron deficiency. You will be treated with iron supplements until the iron deficiency is corrected, with a minimum ferritin level of 20 ng/ml and an optimal level around 60 ng/ml. If these levels are achieved and hair continues to fall, it will be discussed with a dermatologist, it is no longer iron deficiency causing it

Replay from Dr. Shk. Sotiraq Lako, më 13 December 2020 në 08:35

Hello doctor! I have had problems with anemia since I was a child. Today, I did the tests and the results are: HGB 8.8, HCT 29.5, MCV 71.8, MCH 21.4, MCHC 29.8, RDW 16.8, PLT 331, MPV 8.5, PCT 0.281, PDW 14.0. I feel powerless and constantly tired. What would you suggest I use? Thank you

Sent by Matilda , më 28 December 2020 në 11:34

Hello Matilda, the most common type of anemia in medical practice is Iron Deficiency Anemia. If assessed and treated properly, it is fully correctable. Generally, by doctors and patients, it is undervalued and mistreated, so it can never be eliminated. You will start with Ironorm 3 X 1 capsules/day, during or after meals. After 1 month, a follow-up with complete blood count and ferritinemia. The treatment lasts at least 3 consecutive months. The cause(s) are also important, and in general, in females, the menstrual cycle plays a key role

Replay from Dr. Shk. Sotiraq Lako, më 29 December 2020 në 03:32

Hello doctor. I had my blood tests done and my hemoglobin was 11.7 while ferritin was 9.7. I drank Ferro Forte, two bottles a day, but I had stomach pain and constipation. I stopped for a few days and then started again but with one a day. I am on day 6 and I see that the stomach pain is starting again. What should I do?

Sent by Marinela, më 09 January 2021 në 17:56

Hello Marinela, I do not prefer juices. 1-Hepherol 350 mg 2 x 1 capsule/day – 60 capsules/month.
2-Vitamin C 100 mg 2 x 1 tablet/day – 60 tablets/month, during or after a meal, for at least 2 consecutive months

Replay from Dr. Shk. Sotiraq Lako, më 12 January 2021 në 04:09

Hello doctor! I hope you are well! I wanted to ask you what I can use please because my ferritin is 8.6. All the other blood tests are within normal limits. Thank you!

Sent by Bardha , më 11 January 2021 në 08:06

Hello Bardha, for adults, 1-Hepherol 350 mg 2 x 1 capsule/day – 60 capsules/month. 2-Vitamin C 100 mg 2 x 1 tablet/day – 60 tablets/month, during or after a meal, for at least 2 consecutive months

Replay from Dr. Shk. Sotiraq Lako, më 12 January 2021 në 04:06

Hello again, doctor. Thank you very much for your answer. I wanted to ask if instead of Hiferol, I could take Ferro Sanol Duodenal. Secondly, how long apart should iron supplements be taken if I don't consume dinner to take it after a meal? Thirdly, I have taken vitamin C 500 mg to accompany the iron, but you tell me it should be twice from 100. Is there a problem that I'm taking such a high dose of vitamin C (I've been taking it once a day)? I wish you all the best and thank you for listening to us

Sent by Marinela, më 13 January 2021 në 05:10

Hello Marinela, Ferro Sanol Duodenal 2 x 1 capsule/day is a very good preparation, for 2 consecutive months. It contains Vitamin C itself and there is no need to take an extra supplement. You can set the schedule yourself, the important thing is to take the daily dose

Replay from Dr. Shk. Sotiraq Lako, më 13 January 2021 në 05:46

Hello doctor, I am 15 years old and I did my blood tests and got the results: Hgb 12.5, MCH 26.6, MCV 79.7, MCHC 333, WBC 5.69, HCT 37.6, serum iron 40, ferritin 107.8. I am experiencing hair loss, feel powerless meaning I don't have immunity, get tired quickly, and have joint pain especially these last two months. Thank you in advance for your response

Sent by Ornela, më 22 January 2021 në 14:03

HELLO ORNELA, THE CURRENT VALUES ARE NORMAL. There is no justification for your complaints. Is the ferritin level 107.8, or just 10.8 ng/ml?

Replay from Dr. Shk. Sotiraq Lako, më 23 January 2021 në 02:48

Hello doctor. I have been taking tardyferon 2x1 for a month for iron deficiency anemia due to hemorrhoids. Ferritin has reached 30 for now so it has increased. The question is what happens after the treatment because more or less I know that the iron balance will be negative again. How many times can the tardyferon treatment be repeated? How many weeks or months do the iron reserves last if there is an average daily bleeding of about 10 ml for example? An approximate calculation would help me know when to repeat the analysis. Thank you

Sent by Beni, më 24 January 2021 në 16:15

Hello Beni, your first task is to definitively resolve the bleeding. Any bleeding, especially if recurrent, except for the Menstrual Cycle in females, is a disease. You will go to a proctologist, with local treatment or surgery, laser, etc., this problem must be solved definitively. If you solve this problem, you will be successful in treating with iron supplements. Otherwise, you are only "beautifying" the problem, the Iron Deficiency, from time to time. When the treatment is palliative, as in your case until now, be content with the good values you have, if you want to correct the problem and even more so to heal, you must definitively resolve the bleeding

Replay from Dr. Shk. Sotiraq Lako, më 25 January 2021 në 04:07

Hello doctor. I did the tests and have these data HGB 10 FERRITIN 8
ERITROCYTES 3.87. HCT 30.1. I started tardyferon 80 mg 3 times a day. Is it the right treatment or not?

Sent by Bujare, më 05 February 2021 në 12:39

Hello Bujare, you have mild anemia due to iron deficiency.
1-Tardyferon 80 mg 3 x 1 tablets/day – 90 tablets/month.
2-Vitamin C 100 mg 3 x 1 tablets/day – 90 tablets/month.
3-Vitamin B6 25 mg 3 x 1 tablets/day – 90 tablets/month.

For at least 3 consecutive months. The patient will be rechecked with a complete blood count after 1 month of treatment and after 3 months of treatment a complete blood count + ferritin level (the minimum ferritin level in adult females is 20 ng/ml and the optimal value is about 60 ng/ml)

Replay from Dr. Shk. Sotiraq Lako, më 06 February 2021 në 04:56

Hello doctor, I am 46 years old and my ferritin came out to 2.2. I started treatment with intravenous Ferrovin once a day, vitamin C twice a day, and vitamin B2 intramuscularly... is this the right treatment for me?

Sent by Anxhela, më 06 February 2021 në 17:48

Hello Anxhela, it is one of the treatment options. When iron is taken intravenously, Vitamin C is not needed, which is only used when iron is taken orally, to increase stomach acidity (Ascorbic Acid). When iron is lacking, there is no need for Vitamin B12. If you also have a deficiency of it, of course, you will receive it, according to the respective protocol. Vitamin B2 is not needed

Replay from Dr. Shk. Sotiraq Lako, më 07 February 2021 në 05:54

Hello doctor, I am 46 years old and my ferritin level is 2.2. I started treatment with intravenous ferrovin once a day, vitamin C twice a day, and intramuscular vitamin B2. Is this the right treatment for me?

Sent by Anxhela, më 07 February 2021 në 03:02

Hello Anxhela, it is one of the treatment options. When iron is taken intravenously, Vitamin C is not needed, which is only used when iron is taken orally to increase stomach acidity (Ascorbic Acid). When iron is lacking, there is no need for Vitamin B12. If you also have a deficiency of it, of course, it will be taken according to the respective protocol. Vitamin B2 is not needed

Replay from Dr. Shk. Sotiraq Lako, më 07 February 2021 në 05:54

Thank you very much for the clarification, doctor. I wanted to ask about my relative who is 24 years old. They have a ferritin level of 9, but their hemoglobin is within normal limits. They have started a treatment with iron supplements twice a day and vitamin C twice a day. How long should they continue this treatment? It has been 16 days since they started this medication

Sent by Anxhela, më 07 February 2021 në 10:50

Hello Anxhela, your relative only has an iron deficiency, and Tothema is not the right preparation to correct it

Replay from Dr. Shk. Sotiraq Lako, më 08 February 2021 në 10:00

Hello doctor, I had my 4-year-old daughter tested and the results are:
WBC 11.63;
RBC 4.68;
Hgb 10.7 low;
HCT 32.8 low;
MCV 70.2 low;
MCH 22.8 low;

Ferritin 13.47;
Serum Iron 32;
Vitamin D 9.4;
I have used the iron supplement Feramax in powder form for about 1-2 months but these are the iron parameter values. Should we continue with Feramax or is there a more effective preparation we can use and at what dose please? Also, does the lack of vitamin D have any effect on iron absorption in the body?
Thank you for your time!

Sent by Marsida, më 13 February 2021 në 04:03

Hello Marsida, the girl has mild anemia. She may need to take iron. Feramax, at least in adults, does not ensure the replenishment of iron stores. Hemoglobin Electrophoresis will also be done

Replay from Dr. Shk. Sotiraq Lako, më 13 February 2021 në 09:57

Hello Doctor! You recommended heferolin for my ferritin. Please, I wanted to ask if there is any problem since I use levothyroxine and can low ferritin 8.6 affect blood pressure fluctuation? It goes to 14.5 15 with 9 or 10. THANK YOU!

Sent by Bardha , më 25 February 2021 në 08:04

Hello, you take levothyroxine on an empty stomach and ferrous sulfate in the middle or after food, usually 30 minutes after levothyroxine. So, they are not related. Sometimes, iron deficiency is accompanied by a decrease in blood pressure. But, iron deficiency is corrected by treatment, so its consequence should also be corrected

Replay from Dr. Shk. Sotiraq Lako, më 26 February 2021 në 09:41

Hello doctor! I am a 25-year-old girl! I did a blood test and my hemoglobin is 14.5, while my ferritin is 10. I consulted with a doctor in the country where I live, and he told me that I just need to eat well, so I shouldn't take any medication. Generally, by nature, I have low blood pressure 9 over 5. Does iron deficiency affect this? And please what do you suggest I do about the iron deficiency. Should I take medication? Thank you, Brisi

Sent by Brisilda, më 22 March 2021 në 08:11

Hello Brisi, you have an iron deficiency (ferritinemia < 20 ng/ml and the optimal value for females is around 60 ng/ml). It's impossible to correct iron reserves with food alone. If you do not address this with medication, your levels will remain low for life and you may also develop iron deficiency anemia = increase in complaints and a doubling of the treatment time. You can use Heferol 350 mg 2 x 1 capsule/day plus Vitamin C 100 mg 2 x 1 tablet/day, for at least 2 consecutive months. Recheck after 2 months with ferritinemia, with the above objectives in mind

Replay from Dr. Shk. Sotiraq Lako, më 22 March 2021 në 11:02

Hello doctor!
10 days ago I had my 2-year-old son's tests done and the results were as follows:
WBC - 8.89, RBC-4.92, HGB -12.5, HCT 35.2, MCV -71.5, MCH -25.4, Ferritin 17.3 ng/ml. My son had Covid 19, did not take any medication, then had a urinary infection, for which he received medication.
Currently, he is taking Vitamin D, Vitamin C, and some probiotics to boost his immunity.
The pediatrician prescribed Feramax (powder) 2 spoons/20 ml water, for about a month.
I welcome Your suggestions,
Thank you!

Sent by Aferdita, më 27 March 2021 në 08:55

Hello Aferdita, the boy does not have hematological changes, there is no need for treatment from the hematological side

Replay from Dr. Shk. Sotiraq Lako, më 27 March 2021 në 11:08

Hello Dr. Sotiraq!
I went through COVID-19 without any worrisome symptoms, I only realized I had it because of a slight shortness of breath that resulted in a positive identification, without symptoms, going through it for two weeks with oxygen levels between 94-96. At that time, the local doctor recommended that I undergo a D-Dimer test which was 0.6; PCR 185 ug/mL and my Ferritin came out at 25 ng/ml, Serum Iron 162.
Here I realized that my Ferritin was low, so I also did a complete blood count where my hemoglobin came out as 14.4 g/dl, Hematocrit was 40.9%, Platelets: 241 103/uL; Red Blood Cells 4.83 103/uL; Erythrocyte Sedimentation Rate 26 mm/h. From September 2020 until now in March, I avoided riding buses, but at the same time, during a 3 km walk, I would sweat, I don't know if this sweating has affected the decrease in my iron stores? I have felt in calm moments pulsations of the veins in my legs. As for food, I eat very well in this regard. Please based on these data, what treatment do you recommend, and how is it explained that the storage is 25, have the elements escaped through my sweat or is there something else?
Respect for everything you will do for me!

Sent by Dritani, më 31 March 2021 në 10:45

Hello Dritan, low ferritinemia is not related to Covid. You need to correct it with medication. Heferol is a very good preparation to replenish the stores. The goal is a minimum of 30 ng/ml and the optimal value for males is around 100 ng/ml

Replay from Dr. Shk. Sotiraq Lako, më 01 April 2021 në 04:38

Hello Dr. Shk Sotiraq!
I went through COVID-19 without worrying signs, I only realized this condition through a mild breathlessness which identified me as positive, without symptoms and I got through it for two weeks with oxygen levels between 94-96. Then, my local doctor recommended doing D-Dimer 0.6; PCR 185 ug/mL and my Ferritin turned out to be 25 ng/ml, and Iron 162.
Here I realized that my Ferritin was low, so I also did a complete blood test where my hemoglobin turned out to be 14.4 g/dl, Hematocrit was 40.9%, Platelets: 241 103/uL; Red Blood Cells 4.83 103/uL; Erythrocyte Sedimentation Rate 26 mm/h. From September 2020 until now in March, I avoided getting on buses, but at the same time, during a 3 km walk, I would sweat, I don't know if this sweating has affected the reduction of my iron reserves? I have felt quiet pulse sensations in my leg. As for food, I eat very well in this regard. Please, based on this data, what treatment would you recommend, and how is it explained that the reserve is 25, have I lost elements through my sweat or is there something else?
Respects for everything you will do for me!

Sent by Dritani, më 31 March 2021 në 10:55

Hello Dritan, low ferritinemia is not related to Covid. It needs to be corrected with medication. Heferol is a very good preparation to replenish the stores. The goal is a minimum of 30 ng/ml and the optimal value for males is about 100 ng/ml

Replay from Dr. Shk. Sotiraq Lako, më 01 April 2021 në 04:38

Hello Dr. Sotiraq,

I went through COVID-19 without alarming symptoms; I realized I had the virus only because of a mild breathlessness, which led to my positive diagnosis, without symptoms, and I got through it over two weeks with oxygen levels between 94-96. Then, my local doctor recommended that I undergo tests for D-Dimer 0.6; PCR 185 ug/mL and my Ferritin turned out to be 25 ng/ml, Serum Iron 162.
Here, I realized that my Ferritin was low, so I had a complete blood test where my Hemoglobin was 14.4 g/dl, Hematocrit was 40.9%, Platelets: 241 103/uL; Red Blood Cells 4.83 103/uL; Erythrocyte Sedimentation Rate 26 mm/h. From September 2020 to now in March, I avoided getting on buses, but at the same time, while walking 3 km, I would sweat, I don't know if this sweating has affected the reduction of my storage? I have felt in peace the pulsation of the vein in my leg. As for food, I eat very well in this regard. Please, based on this data, what treatment do you recommend, and how is it explained that the storage is 25, have the elements escaped through my sweat or is there something else?
Respect for everything you will do for me!

Sent by Dritani, më 31 March 2021 në 11:05

Hello Dritani, low ferritinemia is not related to Covid. It needs to be corrected with medication. Heferol is a very good preparation for replenishing the stores. The goal is a minimum of 30 ng/ml and the optimal value for males is about 100 ng/ml

Replay from Dr. Shk. Sotiraq Lako, më 01 April 2021 në 04:38

Hello Doctor! I have done a ferritin test and it came out 24.25 ng/ml. According to the laboratory reference values, it was within the norms, but in fact, I feel a bit tired and have had problems with anemia in the past years. I would very much appreciate your opinion and suggestion. Thank you

Sent by Austela Fejzaj, më 02 April 2021 në 03:32

Hello Austela, the value you have is within the normal range (the lower limit of normal values for females is 20 ng/ml), the optimal value is around 60 ng/ml. Therefore, the goal is to have higher values, so for 1.5 months you can use medication. The preparation we most often use: Heferoli

Replay from Dr. Shk. Sotiraq Lako, më 02 April 2021 në 06:53

Hello doctor! I am 33 years old and my ferritin test came out 11.8 ng/ml. How can I treat it? Thank you and I wish you all the best

Sent by Denisa, më 06 April 2021 në 10:17

Hello Denisa, you have an iron deficiency (the normal minimum value of ferritinemia for adult females is 20 ng/ml and the optimal value is 60-70 ng/ml). The most commonly used preparation in Albania is Heferoli 350 mg, accompanied by Vitamin C 100 mg, twice a day, for 2 consecutive months

Replay from Dr. Shk. Sotiraq Lako, më 07 April 2021 në 08:49

Hello doctor! I am 33 years old and my ferritin test result came out as 11.8 ng/ml. How can I treat it? Thank you and I wish you all the best

Sent by Denisa, më 06 April 2021 në 10:30

Hello Denisa, you have an iron deficiency (the minimum normal value of ferritinemia for adult females is 20 ng/ml and the optimal value is 60-70 ng/ml). The most commonly used preparation in Albania is Heferol 350 mg, accompanied by Vitamin C 100 mg, twice a day, for 2 months in a row

Replay from Dr. Shk. Sotiraq Lako, më 07 April 2021 në 08:49

Hello doctor, I have done a complete blood test, neutrophils 46.1, lymphocytes 47.1, RBC 5.04, MCV 79.6, MCH 27.2, HGB 13.7. Ferritin 10.23. What do you suggest? Best regards

Sent by Matilda, më 06 April 2021 në 10:45

Hello Matilda, you have an iron deficiency (the normal minimum ferritin value for adult females is 20 ng/ml and the optimal value is 60-70 ng/ml). The most commonly used preparation in Albania is Heferol 350 mg, accompanied by Vitamin C 100 mg, twice a day, for 2 consecutive months

Replay from Dr. Shk. Sotiraq Lako, më 07 April 2021 në 08:50

Hello doctor, I have done a complete blood analysis, neutrophils 46.1, lymphocytes 47.1, RBC 5.04, MCV 79.6, MCH 27.2, HGB 13.7. Ferritin 10.23. What do you recommend?

Sent by Matilda, më 06 April 2021 në 10:51

Hello Matilda, you have an iron deficiency (the minimum normal value of ferritinemia for adult females is 20 ng/ml and the optimal value is 60-70 ng/ml). The most commonly used preparation in Albania is Heferol 350 mg, accompanied by Vitamin C 100 mg, twice a day, for 2 consecutive months

Replay from Dr. Shk. Sotiraq Lako, më 07 April 2021 në 08:50

Hello doctor! I am 33 years old and my ferritin analysis came out at 11.8 ng/ml. How can I treat it? Thank you and I wish you all the best

Sent by Denisa, më 06 April 2021 në 15:13

Hello Denisa, you have an iron deficiency (the minimum normal value of ferritinemia for adult females is 20 ng/ml and the optimal value is 60-70 ng/ml). The most commonly used preparation in Albania is Heferoli 350 mg, accompanied by Vitamin C 100 mg, twice a day, for 2 months in a row

Replay from Dr. Shk. Sotiraq Lako, më 07 April 2021 në 08:49

Hello doctor! I am 33 years old and my ferritin test came out as 11.8 ng/ml. How can I treat it? I thank you and wish you all the best

Sent by Denisa, më 07 April 2021 në 01:24

Hello Denisa, you have an iron deficiency (the minimum normal value of ferritinemia for adult females is 20 ng/ml and the optimal value is 60-70 ng/ml). The most commonly used preparation in Albania is Heferol 350 mg, accompanied by Vitamin C 100 mg, twice a day, for 2 consecutive months

Replay from Dr. Shk. Sotiraq Lako, më 07 April 2021 në 08:50

Hello doctor! I am 33 years old and my ferritin test came out to be 11.8 ng/ml. How can I treat it? Thank you and I wish you all the best

Sent by Denisa, më 07 April 2021 në 03:17

Hello Denisa, you have an iron deficiency (the minimum normal ferritin value for adult females is 20 ng/ml and the optimal value is 60-70 ng/ml). The most commonly used preparation in Albania is Heferoli 350 mg, accompanied by Vitamin C 100 mg, twice a day, for 2 months straight

Replay from Dr. Shk. Sotiraq Lako, më 07 April 2021 në 08:49

Hello doctor!
I am 46 years old and have very low iron stores, my Ferritin is 4.5 and hemoglobin is 9.8... I have taken ironorm, oviron and I stopped them because from the second day I have had swelling and numbness in my eyes, hands, and feet... these days I started heferol and after 7-8 days the same thing is happening to me.
Please, can you suggest what I should do??
Thank you!

Sent by Denis, më 08 April 2021 në 14:59

Hello Deni, you have iron deficiency and if you don't take the proper medication, it won't be corrected. The preparations that have been recommended to you are the right ones. Usually, iron supplements when taken orally cause stomach and intestinal disorders. If you experience swelling, numbness you should first discuss with an allergist. If you are not allergic to iron supplements, the administration of iron through an IV can also be considered

Replay from Dr. Shk. Sotiraq Lako, më 09 April 2021 në 03:36

Hello doctor! I have done a complete blood test for my 4-year-old son, and it shows sideremia 30, ferritin 32, HGB 11, RBC 4.6, HCT 37.2, PLT 480. What treatment can I take to correct the anemia? Please, I need an answer from you

Sent by Alba, më 13 April 2021 në 13:08

Hello Alba, currently the values are good. There is no need for medication

Replay from Dr. Shk. Sotiraq Lako, më 14 April 2021 në 04:28

Hello doctor! I am 31 years old, 7 months pregnant, and I did my tests 1 month after taking Tothema 2x1. Hemoglobin 10.4 did not change after treatment, while Ferritin decreased from 25 to 10. What do you advise me to take, please? Wbc:11.51 RBC:3.36 HCT:30.2 Platelets 144. Tardyferron causes me diarrhea. Now I have been given Ferro 3 1x1

Sent by Eva, më 14 April 2021 në 06:07

Hello Eva. Tothema is not the right preparation for the correction of anemia and iron deficiency. 1-Heferol 350 mg 2 x 1 capsule/day – 60 capsules/month. 2-Vitamin C 100 mg 2 x 1 tablet/day – 60 tablets/month. 3-Vitamin B6 25 mg 2 x 1 tablet/day – 60 tablets/month. 4-Folic Acid 5 mg 2 x 1 tablet/day – 60 tablets/month. During or after meals, for at least 3 months in a row. The patient will be rechecked with a complete blood count after 1 month and after 3 months of treatment complete blood count + ferritin level + Hemoglobin Electrophoresis (the minimum normal value of ferritin in adult females is 20 ng/ml and the optimal value is about 60 ng/ml)

Replay from Dr. Shk. Sotiraq Lako, më 15 April 2021 në 02:30

Hello doctor! I am 31 years old, 7 months pregnant, and I did the tests 1 month after taking Tothema 2x1. Hemoglobin 10,4 did not change after the treatment, while Ferritin decreased from 25 to 10. What do you advise me to take, please? Wbc: 11.51 RBC: 3.36 HCT: 30.2 Platelets 144. Tardyferron causes me diarrhea. Now I have been given Ferro 3 1x1

Sent by Eva, më 14 April 2021 në 06:25

Hello Eva. Tothema is not an appropriate preparation for the correction of anemia and iron deficiency. 1-Heferol 350 mg 2 x 1 capsule/day – 60 capsules/month. 2-Vitamin C 100 mg 2 x 1 tablet/day – 60 tablets/month. 3-Vitamin B6 25 mg 2 x 1 tablet/day – 60 tablets/month. 4-Folic Acid 5 mg 2 x 1 tablet/day – 60 tablets/month. Take during or after meals, for at least 3 consecutive months. The patient will have a complete blood recheck after 1 month and after 3 months of treatment, a complete blood count + ferritinemia + Hemoglobin Electrophoresis (the minimal normal value of ferritinemia in adult females is 20 ng/ml and the optimal value is around 60 ng/ml)

Replay from Dr. Shk. Sotiraq Lako, më 15 April 2021 në 02:30

Hello doctor! I am 31 years old, 7 months pregnant, and I did blood tests 1 month after taking Tothema 2x1. My hemoglobin is 10.4, which did not change after treatment, while Ferritin dropped from 25 to 10. What do you recommend I take, please? Wbc: 11.51 RBC: 3.36 HCT: 30.2 Platelets: 144. Tardyferron causes me diarrhea. Now I have been given Ferro 3 1x1

Sent by Eva, më 14 April 2021 në 06:26

Hello Eva. Tothema is not a suitable preparation for correcting anemia and iron deficiency. 1-Heferol 350 mg 2 x 1 capsule/day – 60 capsules/month. 2-Vitamin C 100 mg 2 x 1 tablet/day – 60 tablets/month. 3-Vitamin B6 25 mg 2 x 1 tablet/day – 60 tablets/month. 4-Folic Acid 5 mg 2 x 1 tablet/day – 60 tablets/month.

During or after meals, for at least 3 consecutive months. The patient will be rechecked with a complete blood count after 1 month and after 3 months of treatment complete blood count + ferritinemia + Hemoglobin Electrophoresis (the minimum normal value of ferritinemia in adult females is 20 ng/ml and the optimal value is about 60 ng/ml)

Replay from Dr. Shk. Sotiraq Lako, më 15 April 2021 në 02:30

Hello doctor!
My mom is 48 years old, and a month ago she had her ferritin levels tested, and the value was 11.38. She also had her vitamin D levels tested, where the value was 17.36. She was given medication for 1 month, specifically Feramax, once a day for ferritin. For correcting vitamin D levels, Rokivit, once a week for 2 months straight. After repeating the tests, the ferritin is at a value of 12.48 (so not a significant increase compared to the first time), while vitamin D - 24.21. Please, what do you suggest I do? Should the treatment be continued?

Sent by Brisilda , më 22 April 2021 në 03:50

Hello Brisilda, FeraMax is not a supplement that replenishes iron stores. 1-Heferol 350 mg 2 x 1 capsule/day – 60 capsules/month. 2-Vitamin C 100 mg 2 x 1 tablet/day – 60 tablets/month.

During or after meals, for at least 2 consecutive months

Replay from Dr. Shk. Sotiraq Lako, më 22 April 2021 në 04:22

Hello doctor! I am 24 years old and a week ago I did some tests where my results were RBC 5.08, WBC 5.89, HGB 10.3, Ferritin 14.56. I also did hemoglobin electrophoresis HbA 95.66% and HbA2 4.34%. Please, what type of anemia is this and if possible, could you recommend any treatment medication? Thank you

Sent by Eranda, më 11 Agust 2021 në 17:56

Hello Eranda, you have Thalassemia. You also have an iron deficiency. You will be treated for 2 months with Ironorm 3 x 1 capsules/day

Replay from Dr. Shk. Sotiraq Lako, më 15 Agust 2021 në 04:59

Hello doctor. When I was 15 years old, I experienced hair loss in one area and after undergoing tests, the doctor told me it was due to iron deficiency. I was treated for 1 month with medication, but afterwards, I did not continue with the treatment. Now I am 18 years old and I feel some concerns such as: occasional headaches and as if my muscles or bones are aching or pricking in every part of my body. In the ribs, behind the shoulders, in the legs, arms, etc. A few days ago, when I was doing some heavy physical activity, it felt like I was running out of oxygen, as if I was fainting. I will go again these days to perform the tests once more. Do my concerns relate to the untreated iron deficiency or could it be something else? Thank you

Sent by ari, më 01 September 2021 në 06:11

Hello Ari, one of the reasons for hair loss is the lack of iron. There are other reasons as well, which should be discussed with a dermatologist. To see if you have an iron deficiency, ferritin levels are tested. It is considered low when it is < 20 ng/ml in females and 30-40 ng/ml in males

Replay from Dr. Shk. Sotiraq Lako, më 01 September 2021 në 06:48

Hello Doctor, I have written to you above, but does Ferotec work as a medicine for the anemia that I have?

Sent by Eranda, më 02 September 2021 në 06:55

Hello Eranda, no

Replay from Dr. Shk. Sotiraq Lako, më 03 September 2021 në 07:26

Hello Doctor, I have written to you above, but does Ferotec work as a medicine for the anemia that I have?

Sent by Eranda, më 02 September 2021 në 15:45

Hello Eranda, no

Replay from Dr. Shk. Sotiraq Lako, më 03 September 2021 në 07:26

Hello Doctor! I have a 4-year-old daughter. Her initial tests were Ferritin 9.21ng/ml, HB 10.2, with low HCT, MCV, MCH. We used Ferrodue 3ml/day for 3 months. After 3 months, her iron level increased very little, from 9.21 to 9.93. At this moment, I'm starting to suspect inherited anemia...what do you recommend, please?

Sent by Jurild, më 03 September 2021 në 12:04

Hello Jurild, inherited anemia is not related to the failure to correct ferritin levels. The preparation and dosage of iron you have used need to be re-evaluated. If you have taken this medication and your hemoglobin has increased, but not your ferritin levels, the former indicates that it is not congenital anemia (which does not change with treatment), the latter indicates that treatment needs to continue, or be replaced to achieve the correction of ferritin levels

Replay from Dr. Shk. Sotiraq Lako, më 04 September 2021 në 11:52

Hello Doctor! I am 29 years old and 10 days ago I did some tests where my results were WBC 7.0, RBC 5.79, HGB 13.2, HCT 45.1, Average MCV 29.3, Ferritin 14.96, Iron 56.63. For the past 10 days, I have been using Emacrit 1 capsule a day. But I don’t see any improvement, I have continuous fatigue, headaches, and dizziness. What do you suggest? Thank you in advance!

Sent by Gladiola, më 08 September 2021 në 10:49

Hello Gladiola, for 1 month you will receive: Ironorm 3 x 1 capsules/day – 90 capsules/month. After 1 month, recheck with complete blood + ferritin + Hemoglobin Electrophoresis (the normal minimum value of ferritin in adult females is 20 ng/ml and the optimal value is around 60 ng/ml)

Replay from Dr. Shk. Sotiraq Lako, më 10 September 2021 në 07:17

Hello doctor,
I am Vjola, 41 years old. I have done my blood tests and these are the values that came out:
WBC=3.65
RBC=4.44
HGB=11.20
HCT=34.50
MCV=76.3
MCH=25.10
LYM=51.70 and Ferritin=3.84.
Please, can you guide me on what to do?

Sent by Vjola, më 23 September 2021 në 13:30

Hello Vjola, you have Mild Anemia due to Iron Deficiency. For 3 months, you need to be treated with an Iron supplement, aiming to correct the anemia after 1 month of treatment and after 3 months of treatment, and the ferritinemia

Replay from Dr. Shk. Sotiraq Lako, më 24 September 2021 në 03:59

Hello doctor. I have a 2-month-old son and I did his blood tests. RBC= 2.96; HGB=8.2; HCT=23.5; ferritin= 508.1. My son also had Covid when he was 3 weeks old and has G6PD deficiency. I am very worried, can you guide me on how I should act and what I should monitor? Thank you very much!

Sent by Lorena, më 13 October 2021 në 14:11

Hello Lorena, for the child's age, you will discuss with the pediatric hematologist, or at least with the pediatrician. I am a hematologist, for adult ages

Replay from Dr. Shk. Sotiraq Lako, më 14 October 2021 në 03:38

Dear doctor, I am 47 years old and from the tests I did, these were the results: HGM-8.7, HCT 29.4, and ferritin 4.6. I took iron medication (Ferro Grad) and I got folic acid and vitamin C in Italy. I have been taking them twice a day for two months, but I want to know... should there be a gap between taking supplements and other foods? And is there any food that should not be consumed at all while taking iron?..thank you

Sent by Gladiola, më 18 November 2021 në 14:53

Hello Gladiola, you have moderate-grade anemia. The treatment you are using is appropriate, taking it twice a day (Ferrograd 105 mg x 2 tablets/day, etc.), for at least 3 months in a row. As a rule, it is taken before meals, but to protect the stomach, we also give it during or after meals. There is no need to eliminate or add any food. Basically, take it whenever you want just "don't complain"

Replay from Dr. Shk. Sotiraq Lako, më 19 November 2021 në 15:57

Hello doctor...I did a complete blood analysis HGB 10.3 MID 0.16 MCH 23.1 MCV 66.9 RDWc 20.7 HCT 29.88 I am noting these values because the laboratory has marked them with a cross...Ferritin 3.21...Please prescribe me a medication

Sent by Denisa, më 09 December 2021 në 06:51

Hello Denisa, you have mild anemia due to iron deficiency and need to be treated with iron supplements for 3 consecutive months

Replay from Dr. Shk. Sotiraq Lako, më 10 December 2021 në 02:49

Hello doctor. I am 57 years old and I did blood tests after COVID and these are the results. Red Blood Cells (RBC) 5.12, Hematocrit (HCT) 34.4 Hemoglobin (HGB) 10.4 MCV 67.2, MCH 20.2 MCHC 30.2, RDW-SD 34.2 RDW-CV 14.6. Please could you tell me what I should do. Can you prescribe me a medication or should I do other tests?

Sent by Jolanda, më 15 December 2021 në 02:19

Hello Jolanda, you have Mild Anemia, probably due to iron deficiency. You will undergo ferritinemia and will be treated with iron supplements, for at least 3 consecutive months

Replay from Dr. Shk. Sotiraq Lako, më 15 December 2021 në 04:50

Hello doctor. I am 57 years old and I did my blood tests after COVID and these were the results. Red Blood Cells (RBC) 5.12, Hematocrit (HCT) 34.4 Hemoglobin (HGB) 10.4 MCV 67.2, MCH 20.2 MCHC 30.2, RDW-SD 34.2 RDW-CV 14.6. Please, can you tell me how I should proceed? Can you prescribe me a medication or do I need to do more tests?

Sent by Jolanda, më 15 December 2021 në 02:57

Hello Jolanda, you have Mild Anemia, probably due to iron deficiency. You will undergo a ferritinemia test and will be treated with iron supplements, for at least 3 consecutive months

Replay from Dr. Shk. Sotiraq Lako, më 15 December 2021 në 04:50

hello. According to your advice, I did an iron analysis and my Ferritin (Serum)* was 113.5 ng/mL. Should I take medication?

Sent by jolanda, më 15 December 2021 në 08:16

Hello Jolanda, the ferritinemia is normal

Replay from Dr. Shk. Sotiraq Lako, më 16 December 2021 në 06:27

There's no need for iron treatment

Replay from Dr. Shk. Sotiraq Lako, më 16 December 2021 në 06:28

Hello,
I have had the omicron version of covid19. I am a 29-year-old female and my current Ferritin is 37ng/ml. Since I have experienced dizziness, I thought to ask if its level has had an impact. Also, lymphocytes are 45.6. Just wanted to thank you in advance!

Sent by Ada, më 09 January 2022 në 08:40

Hello Ada, the values you currently have are considered normal. Repeat the ferritinemia after 1 month

Replay from Dr. Shk. Sotiraq Lako, më 10 January 2022 në 03:53

Hello, I cannot seem to see the answer to the concern I have expressed. Thank you

Sent by Oneda, më 31 January 2022 në 05:20

Hello Oneda, write your concern again

Replay from Dr. Shk. Sotiraq Lako, më 31 January 2022 në 14:10

Hello, I have a 3-year-old daughter. I have started following the instruction to give her iron drops, but her stool is dark black. Is this normal?

Sent by arlinda, më 31 January 2022 në 17:01

Hello Arlinda, it is one of the characteristics of oral iron therapy

Replay from Dr. Shk. Sotiraq Lako, më 01 February 2022 në 09:32

Hello,
I have conducted tests on my 11-year-old daughter and the values are as follows: vitamin D3 16.3, sideremia 52, Ferritin 9, Lymphocytes 50, MCV 76.5, MCH 25.6. The girl has headaches and most of the time is exhausted and just wants to sleep

Sent by Oneda, më 01 February 2022 në 03:20

Hello Oneda, your daughter only has a slight deficiency in iron and Vitamin D. You will discuss with the pediatrician about the appropriate treatment until their correction

Replay from Dr. Shk. Sotiraq Lako, më 01 February 2022 në 09:33

Hello doctor, my 5-year-old daughter has received these results from her tests: erythrocytes 4.63, hemoglobin 11.4, hematocrit 34.4, MCV 74.3, MCH 24.6, MCHC 33.1, RDW-SD 34.9, RDW-CV 17.3, TIBC 473.3 from [200-400], ferritin 15.46, folic acid 12.8 from [2.7-14], vitamin B12 560, iron 38.8 from [32-104], transferrin 339 from [200-350], transferrin saturation 8.07 from [12-45], hemoglobin electrophoresis HbA 63.39%, HbS 34.76%, HbA2 1.85%. Also, from my hemoglobin electrophoresis, I got HbA 60.02%, HbS 38.06, HbA2 1.92% and my other blood parameters are within normal ranges. What is the problem and what solutions could there be? Thank you

Sent by Marsida, më 09 April 2022 në 13:31

Hello Marsida, how are you and the girl are carriers of Sickle Cell Anemia, a congenital pathology of hemoglobin, it is not treated, it is inherited with probability

Replay from Dr. Shk. Sotiraq Lako, më 11 April 2022 në 06:17

Hello Doctor,
I have a 3-year-old daughter and her blood tests always show low ferritin levels: 12, 13. It went up to 20 with medication after passing COVID.
We stopped the medication Hemafer 50 mg*ml and her ferritin levels have dropped to 4.5. She also has a diagnosis of hypothyroidism and is treated with Euthyrox 25 mg once a day in the morning.
When she was 18 months old, we did tests and she was not a thalassemia carrier because my husband is a carrier, and her vitamin D levels were within the normal range.
Thank you

Sent by Lira, më 18 April 2022 në 01:08

Hello Lira, the decrease in iron has nothing to do with thyroid conditions and congenital anemia. If Hemafer does not work, another preparation will be evaluated. Check for parasites

Replay from Dr. Shk. Sotiraq Lako, më 18 April 2022 në 09:20

Hello! I did some tests after experiencing body aches, fatigue, low blood pressure, dizziness, and my hemoglobin came out to 12, while ferritin was 25. What do you recommend, doctor? Thyroid antiTPO 8.99, Tsh 1.9

Sent by Amarilda., më 25 April 2022 në 14:04

Hello Amarilda, the values of hemoglobin and ferritin are within the normal range. There is room to take iron supplements for 1-1.5 months, to increase ferritin to the optimal values of 60-70 ng/ml

Replay from Dr. Shk. Sotiraq Lako, më 26 April 2022 në 05:57

Sure, I got the ferritin results and the result is 4.62, should take tardyferon 80mg 2x1 and b complex and c500 2x1. should I do a complete blood test after 3 months of treatment?

Sent by Ina, më 26 April 2022 në 10:56

Hello Ina, you have an iron deficiency, Tardyferon 3 x 1 capsule/day + Vitamin C 100 mg 3 x 1 tablet/day, for at least 2 consecutive months

Replay from Dr. Shk. Sotiraq Lako, më 27 April 2022 në 03:23

Hello! I've been feeling dizzy for a few days, unable to concentrate on reading, trembling legs, body aches, fatigue, feeling like I'm going to vomit, headache, my left arm feels tingly as if it's falling asleep, and as if I have a blockage. My test results come out okay. What could it be? Where should I go?

Sent by Anduela, më 29 April 2022 në 13:02

Hello Anduela, you can do ferritin, vitamin B12, TSH, FT4, vitamin D, blood electrolytes tests. Consult with a neurologist

Replay from Dr. Shk. Sotiraq Lako, më 04 May 2022 në 02:54

Hello!
My 11-month-old child, from the tests done, had a hemoglobin level of 9.1, iron 58, RBC 3.69, HCT 27.5, MCV 74.5, MCH 24.7. Currently, they are being treated for post-infection thrombocytosis. Are these values concerning?

Sent by Ina, më 06 May 2022 në 11:08

Hello Ina, only a mild anemia is assessed in the values you have sent

Replay from Dr. Shk. Sotiraq Lako, më 07 May 2022 në 11:13

Should coffee be consumed during iron treatment? Thank you

Sent by Ina, më 09 May 2022 në 02:41

Hello Ina, if you take the right preparation, the right dose, there is no problem

Replay from Dr. Shk. Sotiraq Lako, më 09 May 2022 në 05:55

Hello Doctor,
I have a 69-year-old mother who has been treated for osteoporosis for years, after some tests these are the results. Are the white blood cell values very worrying because the red ones are all within the norm?
Besides Vitamin C and D, she has not used any other medication.

White blood cells and WBC morphology -
White Blood Cells (WBC)* 2.93,
Neutrophils NEU% 48.4,
Lymphocytes LYM% 40.3,
Monocytes MONO% 10.6,
Basophils BASO% 0.7,
Eosinophils EOS% 0.0,
Neutrophils NEU# 1.42,
Lymphocytes LYM# 1.18,
Monocytes MONO# 0.31,
Basophils BASO# 0.02,
Eosinophils EOS# 0.00.

Red cells and RBC line -
Red Blood Cells (RBC)* 4.38,
Hematocrit (HCT)* 39.5,
Hemoglobin (HGB)* 13.9,
MCV* 90.2,
MCH* 31.7,
MCHC* 35.2,

PLATELETS -
Platelets (PLT)* 248,

Erythrocyte Sedimentation Rate (ESR) 32,

25-OH Vitamin D 35.94,
Calcium 9.81,
Thank you

Sent by Joni, më 09 May 2022 në 16:21

Hello Joni, your mother has a slight decrease in white blood cells, Currently without medication. Recheck after 2 weeks with complete blood work

Replay from Dr. Shk. Sotiraq Lako, më 10 May 2022 në 09:08

Hello Doctor,
I have a 69-year-old mother who has been treated for osteoporosis for years, and after some tests, these are the results. Are the white blood cell values very concerning since the red ones are all within the norm?
Besides Vitamin C and D, she hasn't used any other medication.

White blood cells and WBC morphology -
White Blood Cells (WBC)* 2.93,
Neutrophils NEU% 48.4,
Lymphocytes LYM% 40.3,
Monocytes MONO% 10.6,
Basophils BASO% 0.7,
Eosinophils EOS% 0.0,
Neutrophils NEU# 1.42,
Lymphocytes LYM# 1.18,
Monocytes MONO# 0.31,
Basophils BASO# 0.02,
Eosinophils EOS# 0.00.

Red cells and RBC line -
Red Blood Cells (RBC)* 4.38,
Hematocrit (HCT)* 39.5,
Hemoglobin (HGB)* 13.9,
MCV* 90.2,
MCH* 31.7,
MCHC* 35.2,

PLATELETS -
Platelets (PLT)* 248,

Erythrocyte Sedimentation Rate (ESR) 32,

25-OH Vitamin D 35.94,
Calcium 9.81,
Thank you

Sent by Joni, më 09 May 2022 në 16:21

Hello Joni, your mother has a slight decrease in white blood cells, Currently without medication. Recheck after 2 weeks with complete blood count

Replay from Dr. Shk. Sotiraq Lako, më 10 May 2022 në 09:09

Hello Doctor,
I have a 69-year-old mother, who has been treated for osteoporosis for years. After some tests were conducted, these are the results. Are the white blood cell values very concerning since the red ones are all within the norm?
Besides Vitamin C and D, she has not taken any other medication.

White blood cells and WBC morphology -
White Blood Cells (WBC)* 2.93,
Neutrophils NEU% 48.4,
Lymphocytes LYM% 40.3,
Monocytes MONO% 10.6,
Basophils BASO% 0.7,
Eosinophils EOS% 0.0,
Neutrophils NEU# 1.42,
Lymphocytes LYM# 1.18,
Monocytes MONO# 0.31,
Basophils BASO# 0.02,
Eosinophils EOS# 0.00.

Red cells and RBC line -
Red Blood Cells (RBC)* 4.38,
Hematocrit (HCT)* 39.5,
Hemoglobin (HGB)* 13.9,
MCV* 90.2,
MCH* 31.7,
MCHC* 35.2,

PLATELETS -
Platelets (PLT)* 248,

Erythrocyte Sedimentation Rate (ESR) 32,

25-OH Vitamin D 35.94,
Calcium 9.81,
Thank you

Sent by Joni, më 09 May 2022 në 16:22

Hello Joni, your mother has a slight decrease in white blood cells, Currently without medication. Recheck after 2 weeks with complete blood work

Replay from Dr. Shk. Sotiraq Lako, më 10 May 2022 në 09:09

Hello Doctor,
My mother is 69 years old, has been treated for osteoporosis for years, after some tests these are the results. Are the white blood cell values very concerning because the reds are all within the norm?
Besides Vitamin C and D, she hasn't used any other medication.

White blood cells and WBC morphology -
White Blood Cells (WBC)* 2.93,
Neutrophils NEU% 48.4,
Lymphocytes LYM% 40.3,
Monocytes MONO% 10.6,
Basophils BASO% 0.7,
Eosinophils EOS% 0.0,
Neutrophils NEU# 1.42,
Lymphocytes LYM# 1.18,
Monocytes MONO# 0.31,
Basophils BASO# 0.02,
Eosinophils EOS# 0.00.

Red cells and RBC line -
Red Blood Cells (RBC)* 4.38,
Hematocrit (HCT)* 39.5,
Hemoglobin (HGB)* 13.9,
MCV* 90.2,
MCH* 31.7,
MCHC* 35.2,

PLATELETS -
Platelets (PLT)* 248,

Erythrocyte Sedimentation Rate (ESR) 32,

25-OH Vitamin D 35.94,
Calcium 9.81,
Thank you

Sent by Joni, më 09 May 2022 në 16:22

Hello Joni, your mother has a slight decrease in white blood cells, Currently without medication. Recheck after 2 weeks with complete blood work

Replay from Dr. Shk. Sotiraq Lako, më 10 May 2022 në 09:08

Hello Doctor,
I have a 69-year-old mother who has been treated for osteoporosis for years. After some tests, these are the results. Are the white blood cell values very concerning because the red ones are all within the norm?
Besides Vitamin C and D, she hasn't used any other medication.

White blood cells and WBC morphology -
White Blood Cells (WBC)* 2.93,
Neutrophils NEU% 48.4,
Lymphocytes LYM% 40.3,
Monocytes MONO% 10.6,
Basophils BASO% 0.7,
Eosinophils EOS% 0.0,
Neutrophils NEU# 1.42,
Lymphocytes LYM# 1.18,
Monocytes MONO# 0.31,
Basophils BASO# 0.02,
Eosinophils EOS# 0.00.

Red cells and RBC line -
Red Blood Cells (RBC)* 4.38,
Hematocrit (HCT)* 39.5,
Hemoglobin (HGB)* 13.9,
MCV* 90.2,
MCH* 31.7,
MCHC* 35.2,

PLATELETS -
Platelets (PLT)* 248,

Erythrocyte Sedimentation Rate (ESR) 32,

25-OH Vitamin D 35.94,
Calcium 9.81,
Thank you

Sent by Joni, më 09 May 2022 në 16:24

Hello Joni, your mother has a slight decrease in white blood cells. Currently without medication. Recheck after 2 weeks with complete blood work

Replay from Dr. Shk. Sotiraq Lako, më 10 May 2022 në 09:08

Hello, doctor! From the tests, I have received the following data: WBC 4.93, RBC 3.85, HGB 11.5, HCT 35.5, Gra% 76.04, Lym% 13.57%, Ferritin 14.83. Thank you!

Sent by Rudina, më 11 May 2022 në 02:54

Hello Rudina, you currently have a slight anemia from iron deficiency. It is advisable to be treated for 2 months in a row with iron preparations. For adults, Heferoli 350 mg is the preparation of choice

Replay from Dr. Shk. Sotiraq Lako, më 11 May 2022 në 05:04

Hello, doctor! Based on your advice, I started treatment with Heferol 5 days ago along with vitamin C, but I continue to feel very fatigued, and I don't feel tired even after a night's sleep. Do the values of the tests justify this condition? In early December and April, I took some antibiotics for a sinus infection... should I get tested for candida since I've read that an overgrowth of candida can cause constant fatigue. What other tests would you recommend I take? (I have also done TSH=2.28 and vitamin D = 18.78). Thank you in advance for your answer and the time you dedicate to us! All the best

Sent by Rudina, më 16 May 2022 në 01:23

Hello Rudina, oral treatment is not miraculous. It will take about 4-6 weeks to correct the anemia and about 12 weeks for the iron reserves

Replay from Dr. Shk. Sotiraq Lako, më 16 May 2022 në 03:34

I am on dialysis, I have anemia WBC 3.9, RBC 2.73, Hgb 8.2, Hct 24.3, and Plt 101.
Iron 25
While Ferritin 1600
Please doctor, what do you advise me?

Sent by Myrvete Zejnullahu, më 22 May 2022 në 16:43

Hello Myrvete, your anemia is assessed and followed by a nephrologist, where the use of Erythropoietin is most recommended

Replay from Dr. Shk. Sotiraq Lako, më 23 May 2022 në 05:57

Hello, dear doctor! I just wanted to ask for knowledge... Why are fibrinogen, CRP, LDH tests recommended by hematologists in cases of anemia? Thank you and good work!

Sent by Alma, më 23 May 2022 në 01:32

Hello Alma, the most common form of anemia is due to iron deficiency and in this case, these tests are usually not required. When it is due to other causes, e.g., inflammatory, CRP, Fibrinogen, LDH will also be requested. LDH also increases in hemolytic anemia, from the lack of Vitamin B12, etc

Replay from Dr. Shk. Sotiraq Lako, më 23 May 2022 në 05:56

Hello Doctor! I am 19 years old and I can't catch my breath, I feel tired, I have fast heartbeats, I have been suffering from this problem for 8 months, I did the tests and it turned out I have low iron, and as a result, they gave me oviron, two capsules a day, but this has not made any kind of difference. What should I do?

Sent by Tea, më 08 June 2022 në 14:40

Hello Tea, if you have had ferritinemia < 20 ng/ml, you are classified with an iron deficiency that can justify weakness and fatigue. It does not justify rapid heartbeats unless you have anemia and are not moving. You will do TSH and consult with the cardiologist. Oviron is a good preparation, 3 times a day

Replay from Dr. Shk. Sotiraq Lako, më 09 June 2022 në 06:30

Hello Doctor! I am 19 years old and I can't catch my breath, I feel tired, I have fast heartbeats, and I have been suffering from this problem for 8 months. I did the tests and it turned out I have low iron levels, and as a result, they prescribed me Oviron, two capsules a day, but this hasn't made any kind of difference. What should I do?

Sent by Tea, më 08 June 2022 në 17:36

Hello Tea, if you have had ferritinemia < 20 ng/ml, you are classified with an iron deficiency that can justify weakness and fatigue. It does not justify rapid heartbeat, if you do not have anemia and are not moving. You will do TSH and consult with the cardiologist. Oviron is a good preparation, 3 times a day

Replay from Dr. Shk. Sotiraq Lako, më 09 June 2022 në 06:30

Hello doctor,
First of all, thank you very much for
the straightforward answers.
My daughter is 4 years old and a week ago we did a complete blood count where these are the results
Red blood cells 5.26 (5.1 normal)
Hgb 123 (110-160 normal)
Hct 37.8
Mcv 71.7
Mch 25.1
Mchc 350
PLT 264
I am a bit concerned about the increased red blood cells. Does this have to do with an iron deficiency or not?
Thank you in advance :)

Sent by Ereza, më 15 June 2022 në 04:39

Hello Ereza, you will do ferritinemia and hemoglobin electrophoresis

Replay from Dr. Shk. Sotiraq Lako, më 16 June 2022 në 02:44

Hello doctor,
Firstly, thank you very much for the straightforward answers.
My daughter is 4 years old and a week ago we did a blood test where these are the results:
Red blood cells 5.26 (normal 5.1)
Hgb 123 (normal 110-160)
Hct 37.8
Mcv 71.7
Mch 25.1
Mchc 350
PLT 264
I am a bit concerned about the increased red blood cells. Does this have to do with an iron deficiency or not?
Thank you in advance:)

Sent by Ereza, më 15 June 2022 në 08:40

Hello Ereza, you will do ferritinemia and hemoglobin electrophoresis

Replay from Dr. Shk. Sotiraq Lako, më 16 June 2022 në 02:44

Hello doctor,
First of all, thank you very much for the straightforward answers.
My daughter is 4 years old and a week ago we did a complete blood count, here are the results:
Erythrocytes 5.26 (5.1 is the norm)
Hgb 123 (110-160 is the norm)
Hct 37.8
Mcv 71.7
Mch 25.1
Mchc 350
PLT 264
I am a little worried about the increased erythrocytes. Does this have to do with an iron deficiency or not?
Thank you in advance :)

Sent by Ereza, më 15 June 2022 në 18:51

Hello Ereza, you will do ferritinemia and hemoglobin electrophoresis

Replay from Dr. Shk. Sotiraq Lako, më 16 June 2022 në 02:44

Hello doctor,
First of all, thank you very much for
the straightforward answers.
My daughter is 4 years old and a week ago we did a complete blood count, here are the results
Erythrocytes 5.26 (5.1 normal)
Hgb 123 (110-160 normal)
Hct 37.8
Mcv 71.7
Mch 25.1
Mchc 350
PLT 264
I'm a bit concerned about the increased erythrocytes. Does this have to do with iron deficiency or not?
Thank you in advance :)

Sent by Ereza, më 15 June 2022 në 20:52

Hello Ereza, you will do ferritinemia and hemoglobin electrophoresis

Replay from Dr. Shk. Sotiraq Lako, më 16 June 2022 në 02:44

Hello doctor,
First of all, thank you very much for the straightforward answers.
My daughter is 4 years old and a week ago we did a complete blood count, here are the results:
Red blood cells 5.26 (5.1 is the normal range)
Hgb 123 (110-160 is the normal range)
Hct 37.8
Mcv 71.7
Mch 25.1
Mchc 350
PLT 264
I am a little worried about the increased red blood cells. Does this have to do with an iron deficiency or not?
Thank you in advance :)

Sent by Ereza, më 16 June 2022 në 00:27

Hello Ereza, you will do ferritinemia and hemoglobin electrophoresis

Replay from Dr. Shk. Sotiraq Lako, më 16 June 2022 në 02:44

Hello doctor,
Firstly, thank you very much for the straightforward answers.
My daughter is 4 years old and a week ago we did a blood test where these are the results:
Erythrocytes 5.26 (5.1 normal)
Hgb 123 (110-160 normal)
Hct 37.8
Mcv 71.7
Mch 25.1
Mchc 350
PLT 264
I am a bit concerned about the increased erythrocytes. Does this have to do with iron deficiency or not?
Thank you in advance :)

Sent by Ereza, më 16 June 2022 në 00:54

Hello Ereza, you will do ferritinemia and hemoglobin electrophoresis

Replay from Dr. Shk. Sotiraq Lako, më 16 June 2022 në 02:43

Hello doctor,
First of all, thank you very much for
the straightforward answers.
My daughter is 4 years old and a week ago we did a complete blood count and these are the results
Erythrocytes 5.26 (5.1 normal)
Hgb 123 (110-160 normal)
Hct 37.8
Mcv 71.7
Mch 25.1
Mchc 350
PLT 264
I am a bit concerned about the increased erythrocytes. Does this have to do with an iron deficiency or not?
Thank you in advance :)

Sent by Ereza, më 16 June 2022 në 00:55

Hello Ereza, you will do ferritinemia and hemoglobin electrophoresis

Replay from Dr. Shk. Sotiraq Lako, më 16 June 2022 në 02:43

Hello doctor,
First of all, thank you very much for the straightforward answers.
My daughter is 4 years old and a week ago we did a complete blood count, here are the results:
Red blood cells 5.26 (5.1 is the norm)
Hgb 123 (110-160 is the norm)
Hct 37.8
Mcv 71.7
Mch 25.1
Mchc 350
PLT 264
I am a bit concerned about the elevated red blood cells. Does this have anything to do with an iron deficiency or not?
Thank you in advance :)

Sent by Ereza, më 16 June 2022 në 01:43

Hello Ereza, you will do ferritinemia and hemoglobin electrophoresis

Replay from Dr. Shk. Sotiraq Lako, më 16 June 2022 në 02:43

Thank you very much.
Forgive me, the HGB is 132, I wrote it wrong. And MCV and MCH are slightly reduced.
Is there something worrying??

Sent by Ereza, më 16 June 2022 në 02:49

Hello Ereza, you will undergo ferritinemia. If it is normal (> 20 ng/ml and the optimal value is around 60 ng/ml), you have no problems. If it is low, you will be treated for iron deficiency

Replay from Dr. Shk. Sotiraq Lako, më 17 June 2022 në 14:08

Thank you very much.
Sorry, the HGB is 132, I wrote it wrong. And MCV and MCH are slightly decreased.
Is this something worrisome?

Sent by Ereza, më 16 June 2022 në 17:05

Hello Ereza, you will have a ferritinemia test. If it is normal (> 20 ng/ml and the optimal value is around 60 ng/ml), you have no problems. If it is low, you will be treated for iron deficiency

Replay from Dr. Shk. Sotiraq Lako, më 17 June 2022 në 14:07

Thank you very much.
I apologize because the HGB is 132, I wrote it wrong. And MCV and MCH are slightly reduced.
Is this something worrisome?

Sent by Ereza, më 17 June 2022 në 02:12

Hello Ereza, you will have ferritinemia. If it is normal (> 20 ng/ml and the optimal value is about 60 ng/ml), you have no problems. If it is low, you will be treated for iron deficiency

Replay from Dr. Shk. Sotiraq Lako, më 17 June 2022 në 14:08

Thank you very much.
Sorry, the HGB is 132, I wrote it wrong. And MCV and MCH are slightly decreased.
Is this something worrisome?

Sent by Ereza, më 17 June 2022 në 12:19

Hello Ereza, you will have a ferritinemia test. If it is normal (> 20 ng/ml and the optimal value is around 60 ng/ml), you have no problems. If it is low, you will be treated for iron deficiency

Replay from Dr. Shk. Sotiraq Lako, më 17 June 2022 në 14:07

Thank you very much.
I apologize because the HGB is 132, I wrote it wrong. And MCV and MCH are slightly decreased.
Is there something to worry about??

Sent by Ereza, më 17 June 2022 në 13:24

Hello Ereza, you will do a ferritinemia test. If it is normal (> 20 ng/ml and the optimal value is around 60 ng/ml), you don't have any problems. If it is low, you will be treated for iron deficiency

Replay from Dr. Shk. Sotiraq Lako, më 17 June 2022 në 14:07

Thank you very much.
I apologize, the HGB is 132, I wrote it wrong. And MCV and MCH are slightly decreased.
Is this something worrisome?

Sent by Ereza, më 17 June 2022 në 14:49

Hello Ereza, you will undergo a ferritin test. If it is normal (> 20 ng/ml and the optimal value is around 60 ng/ml), you have no problems. If it is low, you will be treated for iron deficiency

Replay from Dr. Shk. Sotiraq Lako, më 18 June 2022 në 04:02

Thank you very much.
Sorry, the HGB is 132, I wrote it wrong. And MCV and MCH are slightly decreased.
Is this something to worry about?

Sent by Ereza, më 18 June 2022 në 00:24

Hello Ereza, you will do a ferritinemia. If it is normal (> 20 ng/ml and the optimal value is around 60 ng/ml), you don't have any problems. If it is low, you will be treated for iron deficiency

Replay from Dr. Shk. Sotiraq Lako, më 18 June 2022 në 04:02

Hello doctor!
First of all, I thank you for your answers and advice!
Three years ago, I did a ferritin test because my vision would darken when I stood up suddenly, and I had no other issues, no fatigue or dizziness at all. I have taken medication but at different periods. Now my iron is at 35, but I am concerned because I experience dizziness, loss of concentration for moments that don't last long, just seconds, but often... And I notice it when I am sitting and not moving. Please, what tests should I do? Thank you!

Sent by Sidorela, më 25 June 2022 në 17:38

Hello Sidorela, if the value of 35 ng/ml is for ferritinemia, you have a good value. If it is sideremia, it is low and you will receive treatment with iron supplements

Replay from Dr. Shk. Sotiraq Lako, më 27 June 2022 në 06:41

Hello doctor. I have been experiencing iron deficiency for years. My ferritin level in the latest test was 10.20; in September, it was 22. I have used Tardyferon but haven't been very consistent with the medication. My serum iron is 69.9. Electrophoresis results have been good the other times I've had it done. Vitamin D is 12. Should I continue taking Tardyferon three times a day? Or is there another medication you could recommend, please?

Sent by linda, më 28 June 2022 në 07:33

Hello Linda, Tardyferon is a good preparation. 3 times a day, for 2 months in a row

Replay from Dr. Shk. Sotiraq Lako, më 28 June 2022 në 09:43

Hello, doctor. I have had iron deficiency for years. My ferritin level in the latest test was 10.20; in September, it was 22. I have been using Tardyferon but haven't been very consistent with the medication. My serum iron is 69.9. My electrophoresis results have been good the other times I've had it done. Vitamin D is 12. Should I continue with Tardyferon three times a day? Or should I try another medication, please?

Sent by linda, më 30 June 2022 në 06:18

Hello Linda, Tardyferon is a good preparation, but for replenishing iron stores, Heferol is more successful

Replay from Dr. Shk. Sotiraq Lako, më 30 June 2022 në 08:04

Hello Doctor, I hope you are well. I have been experiencing frequent heart palpitations, weakness, and insomnia for about a week. I went for an echocardiogram and the doctor suggested that I get a ferritin test. My blood tests show RBC 4.83, hemoglobin 13.9, MCV 85.3, MCH 28.8, MCHC 33.7. Can ferritin be done and why are the blood tests normal? Thank you in advance for your answer

Sent by Era, më 06 Agust 2022 në 08:44

Hello Era, the palpitations are not related to your hemoglobin level, which is normal. However, ferritin levels need to be checked even when hemoglobin is normal

Replay from Dr. Shk. Sotiraq Lako, më 09 Agust 2022 në 08:36

Hello Doctor, I hope you are well. For about a week, I have been experiencing frequent weak heartbeats, insomnia. I went for an echocardiogram, and the doctor suggested I get a ferritin test. My blood tests are RBC 4.83, hemoglobin 13.9, MCV 85.3, MCH 28.8, MCHC 33.7. Can ferritin still be done and why would it be necessary if the blood tests are normal? Thanks in advance for your response

Sent by Era, më 06 Agust 2022 në 10:13

Hello Era, the palpitations are not related to your hemoglobin level which is normal. But ferritinemia should still be done even when the hemoglobin is normal

Replay from Dr. Shk. Sotiraq Lako, më 09 Agust 2022 në 08:36

Hello doctor. Can you tell me what the long-term side effects of taking intravenous iron are?

Sent by Anisa, më 21 Agust 2022 në 15:11

Hello Anisa, so far we are not aware of any long-term side effects from intravenous iron treatment. The possibility of iron overload is almost impossible, as long as the administration is controlled depending on the values of hemoglobin and ferritin

Replay from Dr. Shk. Sotiraq Lako, më 22 Agust 2022 në 04:42

Hello doctor!
I hope you are well!
On the date of 07/15/2022, I had some tests done and these were the results. Hemoglobin 11.1
Ferritin 8.23
Serum iron 59
I have been using Veltifer for a month. Is it a good medication? Thank you!

Sent by Anxhela, më 28 September 2022 në 13:00

Hello Anxhela, you have mild anemia due to iron deficiency. You will take Heferol 350 mg 1 capsule + Vitamin C 100 mg 1 tablet, in the morning, during or after meals, every 2 days, for 1 month. Recheck after treatment with complete blood count. Then, treatment will continue for correcting ferritinemia for another 2 months. After 3 months, recheck with complete blood count and ferritinemia (the minimum normal value for adult females is 20 ng/ml and the optimal value is 60-70 ng/ml)

Replay from Dr. Shk. Sotiraq Lako, më 29 September 2022 në 06:33

Hello doctor!
I hope you are well!
On 15/07/2022, I did some tests, and these were the results. Hemoglobin 11.1
Ferritin 8.23
Serum Iron 59
I have been using Veltifer for 1 month. Is it a good medication? Thank you!

Sent by Anxhela, më 28 September 2022 në 23:11

Hello Anxhela, you have mild anemia due to iron deficiency. You will take Heferol 350 mg 1 capsule + Vitamin C 100 mg 1 tablet, in the morning, during, or after meals, every 2 days, for 1 month. Recheck after treatment with complete blood count. Afterward, treatment will continue for correcting ferritinemia for another 2 months. After 3 months, recheck with complete blood count and ferritinemia (the minimal normal value for adult females is 20 ng/ml and the optimal value is 60-70 ng/ml)

Replay from Dr. Shk. Sotiraq Lako, më 29 September 2022 në 06:33

Hello doctor!
I hope you are well!
On 15/07/2022, I did some tests and these were the results. Hemoglobin 11.1
Ferritin 8.23
Serum Iron 59
I have been using Veltifer for a month. Is it a good medication? Thank you!

Sent by Anxhela, më 29 September 2022 në 02:41

Hello Anxhela, you have mild anemia due to iron deficiency. You will take Heferol 350 mg 1 capsule + Vitamin C 100 mg 1 tablet, in the morning, in between or after meals, every 2 days, for 1 month. Recheck after treatment with complete blood count. Afterwards, treatment will continue for correcting ferritin levels for another 2 months. After 3 months, recheck with complete blood count and ferritin levels (the minimum normal value for adult females is 20 ng/ml and the optimal value is 60-70 ng/ml)

Replay from Dr. Shk. Sotiraq Lako, më 29 September 2022 në 06:34

Hello doctor!
I hope you are well!
On the date of 15/07/2022, I did some tests and these were the results. Hemoglobin 11.1
Ferritin 8.23
Serum Iron 59
I have been using Veltifer for 1 month. Is it a good medication? Thank you!

Sent by Anxhela, më 29 September 2022 në 07:08

Hello Anxhela, you have mild anemia due to iron deficiency. You will take Heferol 350 mg 1 capsule + Vitamin C 100 mg 1 tablet, in the morning, in the middle or after food, every 2 days, for 1 month. Recheck after treatment with complete blood work. Then the treatment will continue for correcting ferritinemia for another 2 months. After 3 months, recheck with complete blood work and ferritinemia (the minimal normal value for adult females is 20 ng/ml and the optimal value is 60-70 ng/ml)

Replay from Dr. Shk. Sotiraq Lako, më 30 September 2022 në 06:09

Hello doctor!
I hope you are well!
On 15/07/2022, I had my tests, and these are the results. Hemoglobin 11.1
Ferritin 8.23
Serum iron 59
I have been using Veltifer for 1 month. Is it good as a medication? Thank you!

Sent by Anxhela, më 29 September 2022 në 12:27

Hello Anxhela, you have mild anemia due to iron deficiency. You will take Heferol 350 mg 1 capsule + Vitamin C 100 mg 1 tablet, in the morning, during, or after food, every 2 days, for 1 month. Recheck after treatment with complete blood count. Afterwards, treatment will continue for correcting ferritin levels for another 2 months. After 3 months, recheck with complete blood count and ferritin levels (normal minimum value for adult females 20 ng/ml and optimal value 60-70 ng/ml)

Replay from Dr. Shk. Sotiraq Lako, më 30 September 2022 në 06:09

Hello doctor!
I hope you are well!
On 15/07/2022, I did blood tests and these were the results. Hemoglobin 11.1
Ferritin 8.23
Serum iron 59
I have been using Veltifer for 1 month. Is it a good medication? Thank you!

Sent by Anxhela, më 29 September 2022 në 12:33

Hello Anxhela, you have mild anemia due to iron deficiency. You will take Heferol 350 mg 1 capsule + Vitamin C 100 mg 1 tablet, in the morning, during, or after food, every 2 days, for 1 month. Recheck after treatment with complete blood count. Then, treatment will continue to correct ferritinemia for another 2 months. After 3 months, recheck with complete blood count and ferritinemia (normal minimum value for adult females 20 ng/ml and optimal value 60-70 ng/ml)

Replay from Dr. Shk. Sotiraq Lako, më 30 September 2022 në 06:09

Hello doctor.
I am 21 years old. During my last tests, my hemoglobin was 10.7 and my ferritin was 6.6.
Currently, I am using Legofer. I have been taking medication for a year but I have not seen any improvements. Thank you

Sent by Klea Kryemadhi, më 02 October 2022 në 04:35

Hello Klea, you have mild Anemia due to Iron Deficiency. Legofer is not a preparation for correcting iron stores. You will take for at least 3 consecutive months: 1-Heferol 350 mg 1 capsule/day – 30 capsules/month. 2-Vitamin C 100 mg 1 tablet/day – 30 tablets/month. 3-Vitamin B6 25 mg 1 tablet/day – 30 tablets/month. The patient will be rechecked with complete blood count after 1 month of treatment and after 3 months of treatment with complete blood count + ferritinemia (normal minimum value for adult females 20 ng/ml and optimal value 60-70 ng/ml)

Replay from Dr. Shk. Sotiraq Lako, më 03 October 2022 në 02:58

Hello doctor.
I am 21 years old, my last tests showed that my hemoglobin was 10.7 and ferritin was 6.6.
I am currently using Legofer. I have been using medication for a year but have not seen any improvements. Thank you

Sent by Klea Kryemadhi, më 02 October 2022 në 15:11

Hello Klea, you have mild Anemia due to Iron Deficiency. Legofer is not a preparation for correcting iron stores. You will take for at least 3 months in a row:
1- Heferol 350 mg 1 capsule/day - 30 capsules/month.
2- Vitamin C 100 mg 1 tablet/day - 30 tablets/month.
3- Vitamin B6 25 mg 1 tablet/day - 30 tablets/month.
The patient will be rechecked with complete blood count after 1 month of treatment and after 3 months of treatment with complete blood count + ferritinemia (normal minimal value for adult females 20 ng/ml and optimal value 60-70 ng/ml)

Replay from Dr. Shk. Sotiraq Lako, më 03 October 2022 në 02:58

Hello doctor.
I am 21 years old, when I did my last tests, my hemoglobin was 10.7 and ferritin last time was 6.6.
Currently, I am using Legofer. I have been using medication for a year but haven't seen any improvements. Thank you

Sent by Klea Kryemadhi, më 02 October 2022 në 17:04

Hello Klea, you have mild Anemia due to Iron Deficiency. Legofer is not a preparation for correcting iron stores. You will take for at least 3 months in a row: 1-Heferol 350 mg 1 capsule/day – 30 capsules/month.
2-Vitamin C 100 mg 1 tablet/day – 30 tablets/month. 3-Vitamin B6 25 mg 1 tablet/day – 30 tablets/month. The patient will be rechecked with complete blood count after 1 month of treatment and after 3 months of treatment with complete blood count + ferritinemia (the normal minimum value for adult females is 20 ng/ml and the optimal value is 60-70 ng/ml)

Replay from Dr. Shk. Sotiraq Lako, më 03 October 2022 në 02:58

Hello doctor.
I am 21 years old, when I did my last tests, my hemoglobin came out to be 10.7 and ferritin recently 6.6.
Currently, I am using Legofer. I have been using medications for a year but have not seen any improvements. Thank you

Sent by Klea Kryemadhi, më 02 October 2022 në 17:05

Hello Klea, you have mild Anemia from Iron Deficiency. Legofer is not a preparation for correcting iron stores. You will take for at least 3 months straight: 1-Heferol 350 mg 1 capsule/day – 30 capsules/month. 2-Vitamin C 100 mg 1 tablet/day – 30 tablets/month. 3-Vitamin B6 25 mg 1 tablet/day – 30 tablets/month. The patient will be rechecked with a complete blood count after 1 month of treatment and after 3 months of treatment with a complete blood count + ferritinemia (the normal minimum value for adult females is 20 ng/ml and the optimal value is 60-70 ng/ml)

Replay from Dr. Shk. Sotiraq Lako, më 03 October 2022 në 02:58

Hello doctor.
I am 21 years old, when I did my last analysis, my hemoglobin was 10.7 and my last ferritin was 6.6.
I am currently using Legofer. I have been using medication for a year but have not seen any improvements. Thank you

Sent by Klea Kryemadhi, më 03 October 2022 në 15:06

Hello Klea, you have mild Anemia due to Iron Deficiency. Legofer is not the preparation for the correction of iron stores. You will take for at least 3 months in a row: 1-Heferol 350 mg 1 capsule/day – 30 capsules/month. 2-Vitamin C 100 mg 1 tablet/day – 30 tablets/month. 3-Vitamin B6 25 mg 1 tablet/day – 30 tablets/month. The patient will be rechecked with a complete blood count after 1 month of treatment and after 3 months of treatment with a complete blood count + ferritinemia (the minimum normal value for adult females is 20 ng/ml and the optimal value is 60-70 ng/ml)

Replay from Dr. Shk. Sotiraq Lako, më 04 October 2022 në 02:16

Hello doctor! I am 24 years old.
Hemoglobin 12.6
Ferritin 8.12
Please give me advice on how to use these medications: vitamin C 100 mg, vitamin B6 25 mg, and Tardyferon 80 mg

Sent by Xhilda , më 05 October 2022 në 05:05

Hello Xhilda, you have an iron deficiency. You do not need Vitamin B6. Tardyferon is not an effective preparation for replenishing iron stores

Replay from Dr. Shk. Sotiraq Lako, më 05 October 2022 në 05:08

Hello doctor. I wrote to you a few days ago and you wrote back that I need to use heferol every 2 days.
Every 2 days or twice a day?? Because I'm not very clear

Sent by Anxhela , më 06 October 2022 në 02:39

Hello Anxhela, new studies show that by taking it every 2 days, absorption is greater and helps you to lower the dose and side effects. Take it every 2 days

Replay from Dr. Shk. Sotiraq Lako, më 06 October 2022 në 06:43

Hello doctor. I wrote to you a few days ago and you wrote back that I need to use heferol every 2 days.
Every 2 days or 2 times a day?? Because I am not very clear

Sent by Anxhela , më 06 October 2022 në 04:44

Hello Anxhela, new studies show that by taking it every 2 days, absorption is greater and it helps you to lower the dose and side effects. Take it every 2 days

Replay from Dr. Shk. Sotiraq Lako, më 06 October 2022 në 06:42

Hello doctor. I wrote to you a few days ago and you replied that I should use heferol every 2 days.
Every 2 days or 2 times a day?? Because I am not very clear

Sent by Anxhela , më 06 October 2022 në 13:02

Hello Anxhela, new studies show that by taking it every 2 days, absorption is greater and it helps you to reduce the dose and the side effects. Take it every 2 days

Replay from Dr. Shk. Sotiraq Lako, më 07 October 2022 në 05:25

Hello doctor. I wrote to you a few days ago and you wrote back that I need to use heferol every 2 days.
Every 2 days or twice a day?? Because I'm not very clear

Sent by Anxhela , më 07 October 2022 në 00:38

Hello Anxhela, new studies show that by taking it every 2 days, absorption is greater and it helps you to lower the dose and side effects. Take it every 2 days

Replay from Dr. Shk. Sotiraq Lako, më 07 October 2022 në 05:25

Hello doctor, I am 48 years old, HGB 13.5, RDW 15.6%, Ferritin 9.81ng/ml. Please tell me what I should use since they told me I have an iron deficiency. Thank you

Sent by Rajmonda, më 07 October 2022 në 04:24

Hello Rajmonda, you have an iron deficiency. You will be treated with iron supplements. The best supplement is Heferoli

Replay from Dr. Shk. Sotiraq Lako, më 07 October 2022 në 05:27

Hello Doctor! I hope you are well! I am 19 years old and three months ago I did some tests and my hemoglobin was 12.6 while ferritin was 8.4. I used a medication that my doctor recommended for three months, the medication was called Ferrodep, and today I did a complete blood test again and my hemoglobin was 13.1 while ferritin was 8.48. So from what I saw from the analysis and what the lab technician told me, the ferritin had not moved much. Should I continue taking this medication or should I use something else?

Sent by Gejsa, më 10 October 2022 në 06:40

Hello Gejsa, you have an iron deficiency. The best preparation we have in Albania for replenishing iron stores is Heferoli. As a rule, the treatment continues for at least 2 months in a row and you should be re-checked with ferritinemia after the treatment

Replay from Dr. Shk. Sotiraq Lako, më 11 October 2022 në 06:17

Hello Doctor! I hope you are well! I am 19 years old and three months ago I did blood tests and my hemoglobin was 12.6 while ferritin was 8.4. I used a medication recommended by the doctor for 3 months, Ferrodep was the name of the medication, and today I did the complete blood tests again and the hemoglobin was 13.1 while ferritin was 8.48. So, from what I saw in the analysis and what the lab technician told me, the ferritin hasn't moved much. Should I continue to take this medication again or should I use something else?

Sent by Gejsa, më 10 October 2022 në 06:42

Hello Gejsa, you have an iron deficiency. The best preparation we have in Albania for replenishing iron stores is Heferol. As a rule, treatment continues for at least 2 months in a row and you should get rechecked with ferritinemia after the treatment

Replay from Dr. Shk. Sotiraq Lako, më 11 October 2022 në 06:18

Hello Doctor! I hope you are well! I am 19 years old and three months ago I did some tests and my hemoglobin was 12.6 while my ferritin was 8.4. I used a medication that the doctor recommended for 3 months, Ferrodep was the name of the medication, and today I did a complete blood test again and my hemoglobin was 13.1 while my ferritin was 8.48. So, from what I saw in the analysis and what the lab technician told me, the ferritin hadn't moved much. Should I continue to take this medication, or should I use something else?

Sent by Gejsa, më 10 October 2022 në 06:43

Hello Gejsa, you have an iron deficiency. The best preparation we have in Albania for replenishing iron stores is Heferol. As a rule, the treatment continues for at least 2 months in a row, and you should be re-checked for ferritinemia after the treatment

Replay from Dr. Shk. Sotiraq Lako, më 11 October 2022 në 06:18

Hello Doctor! I hope you are well! I am 19 years old and three months ago I did some tests and my hemoglobin was 12.6 while my ferritin was 8.4. I used a medication that the doctor recommended for 3 months, Ferrodep was the name of the medication, and today I did the complete blood tests again and the hemoglobin was 13.1 while the ferritin was 8.48. So, from what I saw in the analysis and what the laboratory technician told me, the ferritin hadn’t moved much. Should I continue taking this medication or should I use something else?

Sent by Gejsa, më 10 October 2022 në 13:12

Hello Gejsa, you have an iron deficiency. The best preparation we have in Albania for replenishing iron stores is Heferol. As a rule, the treatment continues for at least 2 months in a row and you should be rechecked with ferritinemia after the treatment

Replay from Dr. Shk. Sotiraq Lako, më 11 October 2022 në 06:18

Hello Doctor! I hope you are well! I am 19 years old and three months ago I did some blood tests and my hemoglobin was 12.6 while my ferritin was 8.4. I used a medication suggested by the doctor for 3 months, the name of the medication was Ferrodep, and today I did a complete blood test again and my hemoglobin was 13.1 while my ferritin was 8.48. So, from what I saw from the analysis and what the lab technician told me, my ferritin had not moved much. Should I continue taking this medication or should I use something else?

Sent by Gejsa, më 10 October 2022 në 20:37

Hello Gejsa, you have an iron deficiency. The best preparation we have in Albania for replenishing iron stores is Heferoli. As a rule, treatment continues for at least 2 months in a row and you should be rechecked for ferritin levels after treatment

Replay from Dr. Shk. Sotiraq Lako, më 11 October 2022 në 06:18

Hello Doctor! I hope you are well! I am 19 years old and three months ago I did some tests and my hemoglobin was 12.6 while my ferritin was 8.4. I used a medication suggested by my doctor for 3 months, it was called Ferrodep, and today I did a complete blood test again and my hemoglobin was 13.1 while my ferritin was 8.48. So, from what I saw in the analysis and what the lab technician told me, the ferritin hadn't moved much. Should I continue taking this medication or should I use something else?

Sent by Gejsa, më 11 October 2022 në 07:40

Hello Gejsa, you have an iron deficiency. The best supplement we have in Albania for replenishing iron stores is Heferoli. As a general rule, the treatment continues for at least 2 months in a row and you should be rechecked with ferritinemia after the treatment

Replay from Dr. Shk. Sotiraq Lako, më 12 October 2022 në 04:01

Hello doctor.
I am 21 years old, when I did my last tests, my hemoglobin came out to be 10.7 and ferritin was 6.6.
Currently, I am using Legofer. I have been using medication for a year but I have not seen any improvements. Thank you

Sent by Klea Kryemadhi, më 13 October 2022 në 17:09

Hello Klea, you have mild anemia due to iron deficiency. Legofer is not the preparation that regulates iron stores. Heferol is the preparation that can do such a thing

Replay from Dr. Shk. Sotiraq Lako, më 14 October 2022 në 02:03

Hello! Due to some increased concerns lately, such as fatigue, sweaty hands, nervousness, restless sleep, hair loss. I did a series of tests, where the ferritin came out to be 12 and I am currently taking 2×1 capsules of ironorm. Is that enough?
Thank you

Sent by Ana, më 21 October 2022 në 07:46

Hello Ana, you can continue for 1.5-2 months and then repeat the ferritinemia. The goal is > 20 ng/ml and the optimal value is around 60 ng/ml

Replay from Dr. Shk. Sotiraq Lako, më 22 October 2022 në 13:59

Hello! Referring to some concerns that have increased in the last few days, such as fatigue, sweaty hands, nervousness, restless sleep, hair loss. I did a series of tests, where ferritin turned out to be 12. I am currently taking 2×1 capsules of ironorm. Is it enough?

Thank you

Sent by Ana, më 21 October 2022 në 07:48

Hello Ana, you can continue for 1.5-2 months and then repeat the ferritinemia. The goal is > 20 ng/ml and the optimal value is around 60 ng/ml

Replay from Dr. Shk. Sotiraq Lako, më 22 October 2022 në 13:59

Hello Doctor! I hope you are well! I am 19 years old and three months ago I did some tests and my hemoglobin was 12.6 while my ferritin was 8.4. I used a medication that the doctor recommended to me for 3 months, Ferrodep was the name of the medication, and today I did a complete blood test again and the hemoglobin was 13.1 while the ferritin was 8.48. So, as far as I saw from the analysis and as the lab technician told me, the ferritin hadn't moved much. Should I continue taking this medication or should I use something else?

Sent by Gejsa, më 21 November 2022 në 04:29

Hello Gejsa, you had mild anemia due to iron deficiency. The preparation you have taken is an iron supplement and iron stores are not replenished with supplements ("injections"). The best preparation we have is Heferoli, 2 capsules/day, for 1.5 months straight, or in Italy Ferrograd 105 mg 2 x 1 tablet/day, or in Greece Ferro Sanol Duodenal, 2 x 1 capsule/day

Replay from Dr. Shk. Sotiraq Lako, më 22 November 2022 në 09:41

Doctor, hello. My son is 5 years old, he was very pale. I did his tests and they came out bad?! On 08/08/22, he had these results: hemoglobin 5.4, ferritin 1.2, and others were done as well. But these were very low. After continuing with the medication, he now has these values: hemoglobin 9.6 and ferritin 2.9, he also takes a multivitamin. I would like an answer from you, thank you

Sent by Anxhelo Blloshmi, më 05 December 2022 në 14:30

Hello Anxhelo, the previous value indicates significant anemia; the fact that it has improved with treatment suggests that the complexes he has taken did contain iron, but not in the appropriate amount. The goal is to take the proper iron supplement to ensure the correction of anemia (Hb > 11 gr/dl) and then, by continuing the treatment for another 2 months, aim to correct the ferritin levels. Not every supplement can achieve these goals. You should discuss with the pediatrician about the appropriate treatment (e.g., Sol. Hemafer 50 mg/ml etc.)

Replay from Dr. Shk. Sotiraq Lako, më 06 December 2022 në 03:16

Doctor hello, my son is 5 years old and he was very pale. I did his blood tests and the results were bad?! On 08/08/22, he had these results: hemoglobin 5.4, ferritin 1.2, and there were other tests done but these were very low. After continuing with the treatment, now he has these values: hemoglobin 9.6 and ferritin 2.9, he also takes iron and multivitamins. I would like an answer from you, thank you

Sent by Anxhelo Blloshmi, më 05 December 2022 në 14:30

Hello Anxhelo, the previous value indicates significant anemia, the fact that it has improved from treatment shows that the complexes he has received contained iron, but not in the right amount. The goal is to take the right iron supplement to ensure the correction of anemia (Hb > 11 gr/dl) and then by continuing treatment for 2 more months, the correction of ferritin can be aimed for. Not every supplement can achieve these goals. You will discuss with the pediatrician for the appropriate treatment (e.g., Sol. Hemafer 50 mg/ml etc.)

Replay from Dr. Shk. Sotiraq Lako, më 06 December 2022 në 03:16

Doctor, hello how are you? I am 38 years old, weighing 54 kg. Thank you in advance for your answer. I've been feeling unwell for a few days after an antibiotic course. I feel tired, have insomnia, and a rapid pulse. The results of my tests are as follows: hemoglobin 13.9, erythrocytes 4.83, hematocrit 41.3, MCV 85.5, MCH 28.8, MCHC 33.7, RDW-SD 36.8, platelets 240, ferritin 26.41. My question is: can iron preparations or supplements be taken and why is it within the normal range when I read from you that the ideal for ferritin for women is 60-70? And if so, which preparation is better. Heferol or Ironorm? How can they be used? And is it possible to develop iron overload from taking these medications? Thank you and have a nice day!

Sent by Erisa, më 19 December 2022 në 04:33

Hello Erisa, you have good values. Since ferritinemia is recommended to be around 60-70 ng/ml, take iron for 1 month. Heferol and Ironorm are both good. Iron overload does not occur when iron is taken orally

Replay from Dr. Shk. Sotiraq Lako, më 19 December 2022 në 08:08

Doctor, hello, how are you? I am 38 years old, weighing 54 kg. Thank you in advance for your response. I have been feeling unwell for a few days following an antibiotic treatment. I feel tired, have insomnia, and a rapid pulse. The results of my tests are as follows: hemoglobin 13.9, erythrocytes 4.83, hematocrit 41.3, MCV 85.5, MCH 28.8, MCHC 33.7, RDWSD 36.8, platelets 240, ferritin 26.41. My question is: Can iron preparations or supplements be taken, and why is it considered normal when I read from you that the ideal ferritin level for women is 60-70? And if so, which preparation is better, Heferol or Ironorm? How should they be used? And is it possible to develop an iron overload from taking these medications? Thank you and have a good day!

Sent by Erisa, më 19 December 2022 në 04:33

Hello Erisa, you have good values. Since ferritinemia is recommended around 60-70 ng/ml, take iron for 1 month. Heferol and Ironorm are both good. There is no excess iron created when iron is taken orally

Replay from Dr. Shk. Sotiraq Lako, më 19 December 2022 në 08:08

Thank you for your answer. How can I use them? If I take heferol or ironorm, how many times a day? What do you recommend me to drink?

Sent by Erisa, më 19 December 2022 në 11:22

Hello Erisa, for treatment please refer to your hematologist, or the family doctor

Replay from Dr. Shk. Sotiraq Lako, më 20 December 2022 në 04:43

Thank you for your response. How can I use them? If I take heferol or ironorm, how many times a day? What do you recommend I drink?

Sent by Erisa, më 19 December 2022 në 12:38

Hello Erisa, for treatment, please consult your hematologist or family doctor

Replay from Dr. Shk. Sotiraq Lako, më 20 December 2022 në 04:42

Hello Doctor. I am 38 years old. Three years ago, I underwent treatment for anemia. My hemoglobin was 8, while my ferritin was 5. I have done some tests a few days ago, and my hemoglobin came out to be 11, while my ferritin is 3.22.
What medication should I use? Because the ones I have taken did not have much effect.
Thank you!

Sent by Suela Duka, më 20 December 2022 në 09:57

Hello Suela, you currently have mild anemia due to iron deficiency. The most successful preparations in Albania are Heferoli and Ironorm

Replay from Dr. Shk. Sotiraq Lako, më 21 December 2022 në 02:54

Hello Doctor. I am 38 years old. Three years ago, I underwent treatment for anemia. My hemoglobin was 8 while my ferritin was 5. I have had some tests done a few days ago and my hemoglobin came out as 11 while my ferritin is 3.22. What medication should I use? Because the ones I have used did not have much effect. Thank you!

Sent by Suela Duka, më 20 December 2022 në 12:40

Hello Suela, you currently have mild anemia due to iron deficiency. The most successful preparations in Albania are Heferoli and Ironorm

Replay from Dr. Shk. Sotiraq Lako, më 21 December 2022 në 02:55

Hello Doctor. I am 38 years old. Three years ago, I underwent treatment for anemia. My hemoglobin was 8 while my ferritin was 5. I have done some tests a few days ago, and my hemoglobin came out to be 11 while my ferritin is 3.22. What medication should I use? Because the ones I have used did not have much effect. Thank you!

Sent by Suela Duka, më 21 December 2022 në 00:00

Hello Suela, you currently have mild anemia due to iron deficiency. The most successful preparations in Albania are Heferoli and Ironorm

Replay from Dr. Shk. Sotiraq Lako, më 21 December 2022 në 02:54

Hello Dr, I am 51 years old
I always measure my ferritin and it comes out 600/700 or 800. They have told me it's genetic. Is there any risk?

Sent by Selami, më 03 January 2023 në 02:15

Hello Selami, you have a slight increase in ferritinemia. As a value, it does not cause you complaints and consequences. Why it is elevated needs to be assessed

Replay from Dr. Shk. Sotiraq Lako, më 03 January 2023 në 06:02

Doctor, hello. Happy New Year and may it be a better year!
I wanted to consult with you about a concern. In the latest tests, the results are hemoglobin 14.2, sideremia 144, ferritin 22.5, RBC 4.90, hematocrit 42.3, MCV 86.3, MCH 29, MCHC 33.6, RDW 14.3. What do you recommend for me to take to increase my iron stores? My family doctor thinks that eating healthily is enough, but from what I've read from you, the stores don't just increase by themselves. I have never taken iron supplements, only multivitamins that don't contain iron. The dilemma is whether to wait until the ferritin drops below 20 and then take iron, or not let it drop and start taking it now to be optimal. What side effects does it have? Waiting for your response. Thank you

Sent by Alda, më 10 January 2023 në 07:40

Hello, the value of ferritin is at the lower limits of normal. There is room for improvement and it cannot be achieved through food alone. For adult patients, a good option is Heferol

Replay from Dr. Shk. Sotiraq Lako, më 10 January 2023 në 08:06

Please do not tell me to consult with the family doctor as the same told me to eat healthy and that's it

Sent by Alda, më 10 January 2023 në 09:21

Hello Alda, treatment is a responsibility and several criteria need to be evaluated before it is given. The overall condition, age, weight, gastrointestinal disorders, the amount of blood lost during the menstrual cycle, if any, the frequency of the cycle, other possible blood losses, etc., need to be assessed

Replay from Dr. Shk. Sotiraq Lako, më 13 January 2023 në 02:38

Good evening Doctor, how are you? Can you inform me how I should take ironorm, how many times a day and how?
Ferritin 22. I am trying to get pregnant and was advised to take this supplement. Hemoglobin 13, MCV 87, MCH 27.7, MCHC 31.8. Thank you. I haven't had any concerns except for tiredness and dark circles under the eyes.
Thank you

Sent by Daniela, më 11 January 2023 në 10:03

Hello Daniela, 3 times a day, for 1-1.5 months

Replay from Dr. Shk. Sotiraq Lako, më 12 January 2023 në 02:52

Good evening Doctor, I would like some advice and help from you... I've been found to be anemic with hemoglobin at 99L, Hematocrit at 33.2L.. Iron at 2.4L, Ferritin at 6.35L. I've used Heto theme for 1 month straight, Sider AL FORTE INT. NO RESULT, I did this test FOB-TEST(ADLER WEBER) the result <25.0 normal value what would you suggest I use for Anemia please Thank you

Sent by Lolajara, më 12 January 2023 në 10:14

Hello Lolajara, you have mild Anemia due to Iron Deficiency. If you live in Albania, one option is to use Ironorm or Heferol, with a therapeutic dose. The treatment lasts at least 3 consecutive months, with the aim of correcting the anemia and replenishing iron reserves

Replay from Dr. Shk. Sotiraq Lako, më 13 January 2023 në 03:11

Good evening Doctor, I need some advice and help from you... I have been found to be anemic with hemoglobin at 99L, Hematocrit at 33.2L. Iron at 2.4L, Ferritin at 6.35L. I have used Sider AL FORTE INT for a month straight with NO RESULTS. I did this test FOB-TEST(ADLER WEBER) and the result was <25.0, which is normal. What would you recommend for me to use for Anemia, please? Thank you

Sent by Lolajara, më 12 January 2023 në 10:16

Hello Lolajara, you have mild Anemia due to Iron Deficiency. If you live in Albania, one option is to use Ironorm or Heferoli, with a medicinal dose. The treatment lasts for at least 3 months in a row, with the goal of correcting the anemia and the iron reserves

Replay from Dr. Shk. Sotiraq Lako, më 13 January 2023 në 03:12

Good evening Doctor, I needed some advice and help from you... I have been diagnosed with anemia, hemoglobin at 99L, Hematocrit at 33.2L. Iron 2.4L, Ferritin 6.35L. I have used Sider AL FORTE INT for 1 month in a row without any results. I did this test FOB-TEST(ADLER WEBER) and the result was <25.0, which is normal. What would you suggest I use for Anemia, please? Thank you

Sent by Lolajara, më 12 January 2023 në 13:58

Hello Lolajara, you have mild Anemia due to Iron Deficiency. If you live in Albania, one option is to use Ironorm or Heferoli, with a medicinal dose. The treatment lasts at least 3 consecutive months, with the aim of correcting the anemia and the iron reserves

Replay from Dr. Shk. Sotiraq Lako, më 13 January 2023 në 03:11

Hello again, Dr. Lako. Yesterday, I took ironorm one hour after eating. It felt like a rock in my stomach, and I think I won't be able to take it three times a day, especially since it needs to be taken 2 hours away from food and certain specific foods that we will inevitably have in a meal. It's all day managing the situation, plus I had stomach discomfort even though I usually don't suffer from it. Is there any other possibility or another preparation? Reminding you, ferritin 22, hemoglobin 13, erythrocytes 4.7, hematocrit 40.9.
Thank you in advance.
13 mcv 87 mch 27.7 mchc 31.8

Sent by Daniela, më 12 January 2023 në 16:33

Hello Daniela, you can use Sideral forte 30 mg, you can start with 1 capsule/day. If it is not tolerated, treatment with iron preparations with serum will be evaluated, but they are only done in hospital conditions

Replay from Dr. Shk. Sotiraq Lako, më 13 January 2023 në 03:09

Hello again Dr. Lako. Yesterday I took ironorm one hour after eating. It felt like a rock in my stomach, I think I won't be able to take it three times a day especially since it needs to be taken 2 hours away from food and certain foods that we like will be in one meal or another. It means managing the situation all day plus I had stomach discomfort even though I don't usually suffer from it. Is there another possibility or another preparation? Reminding you, ferritin 22, hemoglobin 13, erythrocytes 4.7, hematocrit 40.9
Thank you in advance
MCV 87 MCH 27.7 MCHC 31.8

Sent by Daniela, më 13 January 2023 në 04:23

Hello Daniela, you can use Sideral forte 30 mg, you can start with 1 capsule/day. If it is not tolerated, treatment with iron preparations with serum will be evaluated, but these are only done in hospital conditions

Replay from Dr. Shk. Sotiraq Lako, më 14 January 2023 në 02:13

Doctor, does an iron infusion with a ferritin level of 22 happen?
Where is it done and what side effects does it have?

Sent by Daniela, më 13 January 2023 në 04:33

Hello Daniela, iron with serum is done only in hospital conditions, due to the fact that sometimes it is accompanied by allergic reactions

Replay from Dr. Shk. Sotiraq Lako, më 14 January 2023 në 02:15

Doctor, hello! Happy New Year and may it be a very good one!
I am 32 years old and 5 weeks pregnant.
I did the blood tests and these are the results: leukocytes 7.16, erythrocytes 4.58, hemoglobin 11.7, ferritin 6.4, hematocrit 35.9, MCV 78.4, MCH 25.6, MCHC 32.6, RDW-SD 38.8, RDW-CV 16.7.
What do you recommend for me to take considering that I am in the early stages of pregnancy?
Currently, I am taking 5 mg of folic acid and Vitafusion prenatal vitamins, which do not contain iron.
Waiting for your response.
Thank you!

Sent by Ana, më 13 January 2023 në 06:45

Hello Ana, you have Mild Anemia due to Iron Deficiency. Of course, it needs to be treated, the vitamins you are taking are unrelated to it. The most suitable preparation would be Ironorm, for at least 2 consecutive months. You will discuss the dosage and side effects with the doctor who is treating you

Replay from Dr. Shk. Sotiraq Lako, më 14 January 2023 në 02:16

Hello Doctor! I have an 18-year-old daughter and after performing the tests, the results are WBC 7.23, RBC 4.25, HGB 12.5, MCV 86.4, MCH 29.4, MCHC 34.1, RDW-SD 38.8, RDW-CV 12, Ferritin 19.32. She is being treated with Feramax 150 mg, 1 capsule a day, accompanied by Vit C.
Please, is this the right dosage she is taking and for how long should she continue?

Sent by Teuta, më 19 January 2023 në 05:08

Hello Teuta, the girl is diagnosed with Iron Deficiency, without anemia. Feramax does not replenish the iron stores

Replay from Dr. Shk. Sotiraq Lako, më 20 January 2023 në 03:57

Hello Prof Lako. I did some blood tests and the results are these: Rbc 4.80, hemoglobin 13.9, hematocrit 41.80, mcv 87.10, mch 29, mchc 33.3, plt 248, rdw sd 41, rdw cv 13, iron 94.42, ferritin 22. Age 40 years. The family doctor advised me to take globihem vials only during cycle days but I want to increase my iron stores. What do you suggest? Have a good weekend

Sent by Majlinda, më 21 January 2023 në 10:14

Hello Majlinda, you have Ferritinemia at the lower limits of the norm (20-60/70 ng/ml). You need to take medication to increase your iron stores. Globihem does not fill the stores, the best preparation we have is Heferol

Replay from Dr. Shk. Sotiraq Lako, më 21 January 2023 në 12:28

Hello Dr. Lako. I wanted to ask you a question: Can Ferramax be used to avoid falling into a ferritin deficiency? I am at the lower limit of the norm, 22 ng/ml. And if so, how often do you think it can be taken daily in capsule form? Does it have an impact on the gastrointestinal tract and if so, how far from food should it be taken? Thank you for any response you provide

Sent by Majlinda, më 26 January 2023 në 13:29

Hello Majlinda, FeraMax does not replenish iron stores

Replay from Dr. Shk. Sotiraq Lako, më 29 January 2023 në 12:48

Hello Dr. Lako. I wanted to ask you if Ferramax can be used to prevent falling into a ferritin deficiency. I am at the lower limit of the normal range, 22 ng/ml. And if so, how often do you think it can be taken daily in capsule form? Does it have an effect on the gastrointestinal tract, and if so, how far from food should it be taken? Thank you for any response of yours

Sent by Majlinda, më 27 January 2023 në 05:26

Hello Majlinda, FeraMAx does not replenish the iron stores

Replay from Dr. Shk. Sotiraq Lako, më 29 January 2023 në 12:48

Hello Dr. Lako. I wanted to ask you a question, can Ferramax be used to prevent falling into a ferritin deficiency? I am at the lower limit of the norm, 22 ng/ml. And if so, how often do you think it can be taken in capsule form? Does it affect the gastrointestinal tract, and if so, how far from food should it be taken? Thank you for any response of yours

Sent by Majlinda, më 27 January 2023 në 07:35

Hello Majlinda, FeraMAx does not replenish the iron stores

Replay from Dr. Shk. Sotiraq Lako, më 29 January 2023 në 12:48

Hello Dr. Lako, I hope you are well. I see many messages addressed to you regarding iron deficiency and the anemia caused by it or not. Please, I would like to know why the first thing doctors in Albania do is fill prescriptions with iron salts or various iron medications without addressing the reason for this deficiency/excess. I see many foreign doctors saying the cause must be found. You are the only doctor I have read who says anemia is just the tip of the iceberg. Respectfully, could you mention some of the reasons why ferritin might be low and what tests should be done? And why in Albania is the minimum norm 13 when abroad it is 40 as the minimum norm?
2. And if we take iron medication, it will drop again, so why should we waste time and monetary expenses when the reason is not sought by doing various tests?
3. Why aren't specific analyses done in health centers and state hospitals? Why are we so behind in Albania?
4. Can iron supplements or just iron be taken when hemoglobin, platelets, and RBCs are normal? Do these additions affect their increase, or can iron increase hemogram values?
Thank you for your answer.
I went on a bit, but I believe everyone should be more responsible for their own body. Not just get a prescription, take the medicine, and then the problems return because the main reason is not solved. All the best to you

Sent by Ina, më 28 January 2023 në 03:57

Hello Ina, continuing with your questions: Iron Deficiency Anemia is a consequence of Iron Deficiency. Most people have Iron Deficiency, even without being associated with Iron Deficiency Anemia. Certainly, it should be treated according to standards, especially in the clinic you have, to prevent the overlap of anemia, which would be accompanied by the respective clinic and the doubling of the treatment time (with the same preparations and the same dose) and what is more important is to discover the cause/causes of the iron deficiency. Iron Deficiency Anemia, has its cause, Iron Deficiency (as the name itself suggests)

Replay from Dr. Shk. Sotiraq Lako, më 30 January 2023 në 10:00

Hello Dr. Lako. I hope you are doing well. I see many messages addressed to you about iron deficiency and the anemia caused by it, whether related or not. Please, I would like to know why the first thing doctors in Albania do is fill prescriptions with iron salts or various iron medications without addressing the reason for this deficiency/excess. I see many foreign doctors saying the cause must be found. You are the only doctor I have read who says anemia is just the tip of the iceberg. Kindly, could you mention some of the reasons why ferritin might be low and what tests should be done? And why in Albania the minimum norm is 13 when abroad it is 40 as the minimum norm?
2. And if we take iron medication, will it drop again, so why should we waste time and money when the reason is not sought by doing different analyses?
3. Why aren't specific analyses done in health centers and state hospitals? Why are we so behind in Albania?
4. Can iron supplements or iron itself be taken when hemoglobin, plt, rbc are within the normal range? Do these supplements affect their increase, or can iron increase the values of the hemogram?
Thank you for your answer.
I went on, but I believe that everyone should be more responsible for their body. Not just get a prescription, take the medicine, and then the problems return because the main reason is not solved. All the best, regards

Sent by Ina, më 29 January 2023 në 17:21

Hello Ina, following up on your questions: Iron Deficiency Anemia is a consequence of Iron Deficiency. Most people have Iron Deficiency, even without being associated with Iron Deficiency Anemia. Of course, it needs to be treated according to standards, because of the clinic it has, to prevent the overlaying of anemia, which would be associated with the respective clinic and doubling the treatment time (with the same preparations and the same dosage) and what is most important is to discover the cause/causes of iron deficiency. Iron Deficiency Anemia, has the cause, Iron Deficiency (as the name itself suggests)

Replay from Dr. Shk. Sotiraq Lako, më 30 January 2023 në 10:00

Doctor, hello. I wanted to know where the difference lies between Heferol 350 mg (115 mg elemental iron), Sideral Forte 30 mg elemental iron, and Feramax (150 mg elemental iron). I'm unclear because my family doctor advises that Heferol is strong and has significant side effects. She recommended I take Feramax for 1 month! Ferritin 24.06, HGB, RBC, PLT, HCT, MCV, MCH, MCHC, RDWS are all normal. I don't know what to do or what to take. Most doctors I've consulted say I don't need it, just eat more. Meanwhile, when I show them your articles, they say it's not true that these products replenish the stores. Please, can you explain the differences because I want to be prepared for my next consultation with the doctor (it's better we replace irresponsible doctors ourselves). Thank you

Sent by Elisa, më 30 January 2023 në 19:09

Hello Elisa, the rules that serve to replenish iron are the use of divalent iron, where the best salts are Ferrous Sulfate and Ferrous Fumarate (Heferol), the medicinal dose is 180-200 mg of elemental iron per day (2 Heferol capsules = 230 mg), for at least 6 weeks

Replay from Dr. Shk. Sotiraq Lako, më 01 February 2023 në 07:02

Doctor, hello. I wanted to know what the difference is between Heferol 350 mg (115 mg elemental iron), Sideral forte 30 mg elemental iron, and Feramax (150 mg elemental iron). I am unclear since my family doctor advises that Heferol is strong and has significant side effects. She recommended that I take Feramax for 1 month! Ferritin 24.06, HGB, RBC, PLT, HCT, MCV, MCH, MCHC, RDWS all within the norm. I don't know what to do or what to take. Most doctors I have consulted tell me I don't need anything, just to eat more. Meanwhile, when I show them your writings, they say it's not true that these products replenish the stores. Please, can you explain the differences because I want to go prepared to my next consultation with the doctor (it's better to replace irresponsible doctors ourselves). Thank you

Sent by Elisa, më 31 January 2023 në 04:54

Hello Elisa, the rules that serve to replenish iron are the use of divalent iron, where the best salts are Ferrous Sulfate and Ferrous Fumarate (Heferol), the therapeutic dose is 180-200 mg of elemental iron per day (2 Heferol capsules = 230 mg), for at least 6 weeks

Replay from Dr. Shk. Sotiraq Lako, më 01 February 2023 në 07:02

Doctor, hello. I wanted to know where the difference lies between Heferol 350 mg (115 mg elemental iron), Sideral Forte 30 mg elemental iron, and Feramax (150 mg elemental iron).
I'm unclear because my family doctor advises that Heferol is strong and has significant side effects. She recommended that I take Feramax for one month! Ferritin 24.06, Hgb, RBC, PLT, HCT, MCV, MCH, MCHC, RDWS all within normal range. I don't know what to do or what to take. Most of the doctors I have consulted tell me I don't need it, just eat more. Meanwhile, when I show them your writings, they say it's not true that these products fill up the stores. Please, could you explain the differences because I want to go prepared to my next consultation with the doctor (we might as well replace irresponsible doctors ourselves). Thank you

Sent by Elisa, më 31 January 2023 në 07:22

Hello Elisa, the rules that serve to replenish iron are the use of divalent iron, where the best salts are Ferrous Sulfate and Ferrous Fumarate (Heferol), the medicinal dose is 180-200 mg elemental iron per day (2 Heferol capsules = 230 mg), for at least 6 weeks

Replay from Dr. Shk. Sotiraq Lako, më 01 February 2023 në 07:02

Doctor, hello. I wanted to know where the difference lies between Heferol 350 mg (115 mg elemental iron), Sideral Forte 30 mg elemental iron, and Feramax (150 mg elemental iron).
I'm unclear since my family doctor advises that Heferol is strong and has significant side effects. She recommended taking Feramax for 1 month! Ferritin 24.06, Hgb, RBC, PLT, HCT, MCV, MCH, MCHC, RDWS are all within the normal range. I don't know what to do or what to take. Most doctors I have consulted tell me I don't need it, just to eat more. Meanwhile, when I show them your writings, they say it's not true that these products replenish the stores. Please, could you explain the differences because I want to go prepared to my next consultation with the doctor (it's better we replace irresponsible doctors ourselves). Thank you

Sent by Elisa, më 31 January 2023 në 08:00

Hello Elisa, the rules that serve to replenish iron are the use of divalent iron, where the best salts are Ferrous Sulfate and Ferrous Fumarate (Heferol), the medicinal dose is 180-200 mg elemental iron per day (2 Heferol capsules = 230 mg), for at least 6 weeks

Replay from Dr. Shk. Sotiraq Lako, më 01 February 2023 në 07:02

Doctor Hello. I wanted to know where the difference lies between Heferol 350 mg (115 mg elemental iron), Sideral Forte 30 mg elemental iron, and Feramax (150 mg elemental iron).
I am unclear because my family doctor advises me that Heferol is strong and has significant side effects. She advised me to take Feramax for 1 month! Ferritin 24.06 hgb rbc plt hct mvc mch mchc rdws all within the norm. I don't know what to do or what to take. Most of the doctors I have consulted tell me there's no need, just eat more. Meanwhile, when I show them your writings, they say it's not true that these products replenish the stores. Please could you explain the differences because I want to go prepared to the doctor at the next consultation (better we replace the irresponsible doctors ourselves). Thank you

Sent by Elisa, më 31 January 2023 në 15:44

Hello Elisa, the rules for replenishing iron are the use of divalent iron, where the best salts are Ferrous Sulfate and Ferrous Fumarate (Heferol), the medicinal dose is 180-200 mg of elemental iron per day (2 Heferol capsules = 230 mg), for at least 6 weeks

Replay from Dr. Shk. Sotiraq Lako, më 01 February 2023 në 07:02

Doctor, hello. I wanted to know what the difference is between Heferol 350 mg (115 mg elemental iron), Sideral Forte 30 mg elemental iron, and Feramax (150 mg elemental iron). I am unclear because my family doctor advises that Heferol is strong and has significant side effects. She recommended I take Feramax for 1 month! Ferritin 24.06, Hgb, RBC, PLT, HCT, MCV, MCH, MCHC, RDWS are all normal. I don't know what to do or what to take. Most doctors I have consulted tell me I don't need it, just eat more. Meanwhile, when I show them your writings, they say it's not true that these products replenish the stores. Please, can you explain the differences because I want to go prepared to my next consultation with the doctor (we better replace irresponsible doctors ourselves). Thank you

Sent by Elisa, më 01 February 2023 në 19:00

Hello Elisa, the rules that serve to replenish iron are the use of divalent iron, where the best salts are Ferrous Sulfate and Ferrous Fumarate (Heferol), the medicinal dose is 180-200 mg of elemental iron per day (2 Heferol capsules = 230 mg), for at least 6 weeks

Replay from Dr. Shk. Sotiraq Lako, më 02 February 2023 në 04:05

Doctor, greetings. I wanted to know where the difference lies between Heferol 350 mg (115 mg elemental iron), Sideral Forte 30 mg elemental iron, and Feramax (150 mg elemental iron). I am unclear because my family doctor advises that Heferol is strong and has significant side effects. She advised me to take Feramax for 1 month! Ferritin 24.06, hemoglobin (HGB), red blood cells (RBC), platelets (PLT), hematocrit (HCT), mean corpuscular volume (MCV), mean corpuscular hemoglobin (MCH), mean corpuscular hemoglobin concentration (MCHC), red cell distribution width (RDW) are all normal. I don't know what to do or what to take. Most doctors I have consulted tell me I don't need anything, just to eat more. Meanwhile, when I show them your writings, they say it's not true that these products replenish the stores. Please, could you explain the differences to me because I want to go prepared to my next consultation with the doctor (it's better we replace irresponsible doctors ourselves). Thank you

Sent by Elisa, më 02 February 2023 në 07:57

Hello Elisa, the rules that serve to replenish iron are the use of divalent iron, where the best salts are Ferrous Sulfate and Ferrous Fumarate (Heferol), the medicinal dose is 180-200 mg elemental iron per day (2 Heferol capsules = 230 mg), for at least 6 weeks

Replay from Dr. Shk. Sotiraq Lako, më 03 February 2023 në 08:34

Doctor, hello. I wanted to know where the difference lies between Heferol 350 mg (115 mg elemental iron), Sideral forte 30 mg elemental iron, and Feramax (150 mg elemental iron). I am unclear because my family doctor advises me that Heferol is strong and has significant side effects. She recommended that I take Feramax for 1 month! Ferritin 24.06, HGB, RBC, PLT, HCT, MCV, MCH, MCHC, RDWS are all normal. I don't know what to do or what to take. Most of the doctors I have consulted tell me I don't need it, just to eat more. Meanwhile, when I show them your writings, they say it's not true that these products fill the stores. Please, could you explain the differences because I want to go prepared to my next consultation with the doctor (we better replace irresponsible doctors). Thank you

Sent by Elisa, më 02 February 2023 në 16:54

Hello Elisa, the rules that serve to replenish iron are the use of divalent iron, where the best salts are Ferrous Sulfate and Ferrous Fumarate (Heferol), the medical dose is 180-200 mg elemental iron per day (2 Heferol capsules = 230 mg), for at least 6 weeks

Replay from Dr. Shk. Sotiraq Lako, më 03 February 2023 në 08:35

Hello Doctor Lako, I don't know how aware you are, but recent studies in France have shown that the drug tardyferon is very harmful to the digestive tract, especially to the intestine, stating that all sulfate salts have severe effects (inflammatory bowel disease) on the organism. And according to the latest studies, they say that every dose of iron increases the hormone Hepcidin (automatically blocks the absorption of iron for 24 hours). Unabsorbed iron, the studies have said, brings side effects such as diarrhea/constipation, vomiting, and nausea etc. My question is why take two doses a day when there are side effects and absorption is weaker, and why not take one capsule a day? And my second question is why not prefer irons like: gluconate and bisglycinate??? Thank you

Sent by Jeteza, më 03 February 2023 në 10:48

Hello Life, we know that only a part of the iron taken orally is absorbed; the excess will be eliminated with feces. A portion of the iron that is not absorbed remains in the mucosa of the duodenum causing local inflammation, which results in the release of histidine and a decrease in absorption for subsequent intakes. Studies show that taking iron every other day ensures a larger fraction of absorption than taking it daily, but the absolute amount is much smaller. I have tried with 1/2 of the recommended dose, or every 2 days, and the main goal, the replenishment of iron stores (which to date is achieved only with Ferrous Sulfate and Ferrous Fumarate, with a therapeutic dose of 180-200 mg of elemental iron per day) is not reached. In 20% of cases, it may be accompanied by gastrointestinal disorders. And the best hematological treatises continue to value classical treatment, passing over the alternative daily intake and smaller doses of iron with one paragraph. Iron Therapy Oral ferrous iron salts are the most economical and effective medication for the treatment of iron deficiency anemia. Of the various iron salts available, ferrous sulfate is the one most commonly used. Although the traditional dosage of ferrous sulfate is 325 mg (65 mg of elemental iron) orally three times a day, lower doses (e.g., 15-20 mg of elemental iron daily) may be as effective and cause fewer side effects. To promote absorption, patients should avoid tea and coffee and may take vitamin C (500 units) with the iron pill once daily. [18] However, a randomized trial in 140 adult patients with iron deficiency anemia found that oral iron taken alone and oral iron taken with 200 mg of vitamin C produced equivalent increases in hemoglobin and serum ferritin levels and equivalent rates of adverse events. [19] However, a study by Moretti et al suggests that the standard dosing of iron supplements may be counterproductive. Their research focuses on the role of hepcidin, which regulates systemic iron

Replay from Dr. Shk. Sotiraq Lako, më 04 February 2023 në 09:15

Hello Dr. Lako, I don't know how aware you are, but recent studies in France have shown that the drug tardyferon is very harmful to the digestive tract, especially to the intestine, stating that all sulfate salts have severe effects (inflammatory bowel disease) on the organism. And according to the latest studies, they say that every dose of iron increases the hormone Hepcidin (automatically blocks iron absorption for 24 hours). Unabsorbed iron, the studies have said, brings side effects such as diarrhea/constipation, vomiting, and nausea, etc. My question is why take two doses a day when there are side effects and absorption is weaker, and why not take one capsule a day? And the second question is why are irons like gluconate and bisglycinate not preferred??? Thank you

Sent by Jeteza, më 03 February 2023 në 16:47

Moretti et al concluded that providing lower dosages and avoiding twice-daily dosing will maximize fractional iron absorption, and that their results support supplementation with 40-80 mg of iron taken every other day. A possible additional benefit of this schedule may be that improving absorption will reduce gastrointestinal exposure to unabsorbed iron and thereby reduce adverse effects from supplements. Subsequent longer-term studies confirmed that in iron-depleted women, taking iron supplements daily in divided doses increases serum hepcidin and reduces iron absorption, whereas taking iron supplements on alternate days and in single doses optimizes iron absorption.

Stoffel et al also concluded that alternate-day dosing of oral iron supplements may be preferable because it sharply increases fractional iron absorption. In their study, conducted in 19 women with iron deficiency anemia, total iron absorption from a single 200-mg dose given on alternate days was approximately twice that from 100 mg given on consecutive days.

Claims are made that other iron salts (e.g., ferrous gluconate) are absorbed better than ferrous sulfate and have less morbidity. Generally, the toxicity is proportional to the amount of iron available for absorption. If the quantity of iron in the test dose is decreased, the percentage of the test dose absorbed is increased, but the quantity of iron absorbed is diminished

Replay from Dr. Shk. Sotiraq Lako, më 04 February 2023 në 09:17

Hello Dr. Lako, I don't know how aware you are, but recent studies in France have shown that the Tardyferon drug is very harmful to the digestive tract, especially to the intestine, stating that all sulfate salts have severe effects (inflammatory bowel disease) on the organism. And according to the latest studies, they say that every dose of iron increases the hormone Hepcidin (automatically blocks the absorption of iron for 24 hours). Unabsorbed iron, the studies have mentioned, brings side effects such as diarrhea/constipation, vomiting, and nausea, etc.
My question is why take two doses a day when there are side effects and absorption is weaker, and why not take one capsule a day? And the second question is why are irons like gluconate and bisglycinate not preferred??? Thank you

Sent by Jeteza, më 04 February 2023 në 06:48

Generally, the toxicity is proportional to the amount of iron available for absorption. If the quantity of iron in the test dose is decreased, the percentage of the test dose absorbed is increased, but the quantity of iron absorbed is diminished

Replay from Dr. Shk. Sotiraq Lako, më 04 February 2023 në 09:39

Currently, we are trying an iron preparation such as Sideral Forte with 30 mg of elemental iron - 2 capsules/day, compared to the traditional medication with 180-200 mg of elemental iron per day, to evaluate the goals of the treatment, the correction of anemia within 4-6 weeks and the correction of iron stores for an additional 6 weeks. If the objectives are not met, the medication is not suitable

Replay from Dr. Shk. Sotiraq Lako, më 04 February 2023 në 09:41

Doctor, hello. Is it true that an American study says that iron tablets increase the risk of colon cancer? The study found that even lower doses of EDTA, which are usually found in over-the-counter iron supplements, increase the levels of proteins associated with colon cancer. "We believe that iron supplementation could be carcinogenic, as it is the formation of ampergoline, a protein associated with long-term cancer with poor diagnosis," said Professor Sachs from the Chalmers University of Technology in Gothenburg. Therefore, iron tablets should be eliminated at all costs, and instead, iron should be supplemented from natural food sources. I wanted to discuss this study with you

Sent by Bruna, më 04 February 2023 në 06:52

Hello Bruna, there are studies that evaluate that the consumption of red meat, and red meat has its color from the heme ring, which has iron at its center, causes colon cancer. https://www.health.harvard.edu/staying-healthy/red-meat-and-colon-cancer

Replay from Dr. Shk. Sotiraq Lako, më 04 February 2023 në 09:25

Doctor Hello. Is it true a American study which says that iron tablets increase the risk for colon cancer? The study discovered that even the lower doses of
EDTA, which are usually found in the
over-the-counter iron supplements, increase the levels of proteins
associated with colon cancer. "We believe that iron supplementation could
be carcinogenic, as it is the formation of
ampelopsin, a protein that is associated with
long-term diagnosed poor prognosis cancer," said Professor Sachs of Chalmers University of
Technology in Gothenburg. Therefore,
iron tablets should be eliminated at all costs, and instead, iron
should be supplemented from natural food sources.
I wanted to discuss this study with you

Sent by Bruna, më 04 February 2023 në 09:12

Red meat and colon cancer
January 1, 2008
You are what you eat.
Diet has a powerful influence on many diseases, including America's number two killer, cancer. But because cancer is so complex, with many genetic and environmental factors affecting risk, the link between your menu and your risk has been hard to decipher. In the case of red meat and colon cancer, however, new research provides a plausible explanation for a long-suspected association.
Establishing a link
Although the results vary, studies from around the world have suggested that a high consumption of meat is linked to an increased risk of colon cancer. In some studies, fresh meat appears culpable; in others, it's processed, cured, or salted meat — but in all cases the worry is confined to red meat, not chicken.
The best evidence comes from a pair of large 2005 studies, one from Europe, the other from the United States. The European research tracked 478,000 men and women who were free of cancer when the study began. During nearly five years of follow-up, 1,329 people were diagnosed with colon cancer. The people who ate the most red meat (about 5 ounces a day or more) were about a third more likely to develop colon cancer than those who ate the least red meat (less than an ounce a day on average). Their consumption of chicken did not influence risk one way or the other, but a high consumption of fish appeared to reduce the risk of colon cancer by about a third. The effects of red meat and fish held up after the results were adjusted for other potential colon cancer risk factors, including body weight, caloric consumption, alcohol consumption, smoking, physical exercise, dietary fiber, and vitamins.
The U.S. study, sponsored by the American Cancer Society, added important information about the effects of long-term meat consumption. The subjects were 148,610 people between the ages of 50 and 74. Each person reported on his or her dietary patterns and health habits when the study began in 1982 and ag

Replay from Dr. Shk. Sotiraq Lako, më 04 February 2023 në 09:27

Hello, Doctor, my name is Jona, I am 26 years old.
I did a complete blood test and the results were these: WBC 6.8, RBC 4.5, Hgb 12, Hct 35.8, MCV 80, MCH 27.8, PLT 290, MPV 9.3. My LYMPH% came out to be 44.4% from the max that should be 40%, the rest are normal.
My Ferritin came out to be 6.8.
What treatment do you suggest because I am in a dilemma about what to use since every doctor tells me something different?

Sent by Jona, më 04 February 2023 në 09:57

Hello Jona, you have only an iron deficiency, which needs to be corrected. The best preparation in our country is Heferol

Replay from Dr. Shk. Sotiraq Lako, më 04 February 2023 në 11:39

Thank you Doctor for your response. Since the new sideral forte 30 mg iron is being tried, if you do not have anemia and your ferritin is 27 mg/nl, occult blood negative, and hemoglobin, platelets, erythrocytes, and the entire hematocrit are normal, can it be used just to fill up the iron stores? And if so, for how long and how many capsules per day?

Sent by Jeteza, më 04 February 2023 në 11:11

You can use it twice a day, for at least 6 consecutive weeks. Several cases need to be evaluated to see the effectiveness, the increase of ferritin levels in women to 60-70 ng/ml, which is the norm

Replay from Dr. Shk. Sotiraq Lako, më 04 February 2023 në 11:39

Professor, hello. I saw some of your interviews on YouTube, and one of the reasons mentioned for iron deficiency was cancer. Please, I would like to know the list of what tests need to be done to find out the reason for the iron deficiency. Thank you

Sent by Ina, më 20 February 2023 në 09:26

Hello Ina, for younger ages, the female gender, the cause is almost always related to the repeated loss of blood from the Menstrual Cycle. For older ages, one of the causes might be cancer, usually of the gastrointestinal or urinary tract

Replay from Dr. Shk. Sotiraq Lako, më 21 February 2023 në 08:08

Hello Professor. I am quite worried, to tell you the truth, about the responsibility of the doctors and us who trust them even more. I wanted to share a concern with you. After my family doctor recommended that I take feramax 150 mg for a month to keep my ferritin under control, which was 22.5 mg and hemoglobin 13.9. After a month of using the product (which is also expensive, moreover), the results of the analysis are as follows: ferritin 16.9, sideremia 158, RBC 4.71, hematocrit 40.6, hemoglobin 13.6, MCV 86.1, MCH 28.9, MCHC 33.5, RDW 14.7%, PLT 240, MPV 9.6, EOS# 0.44, EOS% 4.4.
What should I do? Thank you for any answer

Sent by Ajla, më 03 March 2023 në 14:11

Hello Ajla, you have Iron Deficiency and you can start Heferol

Replay from Dr. Shk. Sotiraq Lako, më 04 March 2023 në 10:36

Hello doctor. I have been suffering from iron deficiency for years. My cycle is frequent, every 21 days and I would describe the bleeding as terrifying, especially heavy for the first two days. I experience shortness of breath, anxiety, frequent heart palpitations without any reason, fatigue, tightness in the chest, and it feels like a muscle in my left arm is trembling. I have been checked by a cardiologist and had an echocardiogram. I also had a series of X-rays and was diagnosed with esophageal spasms. My ferritin is at 9, hemoglobin at 12. Do these symptoms come from iron deficiency and what treatment would you recommend, please. Respectfully, Evalina

Sent by Evalina Islamllari, më 04 March 2023 në 03:07

Hello Evalina, you haven't been suffering from Iron Deficiency for years, your problem lies with the significant and frequent blood losses through the Menstrual Cycle. Blood is red because of the heme group, which centrally contains iron. So, when you lose blood, you lose iron. Iron Deficiency is not a male issue, it's a female issue, especially during their reproductive period. If you don't reduce the blood loss and correct the iron deficiency, it will recur very quickly

Replay from Dr. Shk. Sotiraq Lako, më 04 March 2023 në 11:56

Hello doctor,
I have done ferritin and iron tests.
Ferritin has come out as 8.38.
Iron 5.6.
B12 489.4.

Symptoms include fatigue, weakness

Sent by Iliriana, më 21 March 2023 në 10:19

Hello Iliriana, your ferritin level is low. The minimum normal value is 20 ng/ml and the optimal value is around 60 ng/ml. You need to take medication to correct it

Replay from Dr. Shk. Sotiraq Lako, më 21 March 2023 në 10:50

Hello doctor, can iron infusions be done instead of pill medication. When ferritin is 16.9 mg but hemoglobin, platelets, and erythrocytes are within the normal range /nl since pill medication has failed. And if so, what are the side effects? Thank you

Sent by Jona, më 07 April 2023 në 06:29

Hello Jona, it's being done. We have two options: Ferrovin 100 mg = 5-6 infusions and Ferinject 500 mg 1 single infusion. Iron with serum is done only in hospital conditions (sometimes allergic reactions) and is expensive

Replay from Dr. Shk. Sotiraq Lako, më 08 April 2023 në 06:24

Hello Prof. Lako, I hope you are doing well in the first place. Meanwhile, I have been taking iron for 4 days, Ferapia 150 mg prescribed by the hematologist. Two weeks ago, I went through an enterovirus infection and stopped the medication because it gave me side effects (diarrhea) and I did some tests to see where my health is.
The analyses are as follows: wbc 9.79 rbc 4.78 hgb 13.7 hct 41.6 mcv 87 mch 28.7 mchc 32.9 plt 255 lymph 29.80 % mono 8.50% eo 1.0% baso 0.30% neut 60% rdw -cv 13.30 mpv 11.9
Among them, I did the analyses cea 3.9 <5.0
Total protein 7.13, LDH 64 (low), sideremia 33 (low)
Vitamins: folic acid 6.40 ferritin 22mg/nl vitamin D 10 (moderate value) vitamin b12 307 pg/ml
I don't know what to do anymore (I was treated for a week for fungus in feces), waited two weeks and did these analyses. Do I need other analyses to know what I have? Thank you and have a good day

Sent by Ina, më 10 April 2023 në 15:17

Hello Ina, iron treatment should continue to improve ferritinemia (the minimum normal value is 20 ng/ml and the optimal value is about 60 ng/ml). Vitamin D needs to be corrected. No other changes

Replay from Dr. Shk. Sotiraq Lako, më 11 April 2023 në 03:41

Hello doctor. Some time ago, I wrote to you about iron deficiency, and you told me to use heferol for 3 months along with vitamin C. I used it for 1 month and you told me to do the tests.. I did them and my Hb was 13.4 while ferritin was 14.42.. I continued for another 2 months and did the tests again and my Hb was 13.7 and ferritin 52.5.. is this good or should I continue the treatment?

Sent by Anxhela Kryemadhi, më 27 April 2023 në 12:49

Hello Anxhela, you have been corrected in terms of hemoglobin and iron reserves. You can stop the medication and after a few months, recheck your ferritin levels. Due to blood loss in women with the Menstrual Cycle, there is a possibility for it to recur

Replay from Dr. Shk. Sotiraq Lako, më 28 April 2023 në 03:03

Hello doctor. I wrote to you some time ago about my iron deficiency and you told me to use heferol for 3 months along with vitamin C. I used it for 1 month and you told me to get tested. I did, and my Hgb was 13.4 while ferritin was 14.42. I continued for another 2 months and got tested again, this time Hgb was 13.7 and ferritin was 52.5. Is this good or do I need to continue treatment?

Sent by Anxhela Kryemadhi, më 27 April 2023 në 23:38

Hello Anxhela, you have been corrected both in terms of hemoglobin and iron reserves. You can stop the medication and after a few months, recheck ferritinemia. Due to blood loss in females with the Menstrual Cycle, the possibility for recurrence exists

Replay from Dr. Shk. Sotiraq Lako, më 28 April 2023 në 03:03

Hello doctor. Some time ago, I wrote to you about iron deficiency, and you told me to use heferol with vitamin C for 3 months. I used it for 1 month and you told me to get my tests done.. I did, and my HGB was 13.4 while my ferritin was 14.42.. I continued for another 2 months and did the tests again and my HGB was 13.7 and ferritin 52.5.. is this good or should I continue the treatment?

Sent by Anxhela Kryemadhi, më 28 April 2023 në 02:03

Hello Anxhela, you have been corrected in terms of hemoglobin and iron stores. You can stop the medication and after a few months recheck your ferritinemia. Due to blood loss in women with the Menstrual Cycle, the possibility of recurrence exists

Replay from Dr. Shk. Sotiraq Lako, më 28 April 2023 në 03:03

Hello doctor! My 11-year-old daughter has a ferritin level of 8.74. Please tell me what the proper treatment for her would be

Sent by Irena muca, më 02 June 2023 në 18:53

Hello, it depends on the weight of the pot, the presence or absence of the menstrual cycle, its amount, whether you can use the tablets, etc

Replay from Dr. Shk. Sotiraq Lako, më 04 June 2023 në 04:47

Hello, doctor!
My 82-year-old mother had a check-up and the results show these data: rbc 3.46, hgb 12.6, hct 34.1, mch 36.3, erythrocyte sedimentation rate 20, alkaline phosphatase 38. The other data are fine. Thank you for your response! Have a good day!

Sent by Rudina, më 09 June 2023 në 02:06

Hello Rudina, probably a change related to vitamin deficiency. 1. Folic Acid 5 mg 3 x 1 tablet/day – 90 tablets/month. 2. Vitamin B Complex 3 x 1 tablet/day – 90 tablets/month. After 1 month, follow up with a complete blood test

Replay from Dr. Shk. Sotiraq Lako, më 09 June 2023 në 02:50

Hello Doctor! First of all, I want to thank you for your dedication. If you could clarify a bit about the results of the following analyses, I would be very grateful. My daughter is 18 years old and has had an iron deficiency (Ferritin 4 ng/ml). She has been medicated, but lately, she has been complaining a lot about fatigue, dizziness, darkening of vision, and pain in the back of the head. The results of the analyses are as follows: WBC 8.01 RBC 4.02 HGB 11.8 HCT 34.6 MCV 86.1 MCH 29.4 MCHC 34.1 PLT 213 RDW-SD 38.4 RDW-CV 12.1 PDW 12.6 MPV 10.7 P-LCR 30.6 PCT 0.23 NEUT 38.5% LYMPH 52.6% MONO 7.2% EO 1.1% BASO 0.5% Erythrocyte sedimentation rate 14 and Ferritin 44.27 ng/ml. The girl has been using Feramax 150 mg. Please, is there any problem, are these concerns due to iron, and what could she use? Thank you and have a nice day!

Sent by Teuta, më 07 July 2023 në 05:08

Hello Teuta, if the ferritin level is 44, we consider it normal, even though we aim for 60 ng per ml. You can take iron for another month. Feramax does not replenish the stores. Heferol is more recommended

Replay from Dr. Shk. Sotiraq Lako, më 07 July 2023 në 05:23

Hello Doctor! I have done the tests and the indexes not within the norm were;
HCT (Hematocrit) 35.9
BASO% 1.21%
PDW 19.1
MPV 6.05

And the EKG had these results
1- SMI age undetermined
2- Normogram
3- Anterior subendocardial ischemia
Thank you for the opportunity to consult with you.
With respect, Ediola

Sent by Ediola, më 12 July 2023 në 13:06

Hello Ediola, the parameters are normal. Discuss the EKG with the cardiologist

Replay from Dr. Shk. Sotiraq Lako, më 12 July 2023 në 13:59

Hello doctor. I have done some blood tests and these are the results. WBC 7.1, lymphocytes 2.0, monocytes 0.6, granulocytes 4.6, lymphocytes% 27.9, monocytes% 8.0, granulocytes% 64.1, RBC 4.57, hemoglobin 12.8, hematocrit 39.6, MCV 86.7, MCH 28.0, MCHC 32.3, RDW 14.5, platelets 277, MPV 8.2, PCT 0.227, PDW 15.9. Leukocyte formula segments 57%, lymph 29, mon 10, eos 1%, bas 0%, bands 3%. ESR 17, ferritin 8.6, serum iron 64, B12 204, CRP 0.09. I am 50 years old. What do you think about these tests and if I can have a hospital consultation with you, doctor. Thank you, Harlet Olldashi

Sent by Harlet olldashi, më 01 October 2023 në 07:28

Hello Harlet, you only have Iron Deficiency, for which you need to take medication

Replay from Dr. Shk. Sotiraq Lako, më 02 October 2023 në 10:59

Hello, my daughter is 2 years and 4 months old and she has lactose intolerance. She has been found to have an iron deficiency and is using Ferrosil. She has developed some pimples on her face and I don't know if the syrup might contain lactose. If you could let me know, I would appreciate it

Sent by Liridona, më 05 January 2024 në 16:56

Hello Liridona, I do not treat such young children, because I am a hematologist for adults. Pediatricians, pediatric hematologists, are the ones who can help you

Replay from Dr. Shk. Sotiraq Lako, më 06 January 2024 në 05:57

Hello doctor..I have an 18-month-old daughter..her hemoglobin is 9, serum iron 31..
She has been treated with ferrodep 150mg for a month..how long do I need to use this for her?? And is it enough?

Sent by Klaudia, më 09 January 2024 në 05:47

Hello Klaudia, I do not treat children. You will discuss with the pediatrician. Essentially, the goal is to correct anemia and ferritin levels. When the latter is normal, treatment is stopped

Replay from Dr. Shk. Sotiraq Lako, më 09 January 2024 në 06:09

Hello doctor!
I have a 3-and-a-half-year-old daughter for whom I've done electrophoresis, and these values came out. Hb A 94.5. Hb F 3.3
Hb A2 2.2. They told me that my daughter has thalassemia minor. But I emphasize that neither my spouse nor I are thalassemic. Could you give me your opinion, please? Thank you!

Sent by Anxhela, më 01 February 2024 në 14:43

Hello Anxhela, we usually do the hemoglobin electrophoresis after the age of 4 years, because it is the time when HbF reaches its lowest values. The best indicator for Thalassemia Minor is HbA2 > 3.5% and your daughter has it within normal range. She probably doesn't have it, but I also need the values of erythrocytes and hemoglobin. If both you and your spouse do not have Thalassemia Minor, there is no way your child can have it

Replay from Dr. Shk. Sotiraq Lako, më 02 February 2024 në 07:05

Hello doctor!
I have a three and a half year old daughter for whom I have done electrophoresis and these values have come out. Hb A 94.5. Hb F 3.3
Hb A2 2.2. I have been told that my daughter has thalassemia minor. But I emphasize that neither my spouse nor I are thalassemic. Can you give me your opinion, please? Thank you!

Sent by Anxhela, më 01 February 2024 në 16:04

Hello Anxhela, we usually do hemoglobin electrophoresis after the age of 4 years, because it is the time when HbF reaches its lowest values. The best indicator for Thalassemia Minor is HbA2 > 3.5%, and the girl's values are normal. She probably doesn't have it, but I also need the values of erythrocytes and hemoglobin. If you and your spouse don't have Thalassemia Minor, there's no way your child could have it

Replay from Dr. Shk. Sotiraq Lako, më 02 February 2024 në 07:06

Hello, doctor. I am 42 years old and in the tests done, my hemoglobin came out with a value of 12 while ferritin 16. I have body fatigue, lack of energy, headache. What do you recommend, please? Thank you!

Sent by Anduela, më 23 February 2024 në 11:58

Hello Anduela, you have an iron deficiency, which might justify your complaints. You will be treated with iron supplements. The best one is Heferoli

Replay from Dr. Shk. Sotiraq Lako, më 23 February 2024 në 14:49

Hello, doctor! From the complete blood test, these deviations result: BASO% 1.3, NEUT% 64.4, PCT 0.36 (male, age 50 years). Do these data indicate anything serious? (other data are within normal limits). Thank you

Sent by Rudina, më 27 February 2024 në 06:58

Hello Rudina. Normal value

Replay from Dr. Shk. Sotiraq Lako, më 27 February 2024 në 08:56

Hello, doctor! From the complete blood count, the following deviations are found: BASO% 1.3, NEUT% 64.4, PCT 0.36 (male gender, age 50 years). Do these data indicate anything serious? (other data are within normal limits). Thank you

Sent by Rudina, më 27 February 2024 në 07:02

Hello Rudina. normal value

Replay from Dr. Shk. Sotiraq Lako, më 27 February 2024 në 08:56

Dr. good evening. I hope you are well. I am 52 years old. From the tests, it resulted in Sideremia (Iron)* 51.75 μg/dl, while Ferritin (Serum)* 146.8. Should I take any supplements and two dietary tips if it can be recovered with diet. thank you very much

Sent by Rezart, më 28 February 2024 në 13:00

Hello Rezart, the ferritin level is normal

Replay from Dr. Shk. Sotiraq Lako, më 28 February 2024 në 14:38

Hello Rezart, the ferritinemia is normal

Replay from Dr. Shk. Sotiraq Lako, më 28 February 2024 në 14:39
Survey

Would you conduct an online paid consultation with a doctor?

Send vote